math 1ere stmg

120
M a t h é m a t i q u e s Livre du professeur Lydia Misset • Marie-Andrée Belarbi STMG

Upload: quel-quun

Post on 02-Jan-2016

6.399 views

Category:

Documents


6 download

TRANSCRIPT

Page 1: Math 1ere STMG

Mathématiq

uesLivre du professeur

Lydia Misset • Marie-Andrée Belarbi STM

G

978-2-01-182123-2_STMG_LDP_PDT.indd 1 20/07/12 10:46

Page 2: Math 1ere STMG

Mise en pages : IDTCouverture : Oxygène Multimédia

www.hachette-education.com© Hachette Livre 2012, 43 quai de Grenelle, 75905 ParisISBN 978-2-01-182123-2

Tous droits de traduction, de reproduction et d’adaptation réservés pour tous pays.Le Code de la propriété intellectuelle n’autorisant, aux termes des articles L. 122-4 et L. 122-5, d’une part, que les « copies ou reproductions strictement réservées à l’usage privé du copiste et non destinées à une utilisation collective », et, d’autre part, que « les analyses et les courtes citations » dans un but d’exemple et d’illustration, « toute représentation ou reproduction intégrale ou partielle, faite sans le consentement de l’auteur ou de ses ayants droit ou ayants cause, est illicite ».Cette représentation ou reproduction, par quelque procédé que ce soit, sans autorisation de l’éditeur ou du Centre français de l’exploitation du droit de copie (20 rue des Grands-Augustins, 75006 Paris), constituerait donc une contrefaçon sanctionnée par les articles 425 et suivants du Code pénal.

Sur le site www.hachette-education.com, les fichiers signalés par dans le manuel sont disponibles en téléchargement pour les enseignants.

En complément

Page 3: Math 1ere STMG

Sommaire

Chapitre 1 : Pourcentages 5

Chapitre 2 : Second degré 19

Chapitre 3 : Statistique 35

Chapitre 4 : étude de fonctions 47

Chapitre 5 : Probabilités 69

Chapitre 6 : Suites numériques 83

Chapitre 7 : Loi binomiale – échantillonnage 99

Page 4: Math 1ere STMG
Page 5: Math 1ere STMG

Chapitre 1 pourCentages 5

Chapitre 1 Pourcentages

Intentions des auteuresVu la structure et le nombre de rubriques de chaque chapitre, nous avons fait le choix de faire peu de chapitres, ce qui permet un ouvrage de seulement 208 pages.Si ce premier chapitre est assez long, il a été conçu pour être facilement coupé en deux parties : les pourcentages de sous-populations (parties 1 à 3) et les pourcentages d’évolution (parties 4 et 5). Le rangement des exercices, selon l’ordre du cours, facilite ce découpage. De même les deux premiers Ateliers TICE sont liés aux trois premières parties du chapitre. Quant aux exercices d’approfondissement, pour la plu-part, ils utilisent la totalité du chapitre. Nous avons fait le choix de faire étudier la pre-mière rubrique du programme « Feuilles auto-matisées de calcul » dans tous les chapitres au fur et à mesure. Si les ateliers TICE se prêtent à cet apprentissage, nous avons présenté ces outils dans tous les chapitres, aussi bien dans le cours ou mise en pratique que dans les exercices, afin de diversifier la pratique de ces techniques. Le travail sur les tableaux croisés est abordé dans les exercices d’approfondisse-ment et Vers le Bac.Les QCM Avant de commencer permettent de revenir sur la notation % et le lien avec leur utilisation dans la vie quotidienne. Nous avons choisi d’éviter autant que faire se peut la nota-tion 0,2 = 20 % . Cette notation était écrite dans le programme de STG précédent. Elle ne fait pas l’unanimité chez les collègues. Nous pensons que le symbole % doit être réservé au langage parlé et aux conclusions des exercices.

Avant de commencer

Réponses fausses aux QCM 1. a) 2. c) 3. b) 4. a) 5. c)

Cours

Proportion dans un ensemble Introduction du vocabulaire des proportions : part, taux, rapport, fréquence, mais aussi des ensembles ou population au sens large : sous-populations, ensemble de référence, total, élé-ment d’un ensemble…L’écriture d’une proportion est vue en frac-tion, en décimale et en pourcentage. Le choix de la notation p pour une proportion va per-mettre de conserver cette notation, y com-pris dans le Chapitre7 sur l’échantillonnage et le programme de Terminale. Elle facilite le rapprochement avec les probabilités vues en Seconde et dans le Chapitre 5. Nous avons jugé bon de différentier la pro-portion p , nombre entre 0 et 1 , et le taux t en pourcentage t = 100 × p .Il sera bon de voir les collègues de gestion et économie de la classe pour harmoniser le vocabulaire et les notations.Pour les conclusions, nous conseillons de tou-jours indiquer l’ensemble de référence afin d’habituer les élèves à se poser la question « un pourcentage de quoi ? ».

➜ page 190 des Techniques de base

Réponses de l’Étude de situation p. 10a) 18 élèves de 17 ans : 18/32 = 0,5625 , soit 56,25 % de la classe .b) 8 filles de 17 ans : 8/18 ≈ 0,4444 , soit 44,44 % des élèves de 17 ans.c) 12 garçons : 12/32 = 0,375 , soit 37,5 % de la classe.d) 10 garçons de 17 ans : 10/12 ≈ 0,8333, soit 83,33 % des garçons.e) Non.

Corrigés des Exercices d’application p. 11

1 a) T = 12 9979

0, 797 ≈ 163 085,3

soit 163 085 candidats reçus.

1

Page 6: Math 1ere STMG

6 Chapitre 1 pourCentages

b) Le taux d’échec est de 15 % .

T = 12

0,15 = 80

soit 80 candidats dans ce lycée.

3 a) +9

21 9 ≈ 0,3

ou 30 % de places assises.b) La proportion de places debout :100 – 30 = 70 , soit 70 %

ou bien 2130

= 0,7  soit 70 % .

4 Les deux pourcentages 23 % et 54 % proportions de la vente de bêtises Afchain dans l’ensemble des ventes de bonbons, ne portent pas sur la même ensemble de réfé-rence.

Si la deuxième boutique ne vend que 100 kg de bonbons, elle vend 54 kg de bêtises, c’est moins que chez Balile.

Réunion et intersection de sous-populations

Le but est d’introduire les notations A ∪ B et A ∩ B . Déjà vues en Seconde, mais trop sou-vent mal comprises, et de faire travailler « et » et « ou ».

➜ page 200 des Techniques de base

Le choix des notations p(A) et p(B) va per-mettre de passer rapidement aux notations en probabilité.

Réponses de l’Étude de situation p.12a) Les filles, les élèves de 17 ans, la classe, les filles ou élèves de 17 ans.b) L’idée est de faire comprendre aux élèves comment on dresse un diagramme de Venn. 20 filles moins les 8 filles de 17 ans, donnent 12 filles qui n’ont pas 17 ans. c) 2 élèves qui sont des garçons qui n’ont pas 17 ans. d) Lien avec la notion de partition en sous populations disjointes. Voir tableau ci-après.

e) 1 .

groupe effectif fréquence

filles de 17ans 8 0,25

filles n’ayant pas 17 ans

12 0,375

garçons de 17 ans 10 0,3125

garçons n’ayant pas 17 ans

32 – 30 = 2

0,0625

total 32 1

Corrigés des Exercices d’application p.13

6 0,65 + 0,55 – 25 = 0,8 .

Le glacier vend 80 % de glaces à la vanille ou au chocolat.

7 D : entrées le dimanche et R : entrées à tarif réduit p(D ∪ R) = p(D) + p(R) – p(D ∩ R)⇔ p(D ∩ R) = p(D) + p(R) – p(D ∪ R)

= 35 + 0,5 - 0,9 = 0,2

20 % des entrées ont lieu le dimanche et à tarif réduit.

8 a) Diagramme de Venn

b) Uniquement le hand : 60/120 = 0,5 .c) Un seul sport : 100/120 ≈ 0,83 .d) Le hand ou le basket : 1 .e) Le hand parmi ceux pratiquant le basket : 20/60 ≈ 0,33 .

Inclusion et proportions

C’est la notion de « pourcentage de pourcen-tage » des programmes précédentsLe programme a privilégié une approche de cette «  formule » p × p’ qui va permettre de passer au principe multiplicatif utile dans les probabilités (Chapitre 5) et dans les

2

3

Page 7: Math 1ere STMG

Chapitre 1 pourCentages 7

probabilités conditionnelles en Terminale. Le choix du thème (actifs, chômeurs…) est conforme au programme et permet d’utili-ser le mot « taux » dans le sens de proportion dans un contexte de vie quotidienne.

Nous avons le plus possible utilisé l’Insee et le T.E.F. comme sources pour les informations chiffrées. Les chiffres peuvent ainsi être actua-lisés par les professeurs qui le désirent.

Les chiffres donnés ici sont « simplifiés » pour permettre d’effectuer rapidement les calculs, mais les proportions restent celles de la France en 2009.

Les pourcentages sont donnés avec deux chiffres après la virgule, et les proportions avec 4 chiffres significatifs. ➜ p. 190

Réponses de l’Étude de situation p. 14a) 0,05417 : 5,42 % des 15 ans ou plus.

b) 8,67 % de 62,5 % , c’est-à-dire 0,0867 ×0,625 = 0,05419 ≈ 0,0542

On retrouve la réponse en a)c) Part des 65 ans ou plus :

4200/9000 ≈ 0,4667 ,soit 46,67 % des inactifs.d) Il y a des personnes de 65 ans ou plus qui sont actifs.e) Les 65 ans ou plus sont les 4200 inactifs, plus 1% des actifs, soit 1 % des 15000 actifs. Donc 4200 + 150 = 4350 .Donc 4350/24000 = 0,18125 , soit 18,13 % des 15 ans ou plus.f ) On peut dans un premier temps mettre les effectifs, plus facile pour les élèves, et ensuite les proportions.

Corrigés des exercices d’application p. 159 Soit B le budget total de ce ménage :

0,35 × 0,16 × B = 378 ⇔ B = 6 750 € .

Taux d'évolution

Nous avons fait le choix de « Ancienne valeur V0 » et « Nouvelle valeur V1 » dans cette par-tie du cours car il n’y a qu’une étape et « valeur initiale » et « valeur finale » dans le paragraphe suivant. Comme d’autres professeurs utilisent « valeur de départ » et « valeur d’arrivée »… nous avons utilisé ce vocabulaire dans la page Faire le point. Nous espérons ainsi ne pas avoir trop perturbé les élèves, le code couleur vert pour le départ, bleu pour l’arrivée n’ayant pas changé. Les formules peuvent paraître nombreuses. Cependant, nombre d’élèves n’arrivent pas à passer d’une écriture à une autre, fraction et simplification n’étant pas maîtrisées, et ils pré-fèrent apprendre plus de formule. Notre pra-tique en classe nous a montré que l’emploi de t = CM × 100 – 100 facilitait le passage du CM au taux en % .

Réponses de l’Étude de situation p. 16a) ∆V = 300 – 1000 = - 700 CM = 300/1000 = 0,3 Variation relative : – 700/1000 = – 0,7 ou – 70% .

4

Page 8: Math 1ere STMG

8 Chapitre 1 pourCentages

b) ∆V = 8000 – 2000 = 6000 CM = 8000/2000 = 4 Variation relative: 6000/2000 = 3 ou + 300 % .

Corrigés des Exercices d’application p.17

11 1. Nouveau budget du service D :2 000 × 1,3 = 2 600 € .Ancien budget du service E :4 200/1,4 = 3 000 € .

2. Service D : 2 600 × 0,85 = 2 210 € .Service E : 4 200 × 0,65 = 2 730 € .

13 a) 140/1,38 ≈ 101 Soit 100 millions de personnes qui ont uti-lisé leur portable en 2010 pour payer leurs achats.

b) 86 000/140 ≈ 614 .Le montant moyen d’une transaction en 2011 a été de 614 $ .

c) 86/1,76 ≈ 48,9 .Soit 49 milliards, montant des transactions en dollars $ en 2010, arrondi à 1 milliard près.

Évolutions successives - Évolution réciproque

L’évolution réciproque a déjà été abordée dans la partie précédente, du fait de la recherche d’une valeur initiale, elle est vue ici comme un taux de compensation.Ce thème sera repris dans les suites, surtout en terminale, par la recherche d’une formule explicite des suites géométriques.

Edition 01, pour le calcul de V2 : lire 17500 – 30/100 × 175000.Dans la mise en pratique, pour mieux appré-hender la notion de taux global d’évolution, le choix a été fait d’étudier l’inflation en France à l’époque où elle était très forte, car dans ce cas, l’approximation n’est pas possible et le calcul par la somme des taux est «visiblement» faux. Ces valeurs seront reprises dans l’atelier 3 TICE.

Réponses de l’Étude de situation p. 18a) Au final, le CA a diminué.b) CM = 122500/140000 = 0,875

c) t = 0,875×100 – 100 = – 12,5 baisse de 12,5 % .d) Même taux d’évolution. Faire faire les deux calculs : 140000 – 30/100 ×140000 = 9800098000 + 25/100 × 98000 = 122500 .Et faire une généralisation à l’aide du produit des CM.e) Faux, la baisse est de 12,5 % et non de 5 %.f) 122500+12,5/100×122500 = 137812,5 g) CM’ = 140000/122500 = 1,14286 Taux d’évolution réciproque : 14,29 % .

Corrigés des Exercices d’application p. 1914 CMglobal = 1,2 × 1,3 × 0,6 = 0,936

La subvention diminue globalement de 6,4 % .

15 1. a) CMglobal = 0,9 × 0,88 = 0,792.Soit une baisse de 20,8 % .b) Le coefficient multiplicateur qui permet de retrouver la valeur initiale est :CM’ = 1/0,792 ≈ 1,2626 .La hausse réciproque est 26,26 % .2. a) CMglobal = 1,20 × 0,6 = 0,72b) CM’ = 1/0,72 ≈ 1,3889La hausse réciproque est 38,89 % .

Atelier TICE Taux de TVAA. à la calculatrice 1. Cellules en C, D et E , puis F, G et H

5

1

Page 9: Math 1ere STMG

Chapitre 1 pourCentages 9

la part de la TVA est de 9,29 % du montant total.

B. à l’aide d’un tableur1. 2. Formules : C3 = F3/(1+D3/100) et E3 = F3 – C3 C4 = E4/(D4/100) et F4 = C4 + E4 E5 = C5*D5/100 et F5 = C5*(1+D5/100) E6 = F6 – C6 et D6 = E6 / C6 *100 3.a) En G2, on calcule le prix unitaire par la quantité de lecteurs multimédia commandés. En H2 , on calcule le montant de la TVA sur un lecteur par la quantité commandée. b) En tirant on obtient pour tous les types de produits.4. a) G7 = 81241,02 et H7 = 7544,52 b) H8 = H7/G7*100 .5. a) Si les quantités changent, la proportion de TVA va changer puisque le taux de TVA n’est pas constant.b) Si les prix unitaires changent, la proportion de TVA change aussi.6. Nouveau taux de TVA à 7% pour les livres et les billets. On admet que les prix unitaires HT ne changent pas .

La TVA augmente de 645,45 € .8189,97 - 7577,52 = 645,45 .

Billets de train

1. C4 = 702. a) En D2, pour obtenir le nombre de billets achetés sur internet.

b) C2 = C4*30/100

3. Tableau complet

4. I5 = H4 + I2 – H2 ou = G2 + H2 + H3 51,5 % de billets rentables.

Taux d'inflation1. Pour 500 francs, le budget en décembre 1979 doit être multiplié par :1,116×1,096×1,094×1,091×1,108 ≈ 1,617 5soit 810,22 francs.Puis en décembre 1980 : 920,41 francs.Puis en décembre 1984 : 1373,57 francs.2. a) b) B3 = 1+B2/100 c) L3 = 2,7471. En 10 ans les prix sont multi-pliés par 2,7471. d) L2 = L3*100 – 100 . Soit un taux d’inflation global de 174,71 % .

Corrigés des exercices

à l’oral

16

a) 0,015 b) 0,004 c) 0,0223d) 0,00052 e) 3 f) 4,5

17

a) 40 % b) 50 % c) 25 %d) 30 % e) 75 % f) 80 %

18

a) 100 b) 3,5 c) 360

2

3

Page 10: Math 1ere STMG

10 Chapitre 1 pourCentages

19

En C2 =A2/B2En A3 =B3*C3 En B4 =A4/C4

20

Proportion du reste : 32 %

22

a) 12 % de 20 % = 20 % de12 % = 2,4 %b) 40 % de 6 % = 60 % de 4 % = 2,4 %c) 15 % de 60 % = 45 % de 20 % = 9 %d) 2/3 × 6/7 = 4/7

23

a) p’ = 0, 240, 4 =

35

b) p’ = 0, 050, 5 = 1

10

24

a) CM = 1,18 b) CM = 0,95c) CM = 0,94 d) CM = 1,40

25

1. b) c)2. b) c)

26

a) + 4 % b) + 4 0% c) + 0,4 % d) – 5 % e) – 41 % f) – 91 %

27

1. c) 1,1 × 0,9 = 0,992. c) + 21 %3. c) 1,10 × 1,08 × 1,06 = 1,25928

29

CMglobal = 1,15 × 0,92 × 1,2 = 1,2696 hausse de 26,96 %.

Pour s’entraîner

Proportion dans un ensemble

30

75/120 = 0,625 , soit 62,5 % de femmes dans l’entreprise.

31

1/5 × 230 = 46Olivier change 46 L d’eau ou 0,046 m3.

32

Loyer : 0,3 × 600 = 180 €Crédit : 0,15 × 600 = 90 € .

33

9/20 = 0,4530% des étudiants en BTS MUC et 45 % des étudiants en BTS Transport.

35

Adhérents de 20 ans ou plus : 66/160 = 0,4125 soit 41,25 %Adhérents de moins de 20 ans : 94/160 = 0,5875, soit 58,75 % .

36

Temps (en min) Nombre de fois

Fréquence (en %)

moins de 15 18 15

de 15 à 16 exclu 30 25

de 16 à 17 exclu 54 45

de 17 à 18 exclu 12 10

18 ou plus 6 5

Total 120 100

38

L’effectif total des différents salaires est 305 et les fréquences en pourcentage des classes de salaire sont en liste L2.

1

Page 11: Math 1ere STMG

Chapitre 1 pourCentages 11

39

L’affirmation de Benjamin n’est pas justifiée.Les deux pourcentages ne portant pas sur le même ensemble de référence, on ne peut pas comparer les effectifs des moins de 12 ans dans les deux clubs.

40

1.

2. Le montant de la réduction de TVA est donné par :

(19,6/100 – 8,5/100) × Prix HT

3. Prix TTC = prix HT + TVA = TVA/0,055 + TVA

= +⎛⎝⎜⎜⎜

⎞⎠⎟⎟⎟⎟

10, 055

1 × TVA .

Réunion et intersection de sous-populations

42

1. A dans E : 34/70 ≈ 0,486B dans E : 32/70 ≈ 0,4572. A ∩ B dans E : 8/70 ≈ 0,1143. A ou B dans E : 58/70 ≈ 0,829

43

Mention AB : 25 %Mention B : 45 %Mention TB : 30 %

44

H l’ensemble des joueurs de handB l’ensemble des joueurs de basketp( H ∪ B ) = p(H) + p(B) – p( H ∩ B ) = 0,3 + 0,18 – 0,1 = 0,38 , soit 38 % .

45

a) Part des clients abonnés au forfait C :1 – 0,35 – 0,5 = 0,15 , soit 15 % .b) Part des forfaits A dans les forfaits limités : 0,35/0,85 = 7/17 ≈ 0,412 , soit 41,2 % .

46

a) X l’ensemble de ordinateurs équipés du pro-cesseur « Xrion » et P l’ensemble de ordinateurs équipés de la carte graphique « Plomba 13 »p( X ∪ P ) = p(X) + p(P) – p( X ∩ P )= 0,45 + 0,6 – 0,2 = 0,85 , soit 85 % .

b) X non X total

P 20 40 60

non P 25 15 40

total 45 55 100

47

1.a) p(A) = 90/1500 = 0,06b) p(B) = 120/1500 = 0,08c) p( A ∩ B ) = 27/1500 = 0,0182. p( A ∪ B ) = p(A) + p(B) - p( A ∩ B ) = 0,122D’où la proportion de jouets de cet échantillon possédant un seul défaut:p( A ∪ B ) - p( A ∩ B ) = 0,104 soit 10,4 % .

2

Page 12: Math 1ere STMG

12 Chapitre 1 pourCentages

48

1. p(F ∪ C) = p(F) + p(C) – p(F ∩ C )= 0,6 + 0,2 – 0,1 = 0,7 , soit 70 %. 2.

C non C total

F 15 35 50

H 25 25 50

total 40 60 100

p( H ∪ C ) = p(H) + p(C) – p( H ∩ C) = 0,5 + 0,4 – 0,25 = 0,65 , soit 65 % .

Inclusions et proportions

49 0,6 × 0,3 = 0,18

La collection d’Iléana contient 18 % de films avec Bruce Willis.

51 Soit p = 0,9 la part de l’énergie renouvelable dans la consommation énergétique de l’usine .Soit p’ la part de l’énergie fournie par les éoli-ennes par rapport aux énergies renouvelables.D’après l’énoncé , p × p’ = 0,63 .D’où p’ = 0,63/p = 0,63/0,9 = 0,7 soit 70 % .

52 1/5 × 0,4 = 0,08 .Les cadres supérieurs représentent 8 % des salariés.

54 Soit p = 0,63 la part des téléphones mobiles de marque S et p’ la part du modèle TM66 dans la marque S.D’après l’énoncé , p × p’ = 0,22 .D’où p’ = 0,22/0,63 ≈ 0,35 , soit 35 % .

55 1. Proportion de filles internes dans l’établis-sement :

0,6 × 0,2 = 0,12 , soit 12 % .2.

Externes D.P. Internes Total

Garçons 120 330 150 600

Filles 315 405 180 900

Total 435 735 330 1500

3. a) La proportion de garçons est 0,4 .b) La proportion d’élèves internes et garçons est 0,1 .c) La proportion d’internes parmi les garçons est 0,25 .d) La proportion d’internes ou de garcons est 0,52 .e) La proportion de garçons dans les internes est  ≈ 0,45 .

56

a) Part des journaux nationaux distribués par le groupe H pour la totalité du marché :

0,65 × 0,8 = 0,52 , soit 52 % .b) Soit p la part des journaux nationaux dans le groupe M. Par lecture de l’énoncé et de l’arbre, on obtient :0,8 × 0,65 + 0,2 × p = 0,62 ⇔ p = 0,5 .

57

1.

2.

M LV autres total

H 9 10 41 60

F 9 20 61 90

total 18 30 102 150

3. 10% des enseignantes sont professeures de mathématiques.

Taux d’évolution58

1. 20,6 millions d’habitants .2. 5350 € .

59

1. a) + 2 % b) + 58,5 %c) + 140 % d) + 0,35 %2. a) – 10 % b) – 45 %c) – 1,5 % d) – 24 %

3

4

Page 13: Math 1ere STMG

Chapitre 1 pourCentages 13

60

V1 = 540 × 0,8 = 432 € .

61

V1 = 1,6 × 0,955 = 1,528 milliard de dollars

62

CM = 6 et CM × 100 - 100 = 500 .D’où une hausse de 500%

64

De mars à mai 2011 , CM = 1 – 0,188 = 0,812 .D’où le montant des exportations en mars 2011 :

4761/0,812 ≈ 5860 milliards de yens.

65

66

1. 10 000/1500 ≈ 6,67Soit une hausse de 567 % du nombre d’ordina-teurs fabriqués entre les années 2000 et 2010 2. 510/2100 ≈ 0,24Le prix a baissé de 76 % .3. Le coefficient multiplicateur du chiffre d’af-faires est 6,67 × 0,24 = 1,6008 .Le chiffre d’affaires a augmenté de 60 % pen-dant la même période.

67

1. Le 11 mars 2011:1 yen = 206/23 000 ≈ 0,00895652 € .Pour info le 11 mars 2012 :

1 yen = 0,00923628.2. a) 400 000 milliards représentent 38 % de la dette de 2011. D’où le montant total de la dette en 2011 :

400 000 / 0,38 ≈ 1 052 632 milliards de yens.b) Montant estimé de la dette en 2020 :

1 452 632 milliards de yens .Montant de la dette en 1990 :

1 052 632/3 ≈ 350 877 milliards de yens.

68

70

1. C’est l’indice DAX qui a le plus baissé en per-dant 1745 points.

2.

Page 14: Math 1ere STMG

14 Chapitre 1 pourCentages

71

1. a) De début 1980 à fin 1984, le SMIC a aug-menté de 77,6 % .b) Le SMIC a suivi pratiquement la même évo-lution que le PIB .2. a) De début 2006 à fin 2010, le SMIC a aug-menté de 10,3 % .b) C’est une augmentation supérieure à celle des prix, mais moindre que celle du PIB sur la même période.3. a) De début 1980 à fin 1984, la dette a aug-menté de 143 % , beaucoup plus que l’aug-mentation des prix et du PIB.b) De début 2006 à fin 2010, la dette n’a aug-menté que de 39 % .

évolutions successives – évolution réciproque

72

CMglobal = 1,1 × 0,8 = 0,8 × 1,1 = 0,88Même évolution de – 12 %.

73

a) CMglobal = 1,5 × 0,2 = 0,3Le prix diminue de 70 % pour certains tou-ristes.b) Pour un prix de 23 € avant l’été, le nouveau prix est  6,90 € .

75

a) CMglobal = 1,3 × 1,2 × 0,5 = 0,78L’indice baisse de 22 % .b) Le coefficient multiplicateur de l’évolution réciproque est CM’ = 1/0,78 ≈ 1,2821Il faut compenser une baisse de 22 % par une hausse de 28 % à 1 % près .

76

CMglobal = 1,15 × 1,2 = 1,38CM’ = 1/1,38 ≈ 0,72460,7246 ×100 -100 = – 27,54L’article est soldé à – 28 % , à 1 % près, pour revenir au prix initial.

77

a) CMglobal = 1,15 × 0,85 = 0,9775On obtient une baisse de 2,25 % .

b) CMglobal = 1100

1100

t t+

⎛⎝⎜⎜⎜

⎞⎠⎟⎟⎟⎟ −⎛⎝⎜⎜⎜

⎞⎠⎟⎟⎟⎟

c) Angélique utilise un produit remarquable :

CMglobal = 1 10 000

2

−t

Si t = 30, alors 10 000

2t=

90010 000

= 9

100 .

D’où CMglobal = 1 – 9

100 .

Cela indique bien une baisse de 9 % .

78

a) CMglobal = 1,021 × 1,015 × 1,015 × 1,026 × 1,01 × 1,009 × 1,018 ≈ 1,1196.

Doù le taux global d’inflation de 12 % à 0,1 près, sur la période 2004-2010 .b) Par la somme des taux, le journaliste obtient 11,4 % . L’écart est faible car les taux sont petits. Mais ce n’est pas au programme de cette classe.

79

1. En C3 =1+B3/100Puis recopie vers le bas jusqu’en C9.2. En C10 =PRODUIT(C3:C9)3. Entre 2003 et 2009 , le PIB de la Chine a aug-menté de 107,4 % .La Chine est un pays émergent.

80 a) CMglobal = 0,95 3 ≈ 0,8574 .b) Soit une baisse de 14,26 % .La baisse est inférieure à 15 % .c) Avec les trois baisses successives de 5 %, le prix final est :

200 × 0,953 ≈ 171,48 € .Avec la baisse de 15 %, on obtient un prix final de 170 €. L’erreur commise est de 1,48 € , faible parce que le prix n’est pas très grand.

5

Page 15: Math 1ere STMG

Chapitre 1 pourCentages 15

81

a) 1000 × 1,02 3 ≈ 1061 € .Pour une dépense de 1000 € en 2010, on pré-voit 1061,21 € en 2013.b) Au taux de 2,25 % par an, un capital de 1 000 € placé en 2010 devient égal à 1 000 × 1,0225 3 ≈ 1069 € .Placement très peu intéressant par rapport à l’inflation. La hausse en tenant compte de l’in-flation n’est plus que de 0,75 % sur les trois ans.

82

1. Budget en 2011 : 45 000/0,9 = 50 000 € .2. Variation absolue : 5 000 € .3. CM’ = 1/0,9 ≈ 1,1111 .Pour revenir à la valeur de 2011, il faut aug-menter le budget de 2012 de 11,1 % .

83

1. CMglobal = 1,2 × 0,7 = 0,84Baisse de 16 % .2. CM’ = 1/0,84 ≈ 1,1904 .Il faut une hausse de 19 % de la production pour revenir à la valeur initiale et compenser une baisse de 16 % .

84

Revenu annuel :7 000 × 12 + 90 000 = 174 000 € .Nouveau salaire mensuel :174 000/12 = 14 500 € .Pour couvrir les mêmes frais, il faut augmenter le salaire de 107 % .Une augmentation de salaire cache parfois le problème des charges…

Pour approfondir

85

1. La somme des parts des menus avec apéritif ou avec café est égale à 1.Si T est l’ensemble des menus avec apéritif et F l’ensemble des menus avec café , alors : p( T ∩ F ) = p(T) + p(F) – p( T ∪ F) = 0,3 + 0,7 – 1 = 0 .T et F sont donc disjoints et le « ou » est exclu-sif.2. a) En C3 =2*C4/3

b)

3. a) p( A ∩ T ) = 0,05  ; p( B ∩ F ) = 0,3et p( A ∩ F ) = 0,25 .b) p(A ∪ F ) = p(A) + p(F) – p(A ∩ F ) = 0,3 + 0,7 – 0,25 = 0,75 .

86

1. a) en E2 =SOMME(B2:D2)b) En 1990 : 1166,4/4116,8 ≈ 0,2833 28,33 % de fonctionnaires territoriaux.c) 1805,9/4949,6 ≈ 0,3649 soit 36,49 % en 20092. Entre 1990 et 2001:Fonction publique d’état : + 5,90 % .Fonction publique territoriale : + 17,62 %Fonction hospitalière : +18,79 % .3. a) Les cellules B8 à D9 sont au format pour-centage.En B8 = B4/B3 - 1b) En D8 = D4/D3 - 1c) En B9 = B7/B4 - 1d) Tableau des résultats

87

1. a) En B6 =SOMME(B2:B5)b) En G2 =SOMME(B2:F2)c) Tableau des effectifs

Ce groupe compte 250 jeunes.

Page 16: Math 1ere STMG

16 Chapitre 1 pourCentages

2. Tableau des fréquences

a) La formule =B2/B$6*100 placée en J2 donne la part des jeunes de 15 ans qui prennent leur repas de midi un jour de vacances en restauration rapide.b) Par recopie vers la droite de J2 à N2, on obtient les parts en restauration rapide selon l’âge.

88

1. a) Tableau des effectifs

degré coût en 2000

coût en 2005

coût en 2009

primaire 5 145 5 263 5 696

secon-daire 9 041 9 204 9 376

supérieur 9 544 10 279 11 262

b) Augmentation de 9,6% pour le supérieur entre 2005 et 2009 .

2. a) b)

c)

3. On place les coûts de 2009 en liste1, les taux d’évolution de 2005 à 2009 en liste 2 et on cal-cule les coûts en 2013 en liste 3 en supposant que les taux d’évolution se maintiennent

89

1.a)

2008 2009

consomma-tion

84,63% 83,83%

épargne 15,37% 16,17%

b) De 2008 à 2009, la part de l’épargne brute a augmenté de 0,8 points.2. a) De 2008 à 2009, le PIB a diminué de 2,66 % .b) Le coefficient multiplicateur de cette baisse est CM = 0,9734 .Le coefficient multiplicateur de la hausse réci-proque est CM’ = 1/0,9734 ≈ 1,0273 .Le PIB doit augmenter de 2,73 % en 2009 pour retrouver en 2010 son niveau de 2008.3. Entre 2008 et 2009, la capacité de finance-ment des ménages a augmenté de 43,71 % .

90

1. a) Population mondiale N au 1er janvier 2010. N = 501,1/0,07 ≈ 7 159 millions

≈ 7, 2 milliardsb) Chine : 7159 × 0,2 = 1431,8 millions

≈ 1,432 milliardsÉtats-Unis :

7159 × 0,05 = 357,95 ≈ 358 millions2. a) Variation absolue en Union européenne, de 2009 à 2010 :

1,4 million.b) Hausse de 0,28 % .c) Hausse de 2,8 habitants pour 1000 .3. Avec ce même taux, population prévisible en Union européenne :en 2015 :

501,1 × 1,00285 ≈ 508,2 millions ;en 2020 :

501,1 × 1,002810 ≈ 515,3 millions.

Page 17: Math 1ere STMG

Chapitre 1 pourCentages 17

Vers le Bac

94 1.

effectifs homme femme total

PS ou I 5 3 8

sain 56 140 196

malade 29 17 46

total 90 160 250

2. 140160 × 100 = 87,5 .

87,5 % des femmes sont dans le groupe des personnes saines.

fréquences homme femme total

PS ou I 5,6 1,9 3,2

sain 62,2 87,5 78,4

malade 32,2 10,6 18,4

total 100 100 100

95

1. a) Loyer HC du T2 : 54,6/0,12 = 455 €b) Tableau des résultats

c) Pour l’appartement T1 : 65/461≈ 0,14 ,soit 14% de charges par rapport au loyer men-suel net.2. 260/455 ≈ 0,57 . Donc la part de l’aide repré-sente 57 % du loyer HC.

96

1. a) Diagramme et arbre de répartition

b) Les cadres représentent 20 % des salariésc) F l’ensemble des femmes et A l’ensemble des agents administratifs :p( F ∪ A ) = p(F) + p(A) – p( F ∩ A )

= 0,45 + 0,2 – 0,14 = 0,51 .2.

3. a) Part des femmes dans les ouvriers : ≈ 42 % .b) Part des salariés cadres et hommes : 14 %c) Part des salariés ouvriers ou femmes : 80 % .

97

1.C.A. en € effectif

Haribo 1,6 × 109 6 000

P.V.M. 1,5 × 109 12 000

Chupa Chups 260 × 106 1600

Afchain 1,15 × 106 11

2. a) Chupa Chups représente 17 % du chiffre d’affaires de Perfetti Van Melle.b) Tableau des valeurs en € .

Page 18: Math 1ere STMG

18 Chapitre 1 pourCentages

98

Formules utilisées

2. a) Il n’est pas possible que le taux de réussite pour la ville soit supérieur au taux de réussite du meilleur lycée.b) Meilleur lycée : Victor Hugo.Le moins bon : Saint-Exupéry.3. Les proviseurs souhaitent un taux de réus-site de 96,74 % .

99

1. a) En C3 = B3/B2Puis recopie vers le bas jusqu’en C8b) La colonne D est au format pourcentage . En D3 = C3-12. a) C : « coefficient multiplicateur » ou « CM ».b) T : « taux d’évolution » ou « taux ».

3. a)

b) Taux d’évolution :le plus fort : de 1909 à 1929 .le plus faible : de 1889 à 1909 .

100

1. Réponse 1 et réponse 4.2. a)

b) Hausse de 43,8 % entre janvier et juin 20083. a) Baisse de 72,9 % entre juin et décembre 2008.b) Il faut une hausse de 270 % pour compenser une baisse de 73 %.

Page 19: Math 1ere STMG

Chapitre 2 seCond degré 19

Chapitre 2 Second degré

Intentions des auteuresCe chapitre est nouveau dans la série STMG. La résolution des équations du second degré est facilitée par l’emploi des calculatrices et de l’algorithme.L’étude des fonctions du second degré en Seconde est ainsi poursuivie. Pour permettre aux élèves de STMG de revenir sans trop insis-ter sur ces connaissances, nous avons « rejeté » les pré-requis dans la partie Techniques de base.

➜ p. 193 pour le calcul algébrique.Cette partie est volontairement allégée (trop de « traumatismes » pour les élèves de cette série, en grande majorité fâchés avec l’algèbre). Nous avons rappelé le strict minimum : développe-ment d’un carré, factorisation par x , et un peu de développement d’un produit.

➜ p.194 pour les lectures graphiques. On y trouve image, sens de variation, équation, signe d’une fonction.

➜ p.196 pour l’équation ax + b = 0  : néces-saire pour trouver la valeur annulant la dérivée d’une fonction du 2nd degré.

➜ p. 197 pour le calcul d’image par la calcu-latrice. Il est peu performant de faire faire les calculs « à la main » puisque de toute manière les élèves calculeront avec leur machine le moindre produit. Mieux vaut apprendre à obtenir un tableau de valeurs, qui est un objectif du nou-veau programme. On peut cependant faire faire des petits calculs d’images pour x = 1 , –1 ou 0 .

➜ p. 198 pour un retour sur les droites, pour savoir lire un nombre dérivé, et le signe de ax + b.

➜ p. 199 pour la fonction carrée et le lien avec la parabole et la forme canonique  : le strict minimum pour faire comprendre le sens de variation de la fonction du second degré. Mais, on peut penser que la première partie du cours p. 38, qui fait le lien entre forme canonique et

sens de variation, sera trop délicate à retenir pour certains élèves. Il faudra attendre le Chapitre 4 sur l’étude de la fonction et le calcul de la dérivée, plus simple à l’utilisation. Les QCM Avant de commencer permettent de revenir sur les connaissances de base sur le second degré, les équations et le lien avec le graphique.Nous ne pouvons que proposer de prendre vraiment le temps de travailler ce QCM, d’expli-quer les réponses justes comme les fausses, en particulier la réponse de Léa (que l’on trouve souvent dans les copies) et de Marvin au QCM 3 : celle de Marvin étant la recherche de x tel que P(x) = 0 est correcte car on ne demande pas la résolution de l’équation P(x) = 0 .Au QCM 4, nous avons introduit le vocabulaire « traverse l’axe … aux graduations … ».Ne pas hésiter à renvoyer les élèves aux Tech-niques de base.

Avant de commencer

Réponses fausses aux QCM1. c) 2. c) 3. a) 4. c) 5. b) 6. c)

Cours

Fonction polynôme du second degré

Nous avons fait le choix d’une fonction assez simple, où a = 2 . Les cas où a = 1 ou –1 sont plus délicats à appréhender pour certains élèves. Il sera certainement utile de demander les valeurs de a, b et c de façon systématique dans les premiers exercices. La lecture des valeurs sur un tableau de valeurs de la calcu-latrice est importante et rassurante pour les élèves.

1

Page 20: Math 1ere STMG

20 Chapitre 2 seCond degré

Une astuce que nous avons utilisé en ES  : pour trouver l’abscisse du sommet à partir de la forme canonique, on cherche la valeur qui annule le carré  : ici on résout l’équation x + 1 = 0 , plus facile que de reconnaître –1 dans 2 ( x + 1 ) 2 – 8 .La mise en situation proposée fait revenir sur beaucoup d’éléments. Nous conseillons de ne pas y passer trop de temps pour les élèves qui ont des difficultés en algèbre, car l’étude de fonction vue au chapitre 4 à l’aide de la déri-vée sera plus « simple » à retenir. Deux résultats doivent être mis en valeur et retenus : • le signe du « nombre qui multiplie x au carré » ou « devant x au carré » (corriger « le signe de x2 » ou « devant x deux » ), qui donne le sens de la parabole : les élèves doivent avoir en tête la forme de la parabole, indiquant le sens de variation d’une part, mais aussi le signe du polynôme suivant les solutions de l’équation P(x) = 0.• la lecture des 0 du tableau de valeurs et leur interprétation.Dans l’exercice résolu, nous avons introduit une autre difficulté pour les élèves : l’unité dans laquelle est donnée la variable et les valeurs de la fonction f(x) est ici en centaines. Nous avons délibérément multiplié ces cas dans les exer-cices d’entraînement pour habituer les élèves à cette lecture de l’énoncé. Mis à part les QCM, dans les exercices Vers le Bac tous les exercices sont contextualisés, comme vu au Bac depuis au moins 10 ans.

Réponses de l’Étude de situation p. 38

a) ➜ développement en Techniques de base p. 193Pour de nombreux élèves, mieux vaut revenir à ( x + 1 )2 = ( x + 1 )( x + 1 ) en rappelant que 72 = 7 × 7 , on multiplie le même facteur, car les élèves ont des automatismes de type ( x + 1 )( x – 1 ) …b) Revenir s’il le faut à la forme vue en seconde ➜ p. 198 ou résoudre x + 1 = 0 .c) Retour sur le développement d’une forme factorisée : développer ( x + 3 )( x – 1 ) puis multiplier par 2.d) P(x) = 0 ⇔ 2 ( x + 3 ) ( x – 1 ) = 0

Produit nul x + 3 = 0 ou x – 1 = 0 , 2 jamais nul. On retrouve les abscisses de A et B, aux graduations – 3 et 1 .Faire également le lien avec les deux 0 dans la colonne Y1 du tableau de valeurs à la cal-culatrice. Corrigés des Exercices d’application p. 39

1 (x – 2)2 + 3 = x2 – 4x + 4 + 3 = x2 – 4x + 7. C( 0 ; 7 ) . Au départ, à l’ouverture du site, le site propose 700 flacons.

2 On a P(x) = – (x + 1)2 + 4 donc a = – 1 , b = 4 et a = – 1 (négatif donc) : la parabole est tournée vers le bas. Ne pas hésiter à faire faire le geste.

x

P(x)4–1

3 Exercice où l’ensemble de définition est très important, car – 4 ∉ [ 0 ; 10 ] .Ne pas hésiter à visualiser la courbe à la cal-culatrice sur [ 0 ; 10 ] .

Équation ax2 + bx + c Les formules sont admises. Nous avons donc simplement fait le lien avec le polynôme vu dans la situation précédente. Nous n’avons pas insisté sur le fait qu’au maximum, il y a deux solutions. Le lien avec la parabole et l’inter-section avec l’axe des abscisses peut aider à le comprendre quand on a déjà pratiqué. Devant la difficulté des élèves sur le vocabu-laire mathématique, nous avons délibérément omis de parler de « racines d’un polynôme » : en effet, cela demande une interprétation du mot  ; ce n’est pas comme la racine (carrée) connue, cela signifie que si on remplace x par ce nombre, le polynôme donne 0… Bref, par-ler de «  solution de l’équation  » nous paraît plus direct pour cette classe. D’ailleurs le pro-gramme ne parle que de résolution d’équation et d’inéquation… Nous avons fait le choix de commencer par la

solution xb

a=− − ∆

21 , car c’est la plus petite

lorsque a > 0 et la première alors à placer sur l’axe des abscisses.

2

Page 21: Math 1ere STMG

Chapitre 2 seCond degré 21

Nous n’avons pas insisté sur les calculs avec ∆ non carré parfait.

Pour faire le lien avec la forme canonique, nous avons choisi de nommer a et non x0 la solution lorsqu’elle est unique. Faute de place, nous n’avons pas insisté sur la place centrale de

a = ba

−2 par rapport aux deux solutions.

Le choix de – b placé au numérateur, et non un – devant la fraction, est voulu : mieux vaut l’in-terpréter pour les élèves comme « l’opposé de b » . Ainsi, si b = – 3 , faire écrire directement 3. De même pour b2 , faire écrire directement 9 et non ( – 3 )2 qui devient vite – 32 sur les copies…

De même, dans le calcul « à la main » de – 4 ac , nous conseillons de ne pas faire écrire aux élèves exactement le calcul, mais de tout de suite voir le signe de – ac, puis de faire le pro-duit par 4 des nombres sans signe. Cela évite de très nombreuses erreurs de calcul  ! Sinon, mieux vaut utiliser le programme en page 46.

Réponses de l’Étude de situation p. 40a) Pour R(x) , ∆ = 1 – 8 = – 7 , donc la racine carrée n’existe pas, et on ne peut calculer les formules. b) La position de la parabole par rapport à l’axe des abscisses confirme.c) Pour T(x) , ∆ = 4 – 4 = 0 , on peut calculer les formules, car 0 = 0 et on obtient deux fois la même solution 1 .d) La parabole « touche » l’axe des abscisses en x = 1 .

Pour l’exercice résolu dans la mise en pratique, nous conseillons de visualiser les courbes à la calculatrice. Ces courbes sont visualisées sur TI-Nspire dans l’exercice 6 en page 43 .

Corrigés des Exercices d’application p. 414 a) Recette engendrée par la vente de x

milliers de bouteilles : 10 000 x , en euros.Soit R(x) = 10x , en milliers d’euros.b) Bénéfice en fonction de x, où la quantité x varie de 0 à 10 mille bouteilles et le bénéfice B(x) est en milliers d’euros :B(x) = 10x – (x2 + 2x + 7) = – x2 + 8x – 7 .c) – x2 + 8x – 7 = 0 ⇔ x =1 ou x = 7Lorsque la société vend mille bouteilles ou 7 mille bouteilles, le bénéfice est nul.

5 P(x) = 3x2 – 3x – 6 et R(x) = – x2 + 8x – 16.1. P(x) = 0 : Δ = 81 et S = { – 1 ; 2 } .2. P(x) – R(x) = 0 ⇔ 4x2 – 11x + 10 = 0Δ = – 39 ; l’équation n’a pas de solution, la calculatrice formelle TI-Nspire répond false pour «  pas de solution  ». Les deux para-boles représentant les fonctions P et R ne se coupent pas.

Signe du trinôme ax2 + bx + c, avec a ≠ 0

Nous avons conservé le mot « trinôme » car il est écrit dans le programme officiel.

Depuis 15 ans, le programme de ES indique que le signe de trinôme est lié à la position de la parabole par rapport à l’axe des abscisses. Le « théorème » algébrique « du signe de a à l’extérieur des racines » n’est plus de mise. Le coefficient a donne l’allure de la parabole, le discriminant indique si la parabole traverse, touche, ou reste du même côté de l’axe des abscisses et on obtient ainsi le signe du tri-nôme. Ces paraboles ont été vues en page 40.

➜ p. 194 pour le signe d’une fonction

Nous avons fait le choix de ne pas noter – ∞ et + ∞ dans les tableaux de signe, afin de ne pas surcharger ces tableaux. De plus dans les appli-cations contextualisés, l’intervalle de définition n’est jamais l’ensemble des réels. Le choix de mettre deux fois le signe + (ou –) dans le cas « aucune solution » permet de faire le lien avec le signe dans les autres cas et d’aider les élèves auxquels on parle de « signe de a à l’extérieur des racines » à la maison ou sur Internet.

➜ p. 206 pour la lecture des symboles, en par-ticulier 0 ou < 0.

Les élèves doivent apprendre la traduction en termes de «  positif ou nul  » et «  strictement négatif  ». Nous avons fait le choix de ne pas parler d’inégalité « au sens large » qui n’évoque rien pour nos élèves actuels, surtout dans cette section. La notion de «  positif  » qui englobe zéro n’est pas évidente pour ces élèves.

3

Page 22: Math 1ere STMG

22 Chapitre 2 seCond degré

Réponses de l’Étude de situation p. 421. a) BR est entièrement au-dessus de l’axe des abscisses.b) L’équation R(x) = 0 n’a pas de solution. ∆ est négatif (vu page 40).c) R(x) est strictement positif. 2. a) BT est en dessous de l’axe des abscisses et le touche lorsque x = 1 .b) T(x) vaut 0 en x = 1 .c) Le tableau de signe est à interpréter orale-ment avec les élèves : « Sur R, T(x) est néga-tif » et « T(x) est nul en x = 1 ».Corrigés des Exercices d’applications p. 43

6 La droite D représentant la recette est au-dessus de la courbe C représentant le coût total pour x ∈ [ 2 ; 8 ]. Cet intervalle est la plage de bénéfice.

➜ exercice 45 p. 52 résumant les notions à connaître sur le bénéfice

7 1. P(x) = 2x2 + 4x – 6 , vu p. 41 et repré-senté p. 42.Δ = 64 et les solutions sont – 3 et 1 .

x –3 1

00P (x)

2. a) P(x) 0 . Savoir interpréter 0 en « négatif ou nul ». D’où S = [ – 3 ; 1 ] .b) 2x2 + 4x > 6 ⇔ 2x2 + 4x – 6 > 0 .S = ] – ∞ ; – 3 [ U ] 1 ; + ∞ [ .

8 a) x 2

31

02x + 3

0x − 1

00A (x)

b) A(x) = (2x + 3) (x – 1) = 2x2 – 2x + 3x – 3= 2x2 + x – 3 .On résout l’équation 2x2 + x – 3 = 0

Δ = 25 ; S = { −32

 ; 1 } .

Le coefficient de x2 est a = 2, positif, donc la parabole est tournée vers le haut. D’où le tableau de signe de A(x) :

x 23

1

00A (x)

Atelier TICE

Chiffre d'affairesL’objectif est de savoir choisir une fenêtre pour visualiser la courbe d’une fonction.

➜ p. 197 pour les Techniques de base

➜ p. 202 et 204 pour l’emploi des calculatrices

Sur TI, si l’ensemble de définition [  a  ; b  ] est connu, on utilise le Zoom Min Max, ou Zoom-Fit en anglais.

– On entre dans la fenêtre

Xmin = a et Xmax = b

– Puis Zoom 0:ZMinMax

La fenêtre a été réactualisée :

1. CA = prix × quantité. D’où :

f(x) = x ( 40x – 10 ) = 40x2 – 10x

pour x ∈ [ 0,5 ; 2 ] . Le CA est en millier d’euros.

1

Page 23: Math 1ere STMG

Chapitre 2 seCond degré 23

2. a) b)

CA min = 5 000 et CA max = 140 000 .c) On entre ces valeurs dans la fenêtre

BénéficeLe but de ce TD est de savoir vérifier les diffé-rentes formes d’un polynôme à la calculatrice ou au tableur et de les utiliser suivant la ques-tion posée. Sur TI-Nspire, completeSquare donne la forme canonique. Il existe la même instruction sur Xcas, moins facile à lire.

A. à la « main » On a , sur [ 0,5 ; 2 ] :B(x) = 20x2 – 10x – 30 .1. a) On développe 10 ( x + 1) ( 2x – 3 ) = 10 (2x2 – 3x + 2x – 3 ) = 20x2 – 10x – 30 . On retrouve B(x) .b) On résout B(x) = 0 ⇔ x = – 1 ou x = 1,5 .c) Prix p = 1,5 ∈ [ 0,5 ; 2 ] .Au prix de 1,5 € par kg, le bénéfice est nul.2. a) 20 ( x – 0,25 )2 – 31,25

= 20 ( x2 – 2×0,25 x + 0,0625) – 31,25= 20x2 – 10x – 30 = B(x) .b) Cette fonction polynôme change de varia-tion en a = 0,25 et comme a = 20, elle est décroissante, puis croissante.

Donc la fonction B est croissante sur [ 0,5 ; 2 ] inclus dans [ 0,25 ; + ∞ [ .

Tableau des variations x

B(x) 0

1,50,5

–30

302

c) L’exploitation Lu-Phi réalise un bénéfice (strictement positif ) si elle vend ses fruits à un prix x en euro par kg dans l’intervalle ] 1,5 ; 2 ] supérieur à 1,5 € par kg.3. B(1,8) = 16,8 , en milliers d’euros. Donc le bénéfice est de 16 800 € pour les fruits vendus à 1,8 € par kg.

B. à l’aide du tableur1. a) Le prix égal à 2 est en cellule A8.b) En C2 = 10*(A2+1)*(2*A2-3)c) En D2 = 20*(A2-0.25)^2-31.252. Tableau des valeurs obtenu

Quantité d'équilibreDeux objectifs pour ce TD  : la résolution d’équation pour la recherche de l’équilibre entre offre et demande et un programme de résolution de l’équation du second degré.

2

3

Page 24: Math 1ere STMG

24 Chapitre 2 seCond degré

On rappelle que la fonction d’offre est toujours une fonction croissante : elle est liée à la fonc-tion du coût marginal, elle exprime le prix uni-taire de vente en fonction de la quantité. La fonction de demande exprime la quantité demandée en fonction du prix unitaire. Pour représenter ces deux fonctions, il est néces-saire de prendre la même variable (la quantité ou le prix).Ici, la variable est le prix. f(x) et d(x) sont donc les quantités échangées sur le marché au prix x .

Visualisation sur TI-Nspire

Une difficulté pour les élèves est le problème des unités dans les exercices contextualisés. Pour le programme proposé, il nous a semblé utile de ne pas écrire les programmes de la même façon et d’en demander la lecture :sur TI, on utilise le test :

If … Then … Else …sur Casio, on utilise trois fois :

If …Then … IfEnd

➜ Voir en fin d’ouvrage les correspondances des touchesPar exemple : sur 82 ou 83 et sur 84 ou 83 (non en français) 1. a) Le prix d’équilibre est de 3 € par kg. La quantité d’équilibre est f(3) = 52 tonnes. b) f(x) – d(x) = 3x2 + 8x – 51 2. a) Initialisation :Sur TI, l’entrée des coefficients se fait par l’ins-truction Prompt : le nom de la variable s’affiche au moment de la demande de la saisie. Sur Casio, il est nécessaire de faire écrire le nom de la variable puis la demande d’entrée

se fait par ? qui est l’instruction de saisie de la calculatrice.Calcul de ∆ et affichage :Fait en ligne 2 et stocké en variable D. L’affichage par Disp sur TI

et sur Casio qui est une instruction particu-lière de la calculatrice.Tests sur ∆ et solutions :• Sur TI, on écrit If et le test sur la même ligne, puis à la ligne suivante Then seule et à la ligne suivante (ce peut être sur plusieurs lignes) l’affichage du nombre de solutions et leurs valeurs en fraction. Bien repérer l’emploi de la virgule comme séparateur. Else est seul sur une ligne. Inutile de faire un test D < 0 , puisque si D n’est pas strictement supérieur à 0 et D n’est pas égal à 0 , alors cela signifie que D est strictement inférieur à 0 .• Sur Casio, If est suivi de son test, Then est suivi de l’instruction «2 sol» qui correspond à l’affichage d’un texte, puis aux lignes suivantes l’affichage des solutions  : elles peuvent être mise en fraction si on utilise la touche dans le programme et l’entrée des données. Les instructions se terminent par IfEnd.c) a = 3 , b = 8 et c = – 51

Résultats obtenus sur TI

d) Seule la solution 3 est dans l’ensemble de définition [ 0,8 ; 6 ] . On confirme ainsi le prix et la quantité d’équilibre lu et calculé en 1.

Coût et recetteCe TD permet l’utilisation du programme pré-cédent. On peut envoyer un programme sur une autre calculatrice de même type à l’aide d’un câble de connexion.Sur TI : touches

4

Page 25: Math 1ere STMG

Chapitre 2 seCond degré 25

Choisir RECEPTION sur la calculatrice qui reçoit et attendre…Choisir ENVOI 3:Prgm…

et sélectionner le programme par puis ENVOI 1:TransmissionSur Casio: MENU

Choisir RECU sur la calculatrice qui reçoit et attendre …Choisir TRAN (F1) MAIN (F1) SEL (F1) et choisir PROGRAM .Sélectionner le programme à transmettre, puis TRAN F6 OUI F1 1. On a C(x) = 0,1x2 + 2 et R(x) = 1,05x .

1. R(x) C(x) ó 1,05x – (0,1x2 + 2 ) 0⇔ – 0,1x2 + 1,05x – 2 ≥ 0 .f(x) représente la différence entre la recette et le coût, dont le bénéfice.2. a) On résout l’équation :

– 0,1 x2 + 1,05 x – 2 = 0 .

b) a = – 0,1 (négatif ).D’où l’allure de la courbe de f .Ainsi la plage de bénéfice est [ 2,5  ; 8 ] . Pour une production comprise entre 2,5 et 8 tonnes, la recette est supérieure au coût.

Corrigés des exercices

à l’oral

9

a) c) e) f )

10 La fonction f est représentée par B2 , la fonction g par B3 , la fonction h par B1 et la fonction k par B4 .

x

g(x)00

x

k(x)12

x

f(x)1

0

x

h(x)0

–1

11

a) α = – 4 et a = 2 b) α = 0 et a = 0,1

x1x2

Page 26: Math 1ere STMG

26 Chapitre 2 seCond degré

c) α = – 100 et a = – 0,5

d) α = 12 et a = 125

12 a) 1 est solution.b) – 1 est solution.c) –1 est solution.d) 1 et – 1 sont solutions.e) 1 et – 1 ne sont pas solution.f) 1 est solution.

13

a b c Δ nb de solutionsa) 1 2 – 3 16 2

b) – 1 3 10 49 2

c) 1 – 6 9 0 1

d) 5 2 3 – 56 0

e) 1 – 9 – 10 121 2

f) – 2 3 – 5 – 31 0

14 a) a > 0 et Δ < 0b) a < 0 et Δ > 0c) a < 0 et Δ < 0d) a > 0 et Δ > 0

15 a) x

P (x)

b)x 3

0P (x)

c) x 0 2

00P (x)

d) x 2 5

00P (x)

e)x

P (x)

f) x –1

0P (x)

17 A(x) 0 , expression nulle en x = – 4B(x) > 0C(x) 0 , nulle en x = 0 ou x = –1D(x) 0 , nulle en x = – 3 ou x = 3

Pour s’entraîner

Fonction polynôme du second degré

18 A(x) avec a = – 2 ; b = – 20 et c = 650 .B(x) avec a = 0,1 ; b = – 0,2 et c = – 0,3 .

19 On a P(x) = – ( x + 3 )2 + 4 .

a) x

P(x)4–3

b) P(–1) = 0Au point d’abscisse – 1, la parabole P qui représente le polynôme P traverse l’axe des abscisses.c) P(0) = – 5 . La parabole P coupe l’axe des ordonnées au point d’ordonnée – 5 .

20 a) (x – 4)2 – 9 = x2 – 8x + 7 (x – 1) ( x – 7 ) = x2 – 8x – 7 b) 4 (x + 1)2 – 1 = 4 (x2 + 2x + 1) – 1 = 4x2 + 8x + 3(2x + 3) ( 2x + 1) = 4x2 + 8x + 3

21 On a f(x) = – 2x2 – 4x + 1 .1. a) Sommet S( – 1 ; 3 )b) Forme canonique : – 2 ( x + 1 )2 + 3c) – 2 ( x + 1 )2 + 3 = – 2 ( x2 + 2x + 1 ) = – 2 x2 – 4x + 1 = f(x)2. x

f(x)3–1

1

Page 27: Math 1ere STMG

Chapitre 2 seCond degré 27

3. f(1) = – 5 ; f(–2) = 1 ; f(–1) = 34. Équation f(x) = 1 ; S = { – 2 ; 0 }Équation f(x) = –5 ; S = { – 3 ; 1 }

22 On a f(x) = (x – 1)2 – 9 .

1. x

f(x)–9

1

2. a) (x + 2) (x – 4) = x2 – 2x – 8 .f(x) = (x – 1)2 – 9 = x2 – 2x – 8 .D’où f(x) = (x + 2) (x – 4) .b) Équation f(x) = 0 ; S = {– 2 ; 4}La courbe C traverse l’axe des abscisses aux points d’abscisses – 2 et 4 .

23 On a f(x) = ( x – 1 ) ( x – 5 )

et g(x) = −34

x2 + x + 5 .

1. a) Pour la fonction g , le coefficient a = −34

est négatif, donc la fonction g est représentée par une parabole tournée vers le bas, c’est-à-dire la bleue. La fonction f est représentée par la parabole B1, rouge.b) On a f(x) = (x – 3)2 – 4

et g(x) = −34

( x – 23

)2 + 163

2. a) Équation f(x) = 0 ; S = { 1 ; 5 }b) Équation f(x) = –3  ; S = { 2 ; 4 }c) Équation f(x) = g(x)  ; S = { 0 ; 4 }3. a) Inéquation f(x) 0  ; S = [ 1 ; 5 ]b) Inéquation f(x) – 3  ; S = ]– ∞ ; 2 ] U [ 4 ; + ∞ [c) Inéquation g(x) > f(x)  ; S = ] 0 ; 4 [

équations du second degré

24

a b c Δ nombre de solutions

1. 1 2 – 35 144 22. 2 3 5 – 31 03. 1 – 10 25 0 14. 3 5 2 1 2

25 1. a) Δ = 25 ; S = { – 3 ; 2 }

b) Δ = 0 ; S = { 32

}

2. a) Δ = – 31 ; S = ∅b) 3x2 + 2x – 40 = 0

Δ = 484 ; S = { – 4 ; 103

} .

3. a) On résout – x2 + 3x – 2 = 0 .Δ = 1 ; S = { 2 ; 1 }b) Δ = 0 ; S = { 2 }

28 a) On résout sur [ 0 ; 30 ] l’équation :

10q2 – 203q + 60 = 0 .Δ = 38 809 ; S = { 0,3 ; 20 }b) Sur [ 0 ; 40 ] : 0,01q2 – 0,5q = 0 ⇔ q ( 0,01q – 0,5 ) = 0 ⇔ q = 0 ou 0,01q – 0,5 = 0 ⇔ q = 0 ou q = 50La solution 50 n’est pas dans [ 0  ; 40 ] , d’où S = { 0 } .

29 On résout 3x2 + 5x + 1 = 0 .Δ = 13 ; deux solutions obtenues par les for-mules (à faire écrire aux élèves) :

x1 = − −5 13

6 ≈ – 1,43

et x2 = − +5 13

6 ≈ – 0,23 .

30 1. a) Équation résolue en ligne 2 :

2x2 + 3x – 5 = 0 .

b) En cellule F3 et G3, on a la même valeur 0,7 pour les deux solutions de l’équation. L’équa-tion résolue est donc de la forme a ( x – 0,7)2 = 0 ⇔ a (x2 – 1,4x + 0,49) = 0 .L’énoncé ne donne pas a et b , mais c = 49 .On en déduit 0,49 a = 49 ⇔ a = 100.D’où l’équation :

100x2 – 140x + 49 = 0 .

2

Page 28: Math 1ere STMG

28 Chapitre 2 seCond degré

c) L’équation donnée en ligne 4 a un discrimi-nant Δ = – 16 , négatif ; donc l’équation n’a pas de solution.2. Formule en D2 = B2^2-4*A2*C23. en E2 = SI(D2>=0 ; ‘‘oui ‘‘; ‘‘non’’)4. en G2 = (-B2+RACINE(D2))/(2*A2)

32

On a f(x) = (x – 0,5)2 – 6,25 sur [ – 3 ; 4 ] .1.

2. a) f(x) = x2 – x + 0,25 – 6,25 = x2 – x – 6 b) f(x) + 6 = 0 ⇔ x2 – x = 0 ⇔ x (x – 1) = 0 . D’où S = { 0 ; 1 } .3. Équation f(x) = 0 :Δ = 25 ; S = { – 2 ; 3 }La courbe C traverse l’axe des abscisses aux points d’abscisses – 2 et 3 .4. D’après la forme canonique le sommet S a pour coordonnées ( 0,5 ; – 6,25 ) .a = 1 , positif : la parabole est tournée vers le haut, d’où le tableau des variations suivant.

x

f(x)–6,25

0,5

Signe du trinôme

33 On a f(x) = – x2 + 6x – 10 et g(x) = x2 + 1,5x – 1 .1. étude de la fonction fa = – 1 , f est représentée sur la figure 1.étude de la fonction ga = 1 , g est représentée sur la figure 2.2. Équation – x2 + 6x – 10 = 0Δ = – 4 ; pas de solution

Équation x2 + 1,5x – 1 = 0 :

Δ = 254

 ; S = { – 2 ; 0,5 }

3. Tableaux de signes (lu à l’aide du graphique) x

f (x)

x –2 0,5

00g (x)

35 • On a A(x) = x2 – 3x + 28 Δ = – 103 ; A(x) = 0 n’a pas de solution , donc la parabole ne traverse pas l’axe des abscisses et A(x) reste constamment positif.

x

A (x)

• On a B(x) = – 4x2 +12x – 9 .Δ = 0  ; α = 1,5 . La parabole touche l’axe des abscisses en 1,5 . a = – 4 , d’où l’allure .

x

0

1,5

B (x)

36 • C(x) = – 0,5x2 + 7x – 24Δ = 1 ;C(x) = 0 en 6 et 8

x 6 8

00C (x)

• D(x) = 3x2 + 2x + 2Δ = – 20 et a = 3 ; la parabole a l’allure ci-contre.

x

D (x)

• E(x) = – 2x2 + 9x – 4Δ = 49 ; deux solutions 0,5 et 4

x1x2

x 0,5 4

00E (x)

3

α

x1x2

Page 29: Math 1ere STMG

Chapitre 2 seCond degré 29

• F(x) = – x2 + 4x – 4

Δ = 0 ;

une solution α = 2

x

0

2

F (x)

37

• f(x) = 3x2 + 10 . Somme d’un carré et d’un nombre strictement positif, le résultat est stric-tement positif.

x

f (x)

• g(x) = – 5x2 – 1 : c’est la somme de l’opposé d’un carré et d’un nombre strictement négatif; donc toujours strictement négatif.

x

g (x)

• h(x) = ( x – 20)2

Carré positif ou nul en x = 20 .

x

0

20

h (x)

• k(x) = – 2 (x – 7)2

Opposé d’un carré , nul en x = 7 .

x

0

7

k (x)

39

B(x) = – x2 + 60x – 500 sur [ 5 ; 55 ] .

1. a) B(22) = 336 et B(43) = 231.

Pour la vente de 22 jouets et 43 jouets, les bénéfices respectifs sont 336 € et 231€.

b) Équation – x2 + 60x – 500 = 375

⇔ – x2 + 60x – 875 = 0 .

Δ = 100 ; les solutions sont 35 et 25.

Il faut vendre 35 ou 25 jouets pour que le bénéfice soit égal à 375 €.

2. Il faut vendre 30 jouets pour que le bénéfice soit maximal et il vaut alors 400 € .

3. a) Équation – x2 + 60x – 500 = 0

Δ = 1 600 ; deux solutions : 10 et 50 .

x 105 50 55

00B (x)

b) L’entreprise réalise un bénéfice lorsqu’elle vend entre 10 et 50 jouets.L’intervalle [ 10  ; 50 ] est appelé la plage de bénéfice.

Pour approfondir

40 On a f(x) = 4 ( x – 32

)2 – 1 sur R.

1. Pour expliquer les résultats, on peut déve-lopper la première ligne et la troisième ligne du tableau, on obtiendra :

4x2 – 12x + 8 .Expand se traduit par « développer ».2. a) La forme canonique permet de dresser le tableau de variation de f : a = 4 positif, indique que la parabole est tournée vers le haut et les coordonnées du sommet de la parabole sont

( 32

 ; – 1) . D’où le tableau de variation :

x

f(x)–1

3/2

b) Intersection avec l’axe des ordonnées : on calcule f(0) = 8 .Intersection avec l’axe des abscisses : on résout f(x) = 0 ⇔ 4 (x – 2) (x – 1) = 0 .Les solutions sont 2 et 1. La parabole P traverse l’axe des ordonnées au point d’ordonnée 8 et l’axe des abscisses aux points d’abscisses 2 et 1.

41 On a f(x) = – x2 – 4x + 21 .1. f(0) = 21 . La courbe Bf traverse l’axe des ordonnées au point d’ordonnée 21.On résout f(x) = 0 ⇔ – x2 – 4x + 21 = 0 .Δ = 100 ; les solutions sont 3 et – 7 .La courbe Bf traverse l’axe des abscisses aux points d’abscisses 3 et – 7.

α

Page 30: Math 1ere STMG

30 Chapitre 2 seCond degré

2. A( 3 ; 0 ) et B( –7 ; 0 )a) α =

x xA B+=−

23 7

2 = – 2 .

b) β = f( – 2) = 25.c) g(x) = – ( x – (–2) )2 + 25 = – ( x + 2) 2 + 25 = – ( x2 + 4x + 4 ) + 25 = – x2 – 4x + 21 = f(x) .d) Comme g(x) = f(x) , on en déduit que la forme canonique de f(x) est :

f(x) = – ( x + 2) 2 + 25 .D’où le tableau de variation de la fonction f :

x

f(x)25–2

42 On a P(x) = 2x2 – 16x + 30 .1. a) 2 ( x – 4 )2 – 2 = 2 ( x2 – 8x + 16 ) – 2= 2x2 – 16x + 30 = P(x) .b) S ( 4 ; – 2) c) Tableau de variation :

x

P(x)–2

4

2. a) 2x2 – 16x + 30 = 0Δ = 16 ; deux solutions ; S = { 3 ; 5 }b) P(x) = 30 ⇔ 2x2 – 16x = 0⇔ 2x ( x – 8) = 0 ⇔ x = 0 ou x = 8 .S = { 0 ; 8 }c) P(x) = – 2 ⇔ 2 ( x – 4 )2 = 0⇔ x = 4 . D’où S = { 4 } .3. On développe 2 ( x – 3 ) ( x – 5 ) = 2 ( x2 – 3x – 5x + 15)= 2 ( x2 – 8x + 15) = 2x2 – 16x + 30 .

43 On a f(x) = x2 – 2x – 1et g(x) = 3 – x2 .1. La courbe Ca représente la fonction f et Cb représente la fonction g.2. f(x) = g(x) ⇔ x2 – 2x – 1 = 3 – x2

⇔ 2x2 – 2x – 4 = 0Δ = 36 , d’où S = { –1 ; 2 } Les courbes Ca et Cb se coupent aux points d’abscisses – 1 et 2 .3. f(x) g(x) ⇔ x2 – 2x – 1 3 – x2

⇔ 2x2 – 2x – 4 0 .2x2 – 2x – 4 s’annule en –1 et 2 .

x –1 2

002 x2 – 2 x – 4

La courbe Ca est en dessous de la courbe Cb sur l’intervalle [ – 1 ; 2 ] .

44 Fonction de demande définie par :

f(x) = – 0,1x2 + 0,7x + 9 .Fonction d’offre définie par :

g(x) = 0,4x + 3,6 .x est le prix en euro par kg de viande d’agneau ; x est compris entre 5 et 12.f(x) et g(x) sont en tonne.1. La fonction de demande est décroissante sur [ 5 ; 12 ] .2. La fonction d’offre est affine, de coefficient a = 0,4 , positif. On en déduit que la fonction d’offre est crois-sante sur [ 5 ; 12 ] .3. a) Par lecture graphique, on constate que pour x = 8, f(8) > g(8) .On en déduit que la demande est alors supé-rieure à l’offre et la demande n’est pas satis-faite.b) Si x = 8 ; f(8) = 8,2 et g(8) = 6,8 .À 8 € le kg, la demande est de 8,2 tonnes pour un offre de 6,8 tonnes : la demande n’est pas satisfaite.4. Équation f(x) = g(x) ⇔ – 0,1x2 + 0,7x + 9 = 0,4x + 3,6 ⇔ – 0,1x2 + 0,3x + 5,4 = 0

Page 31: Math 1ere STMG

Chapitre 2 seCond degré 31

Δ = 9/4 ; les solutions sont 9 et – 6 .La solution dans l’intervalle [ 5 ; 12 ] est 9 .b) D’où le prix d’équilibre de 9 € le kg pour lequel l’offre est égale à la demande.c) La quantité d’équilibre est alors :f(9) = g(9) = 7,2 tonnes.

46 1. Augmentation de 68 % CMglobal = 1,68.D’autre part :

CMglobal = p p

+⎛⎝⎜⎜⎜

⎞⎠⎟⎟⎟⎟ +⎛⎝⎜⎜⎜

⎞⎠⎟⎟⎟⎟1

1001

2100

2. a) p vérifie l’équation :

p p+⎛⎝⎜⎜⎜

⎞⎠⎟⎟⎟⎟

+⎛⎝⎜⎜⎜

⎞⎠⎟⎟⎟⎟

100100

100 2100 = 1,68

On multiplie par 100×100⇔ ( 100 + p ) ( 100 + 2p ) = 16 800⇔ 10 000 + 200p + 100p + 2p2 = 16 800⇔ 2p2 + 300p – 6 800 = 0 .Δ = 144 400 et les solutions sont – 170 et 20.On conserve la solution positive p = 20 .La première année, la production de graines a augmenté de 20 % , puis la deuxième année de 40 %.

Vers le Bac

48 1. b) 2. b) 3. c) 4. c) 5. b)

49 1. c) 2. a) b) d) 3. a) b) d)

50 Bénéfice maximal :

B(x) = –6 x2 + 90x – 250 sur [ 3 ; 10 ].Bénéfice en centaine d’euros pour la vente de x dizaines de sacs à main.1. a) Comme B(3) est négatif, on en déduit que l’artisan est déficitaire s’il ne vend que 30 sacs.b) B(4,5) = 33,5 et B(9) = 74.Pour la vente de 45 sacs et 90 sacs, les bénéfices correspondants sont de 3 350 € et 7 400 €.2. B(x) = – 6 ( x – 7,5 ) 2 + 87,5 .a)

x

B(x)87,57,5 10

503

b) Le bénéfice est maximal pour la vente de 75 sacs et vaut alors 8 750 euros.

51 C(q) = 0,1 q2 + 10q + 450, pour q ≥ 0.C(q) représente les coûts de fabrication de q objets, en euro.1. a) C(0) = 450.Les coûts fixes sont de 450 €.b) C(10) = 560 € et C(25) = 762,5 €.c) On résout l’équation :C(q) = 690 ⇔ 0,1q2 + 10q + 450 = 690⇔ 0,1q2 + 10q – 240 = 0Δ = 196 ; deux solutions – 120 et 20On conserve la solution positive q = 20 .Les coûts de fabrication de 20 objets sont égaux à 690 €.2. a) R(q) = 56qb) D’où B(q) = R(q) – C(q) = 56q – ( 0,1 q2 + 10q + 450 ) = – 0,1q2 + 46q – 450 .c) On résout l’équation :

– 0,1 q2 + 46q – 450 = 0 Δ = 1 936 et les solutions sont 10 et 450.Pour la production et la vente de 10 ou 450 objets, le bénéfice est nul. Les quantités 10 et 450 sont les points morts de la production, productions pour lesquelles l’entreprise n’a ni perte ni gain.

52 On a f(q) = – 0,8q2 – 6q + 100 .q est la quantité variant de 1 à 7 tonnes et f(q) le prix de la demande en € par kg.1. a) D’après la courbe obtenue à la calculatrice pour représenter la fonction f, la fonction de demande est décroissante sur [ 1 ; 7 ] . À l’aide de ZMinMax :

Page 32: Math 1ere STMG

32 Chapitre 2 seCond degré

b) D’après le tableau de valeurs, lorsque la quantité demandée passe de 3 à 5 tonnes, le prix passe de 74,8 € le kg à 50 € le kg.

La variation absolue du prix est – 24 € le kg. Donc si la demande augmente de 2 tonnes, le prix au kg diminue de 24 €.c) D’après le tableau de valeurs, lorsque la quantité demandée passe de 5 à 6 tonnes, le prix passe de 50 € le kg à 35,2 € le kg.

CM = 35, 2

50 = 0,704.

Soit une baisse relative de 29,6 % .Si la quantité augmente de 1 tonne, au niveau de production de 5 tonnes, le prix diminue de 29,6 %.On peut faire calculer l’élasticité du prix en fonction de la demande, entre 5 et 6 tonnes demandées :

e = P Pq q

∆ /∆ /

= −35, 5 50

501/ 5

= −29, 6

0, 2 = – 148

L’élasticité est très forte.2. a) f(q) = 50 ⇔ – 0,8q2 – 6q + 100 = 50⇔ – 0,8q2 – 6q + 50 = 0Δ = 196 ; deux solutions 5 et – 12,5.On élimine la solution négative.Le prix au kg est de 50 € lorsque la quantité demandée est de 5 tonnes.b) f(7) = 18,8Pour une quantité demandée de 7 tonnes, le prix au kg est 18,80 €.

53

Fonction d’offre définie par :

f(q) = 2q2 + 1 .Fonction de demande définie par :

g(q) = q2 – 8q + 17 ,q est la quantité variant de 1 à 4 tonnes, f(q) et g(q) sont des prix en euro par kg.

1. a) La fonction d’offre est représentée par la courbe rouge, croissante sur l’intervalle [ 1 ; 4 ] et la fonction de demande est représentée par la courbe verte. On lit le prix sur l’axe des ordon-nées.

Pour un prix de 5 € le kg, la quantité offerte est environ 1,5 tonne pour une quantité deman-dée de 2 tonnes. La demande des consomma-teurs n’est pas satisfaite.

b) Si le prix du marché est 9 € le kg, la demande est satisfaite.

2. a) On résout 2x2 + 1= x2 – 8x + 17

⇔ x2 + 8x – 16 = 0 .

Δ = 128 et 128 = 2 64× = 8 2

Deux solutions :

x1 = − −8 8 2

2 = – 4 – 4 2 = – 4 ( +2 1)

≈ – 9,657

x2 = − +8 8 2

2 = – 4 + 4 2 = 4( −2 1)

≈ 1,657

b) On élimine la solution négative et on en déduit que la quantité d’équilibre est 1 657 kg à 1 kg près.

c) Sur TI-Nspire, le premier cadre donne les solutions de l’équation : 2x2 + 1= x2 – 8x + 17 .

On y retrouve les solutions obtenues dans la question 2. a).

Le dernier calcul donne le prix d’équilibre égal à 6,49 € le kg, au centime près.

54

C(q) = 0,25q2 + 7q + 49 pour q ∈ [ 0 ; 7 ] .

q est la quantité mensuelle produite en cen-taine de pièces et C(q) est le coût total men-suel en millier d’euros.

1. a) C(0) = 49.

Coûts fixes : 49 000 €

b) C(3) = 72,25 et C(6,5) = 105,06

Page 33: Math 1ere STMG

Chapitre 2 seCond degré 33

Fabriquer 300 pièces coûte 72  250 € et en fabriquer 650 coûte 105 060 €.2. Par lecture graphique :a) La fonction C est croissante sur l’intervalle [ 0 ; 7 ] .b) Le fabricant peut produire chaque mois au maximum 600 pièces pour un coût total men-suel inférieur ou égal à 100 000 €.3. a) On résout 0,25q2 + 7q + 49 = 81⇔ 0,25q2 + 7q – 32 = 0Δ = 81 ; solutions – 32 et 4b) On garde la solution positive. Le fabricant peut produire chaque mois au maximum 400  pièces pour un coût total mensuel infé-rieur ou égal à 81 000 €.

55 1. Au prix de 300 € l’abri de jardin, on observe que la représentation graphique d de la recette est en dessous de la représentation graphique C du coût total de fabrication. Dans ce cas, les coûts étant supérieurs à la recette sur [ 0 ; 30 ] , l’entreprise ne fait pas de bénéfice.2. a) 1000 € = 10 centaines d’euros.D’où R(25) = 10 × 25 = 250 .b) La recette s’exprime en centaine d’euros et en fonction de la quantité x d’abris de jardin par :R(x) = 10x  pour x ∈ [ 0 ; 30 ] .3. La droite D est au-dessus (ou sur) de la courbe C pour x ∈ [ 6 ; 24 ] . L’entreprise réalise un bénéfice lorsqu’elle vend entre 6 et 24 abris de jardin chaque mois.4. a) B(x) = R(x) – f(x)

= x x− + −13

10 482 sur [ 0 ; 30 ]

b) On résout x x− + −13

10 482 = 0 .

Δ = 36 ; deux solutions 6 et 24

x 6 24

00B (x)

L’ensemble des solutions de l’inéquation B(x) 0 est S = [ 6 ; 24 ] . On retrouve bien le résultat de la question 3.

56

1. Le foie gras est au prix de 15,8 € le kg.

2. a) f(x) = 2,4 ( x + 0,25 )2 + 12

x est la quantité de foie gras produite, en kg, et f(x) le coût total de production en euro.

b) f(0) = 12

c) La forme canonique de la fonction f permet d’établir le sens de variation de la fonction polynôme sur R.

x

f(x)

–0,25

12

0 7

On en déduit que la fonction f est croissante sur [ 0 ; 7 ] .

3. a) En D3, l’affichage est bénéfice.

b) La société Peroi réalise un bénéfice si elle vend de 1 à 5 kg de foie gras.

Page 34: Math 1ere STMG
Page 35: Math 1ere STMG

Chapitre 3 statistique 35

Chapitre 3 Statistique

Intentions des auteuresUn chapitre assez simple, et pourtant présen-tant plus de difficultés pour les élèves que l’on croit. Tout dépend si les élèves ont vu ou non la statistique en début ou fin d’année de Seconde.Avant ce chapitre, les élèves connaissent, normalement, la médiane, au programme de Collège, et les quartiles, au programme de Seconde. Nous avons repris ces deux notions dans le cours. Le cas d’une série statistique où l’on connaît toutes les valeurs est dans l’Étude de situation du cours et le cas d’une série regroupée à valeurs isolées est dans la Mise en pratique.

➜ obtention à la calculatrice dans les Techniques de base p. 195Nous refusons de faire faire des recherches de médiane et quartiles sur une série statistique regroupée en classe : ce type de recherche n’a pas de sens concret et les valeurs obtenues par interpolation linéaire sur le polygone des fréquences cumulées donnent des valeurs très approchées. Nous avons privilégié la définition de la médiane, des quartiles et déciles donnée par l’Insee (➜ Faire le point p. 64 et exercice 19 p. 68) afin de dégager la définition du côté technique : trop d’élèves confondent la valeur du quartile Q1 avec le calcul N/4 … Il faut faire comprendre que ces caractéristiques ne se « calculent » pas, il n’y a pas de formule. En revanche, moyenne et écart type sont obte-nues par calcul à partir de formules.Nous avons repris le calcul de la moyenne en technique de base page 191.Les calculatrices donnent les valeurs. Les nou-velles Casio 35+ donnent les valeurs iden-tiques à celles de la définition. Les QCM Avant de commencer permettent de revenir sur le chapitre 1 d’une part et les

connaissances de Seconde en statistique d’autre part :– QCM1  : pour l’emploi du vocabulaire «  au moins » et « plus de » ;– QCM2 : pour une autre lecture de la médiane ;– QCM3  : moyenne avec fréquences au lieu d’effectifs ;– QCM4 : calcul de la moyenne de sous popula-tions et effet de structure.

Avant de commencer

Réponses fausses aux QCM1. c) 2. b) 3. a) 4. b)

Cours

Médiane et écart interquartile Diagramme en boîte

La médiane a été vue depuis le Collège. La situation donnant toutes les valeurs de la série, on peut aussi faire calculer la moyenne. Les 40 valeurs donnent une médiane simple à lire et des quartiles correspondants exactement à la définition de l’Insee : le quart inférieur ou égal à Q1…En donnant une 41e valeur, on peut «  voir  » rapidement le changement.

Réponses de l’Étude de situation p.58a) Pour le quart des années, le taux de chô-mage est inférieur ou égal à 5,1 % .b) 9,7 se place en 34e position. Les valeurs Xmax et Xmin ne changent pas.c) Le taux médian est le 21e taux rangé en ordre croissant, donc Me’ = 8,2 .Le quartile 1 est le 11e taux : Q1’ = 5,4 ; et le quartile 2 est le 31e taux : Q3’ = 9 .Q1 a changé.

1

Page 36: Math 1ere STMG

36 Chapitre 3 statistique

Une difficulté est de faire comprendre aux élèves que le segment [ Xmin ; Xmax ] n’est pas coupé en 4 parties de même longueur, mais ce sont les valeurs (comme des petits points) qui se répartissent en 4 groupes.

Mise en pratique p. 59Faute de place nous n’avons pu mettre les deux diagrammes en boîte des industries agroalimentaires et des distributeurs pour aider aux comparaisons.

Scores des distributeurs

x10 2 3 4 5 6 7 8 9

Scores des Industries

x10 2 3 4 5 6 7 8 9

Corrigés de l’Exercice d’application p. 59

1 Nombre de chômeurs en France (en millier par trimestre) sur toutes les années 1980.

hommes

femmes

500 550 600 650 700 750 800 850 900 950 1000 1050 1100 1150 1200450 x

Moyenne et écart typePour la moyenne, nous avons évité d’utiliser la notation S .L’écart type ne s’obtient qu’à la calculatrice et est trop difficile à justifier pour les élèves. Inu-tile de parler de la variance, non utilisé par la suite, même en Terminale sur la loi normale.Le calcul de nombreux cas permet de mieux appréhender la notion de dispersion.Le but des questions posées est de regarder ce que devient cet écart type quand on change les valeurs en gardant une moyenne constante.On peut faire cet exercice sur tableur en pre-nant ECARTYPEP(A1:A8)

Réponses de l’Étude de situation p. 601. Seul 2  130 est en dehors de l’intervalle [ 800 ; 1800 ] .2. a) À l’aide du tableur

b) À la calculatrice

c) L’écart type est beaucoup plus faible (5 fois moins) : les prix sont plus « resserrés », moins « dispersés ». 3. À l’aide du tableur

L’écart type est plus grand. On peut faire d’autres essais pour montrer la dispersion.

Mise en pratique p. 61Les élèves doivent savoir utiliser leur modèle de calculatrice pour obtenir les caractéristiques

1

Page 37: Math 1ere STMG

Chapitre 3 statistique 37

d’une série statistique (terme employé dans le programme). On parle aussi de para-mètres de la série statistique, bien que le mot paramètres soit à réserver pour les lois de probabilité.Corrigé de l’Exercice d’application p. 61

2 On entre les IMC en liste 1 et les effectifs en liste 2.

1. x ≈ 22,4 et σ ≈ 2,31 .2. [ x – σ ; x + σ ] = [ 20,09 ; 24,71 ] 106 92 38 39

400+ + +

= 275400

= 0,6875

68,75 % des femmes ont un IMC dans l’inter-valle [ x – σ ; x + σ ] .

Atelier TICE

Prix dans 80 points de venteL’objectif est de savoir calculer à l’aide du tableur toutes les caractéristiques d’une série statistique dont on connaît toutes les valeurs, non rangées en ordre croissant.

1. a) Xmin = 17,3 et Xmax = 25,5 donc :

e = 25,5 – 17,3 = 8,2 .b) En M1 =SOMME(A2:J9)2. Formules

3. Résultats

Les tests NB.SI sont des connaissances en algorithmique pour les élèves de cette série.

Part des femmes éluesLes 27 pays ne sont pas tous notés, soit ils n’étaient pas en 2004 dans l’UE, soit le nombre de parlementaires est trop faible.On peut rechercher le nombre de parlemen-taires sur le site de l’Insee, et actualiser l’exer-cice.

1. Tableau des résultats à obtenir

2. a) Parts réelles :pour 2009/2014 : 34,9

et pour 2004/2009 : 30,3 .Les parts réelles sont plus faibles, car les moyennes ne tiennent pas compte du nombre de parlementaires dans chaque pays. Or les «  petits  » pays ont un taux de femmes élues assez grand : Estonie (3 pour 6), Danemark (6 pour 13), Pays-Bas (12 pour 25) Lettonie (3 pour 8)…

1

2

Page 38: Math 1ere STMG

38 Chapitre 3 statistique

b) La moyenne des taux n’a pas de significa-tion réelle : il faudrait pondérer ces taux par le nombre de députés de chaque pays.c) Intervalles interquartiles :

pour 2009/2014 : IQ = 42,8 – 26,8 = 16

et pour 2004/2009 : IQ = 38,7 – 27,1 = 11,6 .La série des taux devient plus dispersée  : l’écart entre les pays est de plus en plus grand.

Ça dépend de la structureL’objectif est de travailler sur les proprié-tés de la moyenne et les tableaux croisés. ➜ Techniques de base p. 191

1. a) C5 D5 b) E5 2. Valeurs réactualisées

Les structures dans les deux communes étant les mêmes, les coûts moyens sont tous égaux.

Tirs au butCe TD permet de faire travailler la définition mathématique de l’écart type, en relation avec les professeurs des sections techniques qui l’utilisent parfois.1. Score moyen de 10 buts réussis pour Maxime et Victor. On peut imaginer que ce sont des séances de 20 tirs. Écart type pour Maxime : 3,4 capable de faire un bon score, mais … trop de contre perfor-mances . Et pour Victor : 1,3 moins dispersé.2. a) H1 =MOYENNE(B1:G1)I2 =ECARTYPEP(B1:G1)

b) B2 =B1-$H$1c) B3 =B2^2Résultats pour Maxime

3. Résultats pour Victor

4. a) En ligne 3, pour Maxime, plus la valeur est éloignée de la moyenne 10, plus les carrés des écarts sont grands, ce qui augmente la somme de ces écarts.Pour Victor, les carrés ne dépassent pas 4, la somme des carrés sera plus petite.b) On peut chercher par tâtonnement : mais la somme des carrés doit être égale à 6 pour que la moyenne des carrés soit 1 et la racine carrée soit 1.Ainsi, par exemple :

Corrigés des exercices

à l’oral

3

a) Xmin = 6 Xmax = 20 Me = 12Q1 = 10 Q3 = 17 .b) Xmin = 200 Xmax = 380Me = 280 Q1 = 220 Q3 = 320

4

Liste A : Me = 12 ; Q1 = 8 ; Q3 = 14

3

4

Page 39: Math 1ere STMG

Chapitre 3 statistique 39

Liste B : Me = 9,5 ; Q1 = 7 ; Q3 = 14

5

On a N = 25 et N/2 = 12,5 .Donc la médiane est la 13e valeur.N/4 = 6,25 , donc Q1 est la 7e valeur.3 × N/4 = 18,75 , donc Q3 est la 19e valeur. D’où : Me = 270, Q1 = 259 et Q3 = 279 .

6

a) Faux (25 % des personnes ont entre 54 et 60 ans)b) Vrai c) Fauxd) Vrai e) Vrai

7

1. N = 100 . Le milieu entre la 50e et la 51e valeur est dans la classe de salaire [ 1 500 ; 1 800 [ . Donc Me [ 1 500 ; 1 800 [ .La 25e valeur est dans [1  200  ; 1  500 [ , d’où Q1 [ 1 200 ; 1 500[ .La 75e valeur est dans [ 1  800  ; 2  100 [ , d’où Q3 [ 1 800 ; 2 100 [ .2. a) Vrai b) Vrai

9

1. x = 2 7 1 16 3 8 4 11

10× + × + × + ×

= 9,8

2. Il manque 2 points à Jules pour avoir 10 de moyenne.Si la note 7 passe à 8, ou si la note 16 passe à 18, Jules sera admis.3. a) Vrai

note 7 16 8 11coefficient 1 4 2 3

Dans cette situation, Jules a 12 de moyenne.b) Faux, la moyenne reste la même.c) La moyenne change car les coefficients des deux notes modifiées ne sont pas les mêmes.

10

Pour les trois séries, la moyenne est 11.La série 3 a ses valeurs les plus dispersées autour de la moyenne et la série 2 a ses valeurs les moins dispersées autour de la moyenne.

D’où :Série 1 : résultats c) x = 11 et σ ≈ 2,45 .Série 2 : résultats b) x = 11 et σ ≈ 1,77 .Série 3 : résultats a) x = 11 et σ ≈ 2,84 .

Pour s’entraîner

Médiane et écart interquartile – Diagramme en boîte

11

1. Me = 18,25 ; Q1 = 15,6 et Q3 = 19,4 .

2. Me = 750 780

2+

= 765 ;

Q1 = 395 et Q3 = 970 .3. Tableau des valeurs

poids en g 27 28 29 30 31 32effectif 4 5 18 47 16 10effectif cumulé 4 9 27 74 90 100

Me = 30 , Q1 = 29 et Q3 = 31

13

1. 200 véhicules sont contrôlés.2. a)

vitesse [ 20 ; 30 [ [ 30 ; 40 [ [ 40 ; 50 [

effectif 21 47 82

effectif cumulé 21 68 150

vitesse [ 50 ; 60 [ [ 60 ; 70 [ [7 0 ; 80 [

effectif 41 6 3

effectif cumulé 191 197 200

b) 150200

= 0,75 ; 75 % des véhicules roulent à

moins de 50 km/h.3. Me ∈ [ 40 ; 50 [ , Q1 ∈ [ 30 ; 40 [ et Q3 ∈ [ 40 ; 50 [ .

14

1. a) Répartition de la clientèle A suivant le nombre de factures mensuelles :xmin = 4 xmax = 21 Me = 12

1

Page 40: Math 1ere STMG

40 Chapitre 3 statistique

Q1 = 7 Q3 = 15 b) IQ = Q3 – Q1 = 82. Diagrammes en boîte

A

B

8 10 12 14 16 18 204 6 x

15

Pour les données de la figure 1, l’effectif total est de 60 et 25 sur 60 ont au plus 36 ans, soit moins de 50 %. Donc la figure 1 représente la répartition pour le service B, car le diagramme en boîte de B a sa médiane après 36.2. a) Vrai, car le service A est plus jeune :Me (B) = 36,5 et Q3 (A) = 34,5.b) Vrai : les 75 % des plus jeunes du service B ont au plus 42 ans. Or 75 % des plus jeunes du service A ont au plus 34,5 ans, donc ils sont plus jeunes que 75 % du service B. c) Attention : édition 1. lire B au lieu de A dans la question : « Au moins la moitié du service A … du service B ».Vrai : la moitié du service A a moins de 32 ans, car Me (A) = 30,5 , et dans le service B, comme Q1 (B) = 32,5 , il y a moins du quart qui a moins de 32 ans.d) Vrai : Q3 (B) = 42 et Q3 (A) = 34,5.

16

1. Tableau des effectifs

note 7 8 9 10 11effectif 4 9 13 7 13effectif cumulé

4 13 26 33 46

note 12 13 14 15 16effectif 18 11 12 9 5effectif cumulé 64 75 87 96 101

N = 101 ; N/2 = 50,5 > 46 donc la médiane est la 51e note, d’où Me = 12 .N/4 = 25,25 donc Q1 est la 26e note  ; d’où Q1 = 9 .

3 × N/4 = 75,75 > 75 donc Q3 est la 76e note, d’où Q3 = 14 .

2. Diagramme en boîte des notes

17 1. a) Listes sur calculatrice

b) Résultats : Me = 2 , Q1 = 1 et Q3 = 3 .

2. Diagramme en boîte

18

1. La formule en D3 est : =D2+C3Au total, ce client a payé 133 factures.2. • 133/2 = 66,5  : la 67e facture est de 225 €. Donc Me = 225 .• 133/4 = 33,25  : la 34e facture est de 140  €, donc Q1 = 140 .

Page 41: Math 1ere STMG

Chapitre 3 statistique 41

• 3 × 133/4 = 99,75 : la 100e facture est de 275 €, donc Q3 = 275 .

3. Calcul de la ristourneEn cellule F2, on place la formule =B2*C2 et on recopie vers le bas jusqu’en F13.En F16, on calcule =SOMME(F2:F7)En F17, on calcule =SOMME(F8:F13)

0,03 × 16145 + 0,05 × 15975 = 1283,1€

19

1. Diagramme en boîte élagué aux déciles

2 3 4 5 6 7 8 9 10-1 0 1

2. a) Vrai car Me =2,2 .b) Faux  : comme D9 = 8,6 , au plus 10  % de valeurs dépassent 8,6 µg /m3, donc moins de 10 % des valeurs dépassent 10 µg /m3.c) Vrai, par définition du premier décile.d) Vrai : 80 % est la proportion des valeurs com-prises entre les deux déciles.

e) Faux : 91

8, 60, 6

=DD ≈ 14 .

Moyenne et écart type

20

1. 10 120 5 180

15× + ×

= 140 .

Le bénéfice moyen est de 140 € par jour, sur les 15 jours.2. Le jour où la taxe est payée, le bénéfice du jour passe de 140 € à 115 €.

D’où : 3 (120 25) 7 120 5 180

15× − + × + ×

= 135 ;

ou encore : 15 140 3 25

15× − ×

= 135 .

Le bénéfice moyen de Fanny est de 135 €.

22

1. Série A : fig. 2 et série B : fig. 1.La moyenne est plus basse en A qu’en B, et il y a plus de valeurs faibles dans la fig 2. 2. Résultats pour la série A

Résultats pour la série B

23

1. Résultats à la calculatrice

N = 53 ; x ≈ 8,6 ; σ ≈ 4,9 .2. [ x – σ ; x + σ ] = [ 3,7 ; 13,5 ]

6 14 953

+ + =

2953

≈ 0,5471

Environ 54,7 % des valeurs de la série sont dans l’intervalle [x – σ ; x + σ ] .

24 Série regroupée en classe

centre de classe 50 125 200 375effectif 14 11 31 9

2

Page 42: Math 1ere STMG

42 Chapitre 3 statistique

Le montant moyen des factures est de 180 €, arrondi à 10 € près, et l’écart type est σ ≈ 100 . On ne peut pas donner une valeur arrondie trop «  fine  », car prendre le centre de classe signifie que la répartition est homogène.On peut faire faire aux élèves un essai  : par exemple, en prenant pour centre de classe 75 pour la 1re et 300 pour la seconde, ce qui est fort plausible, et on obtient une moyenne de 174 et un écart type de 71.Donc donner les résultats arrondis à 10 est plus correct.

Le calcul serait correct si on connaissait la moyenne dans chaque classe.

26

1. À Caen : x ≈ 3,7 et σ ≈ 0,9 .

À Aix-en-Provence : x≈ 4,6 et σ ≈ 1,4 .

L’indice moyen ATMO est plus bas à Caen, et les valeurs sont moins dispersées. On peut interpréter en disant que la qualité de l’air est meilleure à Caen.

2. Pour la ville de Caen :

[ x – σ ; x + σ ] = [ 2,8 ; 4,6 ]

153 157365+

= 310365

≈ 0,85

Pour 85  % des jours de l’année 2010, la valeur de l’indice ATMO a été dans l’intervalle [ x – σ ; x + σ ] .Pour la ville d’Aix-en-Provence :

[ x – σ ; x + σ ] = [ 3,2 ; 6 ]

112 76 43317

+ + =

231317

≈ 0,73

Pour 73  % des jours de l’année 2010, la valeur de l’indice ATMO a été dans l’intervalle [ x – σ ; x + σ ] .

Pour approfondir

27 1. Étude du lot Aa) Si la moyenne est égale à la médiane, la valeur de la moyenne se lit sur la gradua-tion repérée par le trait rouge de la boîte. Ici : Me = x = 55 .b) Q1 = 40, d’après le diagramme en boîte, on en déduit que 75 % des bulbes ont une masse supérieure ou égale à 40 g.c) Intervalle [ Q1 ; Q3 ] = [ 40 ; 70 ] .50 % des bulbes du lot A pèsent entrent 40 et 70 g.2. Étude du lot B

a) La masse moyenne d’un bulbe du lot B est 36 g.b) Résultats à la calculatrice

Page 43: Math 1ere STMG

Chapitre 3 statistique 43

Me = 35 ; Q1 = 30 et Q3 = 45 .c) Diagrammes en boîte

Lot A

Lot B

20 30 40 50 60 70 80 90 1000 10

d) Le lot B semble le mieux calibré, la longueur de la boîte et l’étendue sont plus petites que celles du lot A. Les masses pour le lot B sont moins dispersées autour de la médiane.e) Dans le lot B, 85 bulbes sur 120 ont une masse strictement comprise entre 25 et 55 g, soit 71 % , à 1 point de % près.

28

1. a) On classe les valeurs en ordre croissant :

2 3 6 6 6 8 11 14 15 19

D’où par lecture directe :

Me = 6 8

10+

= 7 ; Q1 = 6 et Q3 = 14 .

b) Résultats à la calculatrice

Le nombre moyen de devis demandés chaque jour est 9 et σ ≈ 5,3 .2. a) Si la valeur 19 est remplacée par 24, comme il s’agit de la valeur maximale de la série, la médiane et les quartiles ne changent pas. On a toujours Me = 7 , Q1 = 6 et Q3 = 14 .b) Par contre la moyenne et l’écart-type aug-mentent.

La moyenne devient x = 9,5 et σ ≈ 6,3 .

La moyenne et l’écart type ont été influencés par cette valeur maximale qui a augmenté.3. Troisième série :

13 15 3 6 11 2 14 6 19 6

Comme dans la question 2, en ajoutant 5 devis supplémentaires, la moyenne devient x = 9,5 .Q1 et Q3 ne changent pas, car 8 ou 13 sont deux valeurs contenues dans la boîte.

29

Résultats sous Excel :

1. Moyenne des CA : 2,9 millions d’euros. Et σ ≈ 0,798 » 0,8 .2. a) En B3 =B2-(0,9*B2+0,2)b) Moyenne des bénéfices : 0,09 millions d’euros = 90 000 € et écart type σ ≈ 0,0798 ≈ 0,08 .

30

1. a) Hommes : 60 salariésFemmes : 80 salariésb) Masse salariale des 30 ouvriers hommes :

30 × 1 561 = 46 830 € .c) Salaire moyen des hommes :( 3 × 4 372 + 7 × 2 180 + 20 × 1 491

+ 30 × 1 561)/60 = 105 026

60 ≈ 1750 € .

d) Salaire moyen des femmes :122 511

80 ≈ 1531 € .

Page 44: Math 1ere STMG

44 Chapitre 3 statistique

2. Montant du salaire moyen dans l’entreprise :

60 1 750   80 1 531140

 × + ×

≈ 1625 € .

Vers le Bac

32 1. Résultats sur Excel

a) Nombre moyen de naissances par an en France métropolitaine entre 1901 et 1920  : environ 740 milliers, résultat arrondi à la dizaine de milliers. b) Étendue : 79 milliers.2. a) Pour la moitié des années, le nombre de naissances a été inférieur à 826 400.b) Diagramme en boîte

3. a) Me = 765 ; Q1 = 740 et Q3 = 770 .b) Étendue : 805 – 720 = 85 .4. a) Faux  : de 1981 à 2000, il y a 19 ans et Q3 = 770 . Donc le nombre annuel de nais-sances est supérieur à 770 000 seulement sur le quart de la période, c’est-à-dire moins de 5 ans.b) Vrai  : on compare les étendues, soit 532,4 milliers pour la première période et 85 milliers pour la deuxième. Or 5 × 85 = 425. On a bien 532,4 > 425 .5. La période 1901-1920 inclut la première guerre mondiale de 1914 à 1918, ce qui

explique le nombre de naissances particuliè-rement faible pendant les années de guerre et la plus grande étendue pour cette période du début du XXe siècle.

33

1. a) En B22 =SOMME(B2:B21)b) En C2 =A2*B2c) Valeur moyenne des dépassements de la vitesse autorisée par véhicule :

1301125

≈ 10,4 km/h .

d) En D3 =D2+B3En D10 : 59 véhicules.En D11 : 63 véhicules.e) Me = 10 ; Q1 = 5 et Q3 = 15 .f) 59 véhicules ont dépassé la vitesse autori-sée de 9 km/h, on en déduit que 66 véhicules ont dépassé la vitesse autorisée de 10 km/h ou plus, cela correspond à un pourcentage de 52,8 %.2. a) xmin = 2  ; xmax = 15  ; Me = 8  ; Q1 = 5  ; Q3 = 10 .b) Faux : la moitié des automobilistes en excès de vitesse dépasse la vitesse autorisée de 8 km/h ou plus.

Page 45: Math 1ere STMG

Chapitre 3 statistique 45

Vrai : la médiane est passée de 10 à 8 km/h.3. a) 100/250 = 0,4 . 40 % des conducteurs ont modifié leurs habitudes.b) 60/125 = 0,48 . 48 % des femmes interro-gées ont modifié leurs habitudes.c) Parmi les personnes ayant modifié leurs habitudes, 40 % sont des hommes.

34

Partie11. a) x ≈ 8,6  et σ ≈ 3,5 .[ x – σ ; x + σ ] = [ 1,6 ; 15,6 ]b) On peut chercher le pourcentage de valeurs à l’extérieur de cet intervalle :

5 10 5 6 8700

+ + + + =

34700

≈ 0,049 ,

soit environ 5 %.On en déduit qu’environ 95 % des valeurs sont dans [ 1,6 ; 15,6 ] .c) Le nombre de films vus au cinéma dans l’an-née par 95 % des élèves est dans l’intervalle [ 1,6 ; 15,6 ] , donc de 2 à 15 .2. a) Me = 9 ; Q1 = 6 et Q3 = 11 .b) 75 % des élèves ont vu au moins 11 films au cinéma dans l’année.Partie 21. Faux : dans la classe A, environ le quart des élèves a vu moins de 9 films au cinéma.2. Vrai  : 8 élèves représentent le quart de la classe et 10 est la valeur de Q3. On applique la définition de Q3.3. Vrai : la médiane pour la classe A est supé-rieure à Q3 pour la classe B.

36

1. a) En 2009, la Pologne a vu son PIB augmen-ter de 1,7 %.b) La plus forte diminution de PIB en 2009 a eu lieu en Lettonie avec une baisse de – 18 % .c) Le plus grand écart, avec une baisse de 28 points de pourcentage, s’est produit en Letto-nie entre les années 2007 et 2009.d) Pour l’ensemble des pays de l’Union euro-péenne, en 2007, le PIB a augmenté de 2,9 % et il a baissé de 4,2 % en 2009.2. a) • N/2 = 27/2 = 13,5 . Donc la médiane est la 14e valeur : Me = 4,5 , qui est le taux de crois-sance du PIB de la Grèce en 2007 (revu depuis la crise de la dette en 2011).• 27/4 = 6,75, donc Q1 est la 7e valeur : Q1 = 2,6, taux de croissance du PIB du Royaume-Uni. Q3 = 6,5 ; taux de croissance du PIB au Luxem-bourg.b) Par définition de Q1, en 2007, 75 % des pays de l’UE ont un PIB qui a augmenté d’un taux supérieur à 2,6 %, ce qui est supérieur au taux de 2,4 % réalisé par la France. Donc l’affirma-tion est vraie.c) Le taux de croissance du PIB en Finlande, égal à 4,9  %, est supérieur à la valeur de la médiane (Me = 4,5). Donc la Finlande fait par-tie des 50 % des pays qui ont eu la plus forte croissance du PIB en 2007, avant la crise ban-caire de 2008.3. a) En 2009, le quart des pays de l’UE a vu son PIB diminuer davantage que celui de l’Irlande.b) Le quart des pays de l’UE a vu son PIB moins diminuer que celui de la Belgique.

Page 46: Math 1ere STMG
Page 47: Math 1ere STMG

Chapitre 4 étude de fonCtions 47

Chapitre 4 Étude de fonctions

Intentions des auteuresLe nouveau programme de Première STMG privilégie l’étude du sens de variation d’une fonction à l’aide de la dérivée directement.

En Première, seules les fonctions polynômes de degré 2 et 3 sont étudiées, les fonctions rationnelles étant vues en Terminale (plus d’ex-ponentielle et de fonction ln en Terminale).

Aussi l’accent est mis sur le calcul direct de la dérivée, l’étude de son signe donnant le sens de variation de la fonction, sans s’attarder sur les formules de dérivées.

Le nombre dérivé vient après le calcul de la dérivée, en tant qu’application de la dériva-tion, pour permettre de déterminer une équa-tion d’une tangente. Nous avons conservé la notation K pour le nom du point de tangence, et k pour son abscisse : il permet d’éviter des erreurs de reconnaissance, la lettre a étant réservée au coefficient de x2 dans le polynôme du second degré.

L’équation réduite d’une droite étant de la forme y = m x + p , pour éviter toute confusion. Nous avons insisté sur la reconnaissance du coefficient directeur sur un graphique. Notre pratique en classe nous a conduit à faire lire ce coefficient en commençant par la différence des ordonnées entre deux points de la droite, car une fraction s’écrit en commençant pas le numérateur. Trop d’élèves lisent l’inverse. Ainsi, la lecture se fait dans le même «  sens  » que l’écriture et – 3/4 signifie «  je descends de 3 pour un décalage de 4 ».

L’étude de fonction polynôme de degré 3 per-met de réinvestir le signe du trinôme.

Nous avons mis l’accent sur l’utilisation de ces fonctions dans le cadre de résolution de problème, reprenant les applications « écono-miques » qui sont devenues classiques : étude de coûts, de bénéfice, de recette ou chiffre

d’affaires, de rythme de croissance… offre demande, équilibre… un peu de modélisation, mais le programme de Terminale sera encore plus adapté à ce type d’exercices.

Les QCM Avant de commencer permettent de revenir sur le second degré, les lectures gra-phiques sur les droites, le sens de variations et le signe d’une expression.

On pourra reprendre les techniques de base, si les notions ne sont pas encore acquises.

Avant de commencer

Réponses fausses aux QCM1. b) 2. b) 3. c) 4. a) 5. b)

Cours2 Fonction polynôme

du second degré : dérivéeL’étude de telles fonctions a été vue au cha-pitre 2, à partir de la forme canonique, vue en Seconde.

Nous n’avons pas trop insisté. La dérivée étant admise, nous avons donc donné directement sa forme, sans aucune approche, puisque le nombre dérivé et la tangente sont des consé-quences du calcul de la dérivée. Simplement, nous avons proposé le lien entre parabole et signe de la dérivée dans la partie À retenir.

Un logiciel de géométrie dynamique permet facilement de montrer ce lien. C’est ce que nous proposons dans l’Atelier TICE 1 p. 82.

Dans la mesure du possible, nous avons évité de parler de la fonction f ’, préférant parlant de la fonction dérivée de f . Nous nous sommes

1

Page 48: Math 1ere STMG

48 Chapitre 4 étude de fonCtions

permis les abus : « Calculer la dérivée » et « la dérivée de x2 est 2x », plus facile pour les élèves de cette série.

Réponses de l’Étude de situation p. 76a) L’écran de calculatrice montre le sommet de la parabole, ce qui permet de donner le tableau des variations.

x

f(x)4860 10

0–60

b) et c) Signe de – 6x + 36

6

++

––

On fait facilement le lien.d) Lecture de la forme canonique : comme a = – 3 , α = 6 et β = 48 , en x = 6 , donc pour 60 bijoux, le bénéfice est maximum, et il vaut 48 €.2. Apprentissage de ce qui a été vu en 1.À la calculatrice, on peut faire dessiner la fonction dérivée.

3. Application pour une autre fonction, le but étant de travailler une fonction telle que a est positif, afin de voir les deux cas.a) h’(x) = 8x – 12 et h’(x) = 0 ⇔ x = 1,5b) Visualisation sur la calculatrice

c) La fonction ne peut être un bénéfice, car la fonction h est positive en 0 (quand la vente est nulle, le bénéfice est négatif, car il y a les coûts) et le bénéfice augmente très forte-ment pour de grandes quantités.

Voir Atelier 1 p. 82 pour visualiser la parabole et la droite de la dérivée.

Mise en pratique p. 77L’objectif est l’utilisation de la fonction dérivée pour retrouver les variations du trinôme. Nous avons choisi de donner les courbes représentatives de deux fonctions dont on ne va garder qu’un morceau. Notre pratique en classe a montré que les élèves ont beaucoup de mal quand on n’utilise qu’une partie des informations.De plus, les fonctions d’offre et de demande se prêtent bien aux études de trinôme  : le modèle est souvent approprié.Il faut savoir que, économiquement, les fonctions d’offre sont liées au coût marginal.

Corrigés des Exercices d’application p. 77

1 Sur [ 0 ; 10 ] on définit :C1(x) = 3x2 + 24x + 100C2(x) = 3x2 + 24x + 150C3(x) = 3x2 + 24x + 245Ces trois fonctions de coût ont la même fonction dérivée :

C ’(x) = 6x + 24 .Sur [ 0 ; 10 ] , x est positif, donc 6x + 24 est positif . On peut aussi étudier le signe :

6x + 24 = 0 ⇔ x = – 8donc pour x > – 8 , alors 6 x + 24 > 0 .La fonction dérivée est positive sur l’inter-valle [ 0 ; 10 ] .On en déduit que les trois fonctions C1 , C2 , et C3 sont croissantes sur [ 0 ; 10 ] .

2 La dérivée de f est représentée par la droite 2 , car 2 traverse l’axe des abscisses en 2 et la courbe Bf a son sommet en x = 2 .La dérivée de g est représentée par la droite 3 .La dérivée de h est représentée par la droite 1 .Pour résoudre cet exercice, on peut faire dresser le tableau des variations de la fonc-tion de courbe B, et indiquer le signe que doit avoir la dérivée, puis faire le lien an demandant le signe de mx + p pour chaque droite .Il est plus facile pour ces élèves de faire un tableau ou un schéma plutôt qu’une phrase.

Page 49: Math 1ere STMG

Chapitre 4 étude de fonCtions 49

2 Équation d'une tangente à une parabole

Le programme indique comme application l’interprétation du nombre dérivé en tant que coefficient directeur de la tangente. Rien n’em-pêche de déterminer des tangentes à d’autres courbes. En Première, nous avons respecté la détermination d’une équation seulement pour les trinômes.Dans tout l’ouvrage, nous avons réservé la cou-leur verte pour ce qui a trait aux tangentes et dérivées.Dans la Situation, nous sommes parties de la représentation graphique d’une tangente, en A où la courbe est assez « ronde » pour faire penser à un cercle avec sa tangente.Les logiciels dynamiques sont très utiles pour cette notion : ci-dessous sous TI-Nspire.

Ci-dessous, le tracé d’un cercle qui approche la courbe au point de tangence M et peut donc ancrer cette notion de tangente dans l’esprit des élèves.Ce cercle a pour centre le point d’intersection de la perpendiculaire à la tangente au point M avec l’axe de symérie de la parabole d’équation x = 4 .

Corrigés de l’Étude d’une situation p. 781. Aux alentours de 5, l’écart maximal entre f(x) et t(x) est 0,09 . On peut expliquer que la courbe est pratiquement confondue avec sa tangente (épaisseur du crayon).Ce qui fait que la croissance de la « droite » se retrouve sur la courbe.2. a) 2 a pour coefficient m = – 4 .Et f ’ (2) = 2 × 2 – 8 = – 4.Le nombre dérivé en 2 est le coefficient directeur de la tangente 2 en 2.b) f ’ (4) = 2 × 4 – 8 = 0La droite passant par S de coefficient direc-teur  0 est la droite horizontale d’équation y = 3 . En la traçant, on reconnaît la tangente. Le logiciel dynamique le confirme.

3. f ’ (3) = 2 × 3 – 8 = – 2 et f (3) = 4Sur le graphique, la droite semble être la tan-gente au point d’abscisse 3 .Faire le lien en cherchant une équation de la droite passant par C ( 3 ; 4 ) et de coefficient directeur – 2 .

y = – 2 ( x + 3 ) + 4 ⇔ y = – 2 x +10Mise en pratique p. 79Détermination de tangentes et modélisation concrète. On peut dire aux élèves que les grecs pensaient que la chaînette était une parabole.Visualisation de la courbe représentative

Page 50: Math 1ere STMG

50 Chapitre 4 étude de fonCtions

Edition 1 : au b), il manque « y = f ’ (–2) ».

Corrigés des Exercices d’application p. 79

3 On a f (x) = – x2 + 4x – 1 .a) f ’ (x) = – 2x + 4À l’écran de la calculatrice, on observe que le coefficient directeur de la tangente au point d’abscisse 1 est 2.D’où f ’ (1) = 2 . On le vérifie en remplaçant x par 1 dans f ’ (x) . On obtient f ’ (1) = 2 .b) Équation de la tangente à la courbe au point d’abscisse 0 : y = 4x – 1 .Et au point d’abscisse 3 : y = – 2x + 8 .

4 On a f (x) = 0,5 x2 + x – 1 .a) On obtient f ’ (x) = x + 1 .f ’ (2) = 3 et f ’ (–1) = 0 .b) Tracé de la courbe et des tangentes (à faire aussi sur calculatrice) ➜ pages 202 à 205 pour le tracé

2 3 4-1-2-3-4-5

2

3

4

5

6

-1

-2

-3

-4

0 1

1

x

y

A

B

3 Fonction polynôme de degré 3

Comme les opérations sur les dérivées ne sont au programme qu’en Terminale, la dérivée est admise.

Nous avons voulu présenter les différentes formes que l’on rencontre pour ces fonctions.

Un logiciel dynamique avec curseur est très adapté (comme l’Atelier 1 en prenant quatre cur-seurs, ou avec tableur comme dans l’Atelier 3).

Réponses de l’Étude de situation p. 801. a) D’après la parabole tracée :

x 0

–10

f (x)

0 012

+

b) Sur [ 0 ; 10 ] , la production est croissante.c) Sur [ 10  ; 12 ] , la fonction f est décrois-sante. Malgré l’investissement, la production est décroissante.d) Tableau des variations

x 0

500

–10

f ’(x)

f (x) 0

012

432

+

Au maximum, on peut produire 500 sacs.

2. On a g(x) = –x3 + 15x2 – 52 .On a g ’ (x) = f ’ (x) . Donc l’étude du sens de variation est la même.

Page 51: Math 1ere STMG

Chapitre 4 étude de fonCtions 51

b) Pour un investissement de 10 k€, la pro-duction est maximale, et elle est de 448 sacs :

f (10) – 52 = 500 – 52 = 448 .c) Représentation sur [ 0  ; 12 ] en utilisant

ZMinMax puis la fenêtre Y ∈ [ – 100 ; 500 ]

Lorsque x > 2, alors g(x) est positif : la courbe est au-dessus de l’axe des abscisses ; on peut utiliser les fonctionnalités graphiques.

Mise en pratique p. 81Le choix a été fait d’étudier le signe de la déri-vée (second degré) en un seul schéma sur la parabole  : cela va souvent plus vite pour les élèves et est plus compréhensif que des phrases.Le support concret peut permettre de don-ner des idées aux élèves sur ces courbes. Sur le bijou, on peut retrouver l’origine du repère et les deux courbes.Ci-dessous à l’aide de TI-Nspire :

Les soudures sont sous les brillants.

Corrigés des exercices p. 81

5 a) On a f(x) = – x3 + 1,5x2 + 18x + 25 sur [ – 3 ; 5 ] .Donc f ’(x) = – 3x2 + 3x + 18 .Δ = 225 les solutions sont 3 et – 2 .

3 5a < 0

–2 +

–––3

x –3 –2f ’(x) + – – f(x) 11,5

3

03

65,50

5

27,5

b) f (x) = x3 + 5x + 21 sur [ – 3 ; 4 ]f ’(x) = 3x2 + 5 toujours strictement positif comme somme de positifs. Donc la fonction est croissante sur [ – 3 ; 4 ] .On peut simplement faire le schéma suivant :

5

– 21

x –3 4

105 f ’ (x)

f (x)

+

Atelier TICE

1 Parabole et droiteOn va faire le lien entre le signe de la dérivée et le sens de variation de la fonction poly-nôme du second degré. On peut aussi faire le lien avec la forme canonique et la valeur de α, abscisse du sommet de la parabole et valeur annulant la dérivée.

1. a) a = 1 ; b = 2 et c = – 1 , donc :

f (x) = x2 + 2x – 1 . Et f ’ (x) = 2x + 2 .

Faire faire le tableau des variations de f avec le signe de la dérivée :

x –∞

–2

+10

f ’(x)

f (x)

0+∞

b) g(x) = – 0,5 x2 – 2 x + 2 .

g’(x) = – x – 2 : faire résoudre – x + 2 = 0 et représenter sur la calculatrice la courbe pour vérifier.

x –∞

4

+–2

g ’(x)

g (x)

0+∞

Le maximum est 4, atteint en – 2 .

Page 52: Math 1ere STMG

52 Chapitre 4 étude de fonCtions

2. Résultats obtenus sous TI-Nspire :

a) f (x) = – x2 + 3x + 4 et f ’ (x) = – 2x + 3

b) On a f (x) = 2 x2 – 6x et f ’ (x) = 4x – 6 .

Résultats obtenus sous GeoGebra :

c) On a f(x) = 0,5 x2 – 4x + 2 .

et f’(x) = x – 4 . Sous TI-Nspire :

2 Coût supplémentaireL’objectif est de faire comprendre la notion de coût marginal qui sera utilisé dans les exercices.

On peut aussi faire le lien entre l’accroissement moyen du coût entre deux productions (ici x et x + ∆x , avec ∆x de plus en plus petit), accrois-sement par kg, et le coût marginal, coût supplé-mentaire pour un kg quand on en a produit x .

Cet atelier est à faire avec un petit groupe d’élèves motivés. On peut faire faire des exer-cices plus «  répétitifs  » pour le reste de la classe, ou simplement montrer cette activité au tableau.

Nous avons cherché à bien mettre en place le vocabulaire des coûts, pour les élèves qui vont en BTS.

1. a) Le coût de 2 kg, en plus des 3 kg produits, est la différence des ordonnées entre les points A et D .∆∆

yx

est le quotient de la différence des ordon-

nées yD – yA sur la différence des abscisses xD  –  xA . C’est le coefficient directeur de la droite (AD) .

b) Si on bouge le point M sur la courbe, en

se rapprochant du point A, le quotient ∆∆

yx

diminue.

La droite (AM) se rapproche de la droite verte tracée, tangente à la courbe B en A.

2.a) f (7) est donné par B2 = B1^2 + 1 ou B2 = B$1^2 + 1

b) B4 = B2 – $I$2

3. a) Comme f (x) = x2 + 1 , alors f ’ (x) = 2x et f ’ (3) = 6 .

b) La droite verte semble avoir pour coefficient directeur 30/5 = 6 .

Figure de l’édition 2:

On retrouve le nombre dérivé en 3 .

Page 53: Math 1ere STMG

Chapitre 4 étude de fonCtions 53

c) L’équation réduite de la droite verte esty = 6 (x – 3 ) + 10 ⇔ y = 6x – 8 .4. Le coût marginal par kg, quand on en achète 4 kg, est f ’ (4) = 8 , soit 8 € par kg.On peut l’approcher par le calcul sur tableur :

3 Fonction sur tableurL’étude d’une fonction polynôme est grande-ment facilitée par le tableur, surtout si les points particuliers ont des abscisses entières. La repré-sentation du nuage de points est ci-après.1. a) f (x) =1 × x3 + 3 × x2 + 0 × x + (– 4)= x3 + 3x2 – 4b) f ’ (x) = 3 x2 + 6xOn saisit la formule : C2 = 3*A2^2+6*A2c) En A14, le zéro signifie que x = 0 .Les zéros en colonne C signifient que la dérivée est nulle.Pour C10, la dérivée est nulle en x = – 2 et pour C14 , la dérivée est nulle en x = 0 .Les zéros de la colonne B correspondent aux images, donc l’ordonnée est égale à 0  : pour B10, la courbe traverse l’axe des abscisses en x = – 2 et pour B16, la courbe traverse l’axe des abscisses en x = 1 .

2. On a f (x) = – x3 + 3x2 + 24x – 20 .a) b) Tableau de valeurs et courbes

c) Tableau des variationsx –2

f ’(x) + – – f (x)

–48

04

600

Corrigés des exercices

à l’oral

7

1. f ’ (x) = 2ax + b

2. Réponses b) f ’ (x) = 6x .

g ’ (x) = – 4x – 1 et h ’ (x) = –2x .

8

a) f ’ (x)= – 8x b) f ’ (x) = 4x + 5

c) f ’ (x) = –6x + 2 d) f ’ (x) = 3x .

9

f(x) =3x2 – 12x + 5 sur [ – 3 ; 5 ]

Bonnes réponses :

2. La fonction change de variation en 2.

4. b) f ’ (2) = 0 .

5. Le tableau de variation de f est exact.

Page 54: Math 1ere STMG

54 Chapitre 4 étude de fonCtions

10

1. La fonction dérivée d’une fonction poly-nôme du second degré est de la forme f ’ (x) = 2ax + b . Donc f ’ est une fonction affine, représentée par une droite.

2. La fonction f1 change de sens de variation en –1, sa dérivée est donc représentée par la droite d.

La fonction f2 change de sens de variation en 3, sa dérivée est donc représentée par la droite .

On peut aussi faire dresser le tableau des varia-tions pour f1 et f2 .

12

1. a) C( 4 ; 2 ) . b) f ’ (4) = 3 .

2. f (–2) = 2 et f ’ (–2) = – 3 .

3. f ’ (x) = 0 pour x = 1 .

On repère l’abscisse du point de la parabole où la tangente est horizontale, c’est l’abscisse du sommet A.

4. f (0) = – 2 et f ’ (0) = – 1 .

13

La fonction f est croissante sur [ – 4 ; – 2 ] et sur [ 3 ; 4 ] . La fonction f est décroissante sur [ –2 ; 3 ] . Elle admet un maximum au point d’abscisse – 2 et un minimum au point d’abscisse 3.

Signe de la dérivée  : à faire lire aux élèves ➜ notations p. 206 .

f ’ (x) > 0 pour x ∈ [ – 4 ; – 2 [ ∪ ] 3 ; 4 ] .

f ’ (x) < 0 pour x ∈ ] – 2 ; 3 [ .

f ’ (x) = 0 pour x = – 2 ou x = 3 .

Pour s’entraîner

1 Fonction polynôme du second degré : dérivée

14

a) f’(x) = 2x + 3

b) f’(x) = 10x + 2 c) f’(x) = –2 x + 1

15

a) f ’ (x) = 8x + 7

b) f ’ (x) = 4x c) f ’ (x) = x – 3,5

16

a) f ’ (x) = 260x – 50

b) f ’ (x) = 3x – 7 c) f ’ (x) = 2 × 106 x

18

a) g ’ (x) = x + 1

b) C ’ (q) = –2q + 6 c) f ’ (t) = t + 4

19

a) h ’ (x) = –18 x

b) R ’ (q) = – 40 q +6400

c) B ’ (x) = – 0,24 x + 0,85

20

On a f (x) = 2x2 + 4x – 3 .

1. f (0) = –3 ; P coupe l’axe des ordonnées au point de coordonnées ( 0 ; – 3) .

On résout l’équation 2x2 + 4x – 3 = 0 .

Δ = 40 ; deux solutions :

x1 = – 1 – 12

10 ≈ – 2,6

et x2 = – 1 + 12

10 ≈ 0,6 .

La parabole P traverse l’axe des abscisses aux points d’abscisses – 2,6 et 0,6 (à 0,1 près).

2. a) f ’ (x) = 4x + 4

b) c) Signe de la dérivée et tableau des varia-tions de la fonction

x

–5

+–1

f ’(x)

f (x)

0–

21

x

4

+3 7 –1

4 –4

f ’(x)

f (x)

0 –

Page 55: Math 1ere STMG

Chapitre 4 étude de fonCtions 55

x

–3

+–2 2

55

–6 g ’(x)

g (x)

0–

22

A et 2 B et 1 C et 3

23

a) Faux b) Faux c) Vrai

24

On a f (x) = x2 – 6x + 5 sur [ 0 ; 7] .

1. a) f ’ (x) = 2x – 6

b) c) Signe de la dérivée et tableau des varia-tions de la fonction

x

–4

+ 3 7

125

0 f ’ (x)

f (x)

0–

2.a) S ( 3 ; – 4 ) ; β = – 4 représente le minimum de la fonction f sur [ 0 ; 7 ] .

b) La parabole P traverse l’axe des abscisses aux points d’abscisses 1 et 5 .

c) On résout x2 – 6x + 5 = 0 .

Δ = 16  , on obtient deux solutions : 1 et 5 .

25

1. On a f(x) = – x2 + 2x .

a) f’(x) = –2x + 2

b) c) Signe de la dérivée et tableau des varia-tions de la fonction

x

1

+1

f ’(x)

f (x)

0 –

La fonction f est représentée par la parabole P2 de sommet S (1 ; 1) .

2. On a g(x) = 2x2 – 4x + 5 .

a) g ’(x) = 4x – 4

b) c) La fonction g est représentée par la para-bole P2 de sommet S (1 ; 3) .

27

Dans l’édition 01, il faut lire f (t) = – 0,4 t2 + 60 t + 80 .

1. f ’ (t) = – 0,8 t + 60 : nul en t = 75 .

2. Signe de f ’ (t) et tableau des variations

t

2 330

+75 100 20

2 0801 120

f ’(t )

f (t)

0 –

3. Valeur minimale fmin = 1  120 € par heure pour une utilisation pendant 20 heures.

Valeur maximale fmax = 2 330 € par heure pour une utilisation pendant 75 heures.

On retrouve ces valeurs en utilisant Zoom Min Max de la calculatrice.

28

On a CT(q) = q3 – 8q2 + 32q ; où q est en millier d’objets et CT(q) en k €

et q ∈ ] 0 ; 8 ] .

1. a) CM(q) = +– 8 323 2q q q

q = q2 – 8q + 32

pour q ∈ ] 0 ; 8 ] .

b) La fonction CM est une fonction polynôme du second degré.

c) Le coût moyen s’exprime en euro par objet et n’est pas défini en 0 .

d) CM(2) = 20 ; CM(7) = 25

Le coût moyen d’un objet est de 20 € lorsque 2 milliers d’objets sont fabriqués et le coût moyen d’un objet est de 25 € lorsque 7 milliers d’objets sont fabriqués.

2. a) CM ’ (q) = 2q – 8 .

b) c) Signe de CM ’ et tableau des variations. En 0 , CM n’existe pas.

q

16

+4 80

CM ’(q)

CM (q)

0–

Il faut fabriquer 4 000 objets pour que le coût moyen soit minimum et dans ce cas, il vaut 16 euros par objet fabriqué.

Page 56: Math 1ere STMG

56 Chapitre 4 étude de fonCtions

2 équation de la tangente à une parabole

29

1.

droite 1 2 3 4

coefficient directeur 2 –1

12

–3

2. a) Une droite parallèle à l’axe des abscisses a un coefficient directeur nul.

b) Une droite parallèle à l’axe des ordonnées n’a pas de coefficient directeur.

c) La droite (AB) passant par A ( 1 ; 1 ) et B ( 2 ; 2 ) a un coefficient directeur égal à 1 .

30

Droite coeff m équation1 –2 y = –2x + 52 1/3 y = 1/3 x + 13 5/2 y = 5/2 x – 2

31

1. Le coefficient directeur de la tangente à la courbe au point A d’abscisse –2 est égal à 3 . D’où f ’ (–2) = 3 .

2. f (4) = 1 et f ’ (4) = – 3 .

33

Figure de l’exercice 32

a) Équation de la tangente en A :

y = – 2 ( x + 6 ) + 2 ⇔ y = – 2x – 10 .

Équation de la tangente en C :

y = 3 ( x – 4 ) + 7 ⇔ y = 3x – 5 .

b) Équation de la tangente en B :

y = – x – 5 .

Équation de la tangente en S : y = – 2 .

34

1. a) f (3) = 2 et f ’ (3) = – 3

b) f (1) = 4 et f ’ (1) = 1

c) f (0) = 2 et f ’ (0) = 3

2. f ’ (x) = 0 pour x = 1,5

1,5 est l’abscisse du point où la tangente est horizontale.

En admettant que la courbe soit une parabole de sommet S, on obtient par symétrie :

l’abscisse de S est la demi-somme des abs-

cisses des points A et B, xS = +2

x xA B = 1,5 .

35

Figure de l’exercice 34

Équation de la tangente en A :

y = –3 ( x – 3 ) + 2 ⇔ y = –3x + 11 .

Équation de la tangente en B :

y = 3x + 2 .

37

On a f (x) = 0,25x2 – 2x + 2 .

1. f ’ (4) = 0 f ’ (2) = –1 f ’ (8) = 2

2. f ’ (x) = 0,5x – 2 . On peut alors vérifier les résultats précédents.

3. a) f ’ (x) = 1 ⇔ x = 6

Solution : a = 6 .

b) Équation de la tangente au point d’abscisse a = 6 .

y = f ’ (6) ( x – 6 ) + f (6)

⇔ y = 1 ( x – 6 ) – 1 ⇔ y = x – 7

38

On a f (x) = 0,5x2 – 4x + 6 sur [ 0 ; 9 ] .

1. a) f ’ (x) = x – 4 . Signe de la dérivée et tableau des variations de la fonction :

x

–2

+4 9

10,56

0 f ’ (x)

f (x)

0–

2. a) Tableau de valeurs :

x 0 2 4 6 8 9f (x) 6 0 –2 0 6 10,5f ’ (x) –4 –2 0 2 4 5

On peut l’obtenir sur tableur comme dans l’Atelier 3.

Page 57: Math 1ere STMG

Chapitre 4 étude de fonCtions 57

b) c) Courbes et tangentes

2 3 4 5 6 7 8 9 10-1

2

3

4

5

6

7

8

9

10

-1

-2

0 1

1

x

y

3 Fonction polynôme de degré 3

40

On a f (x) = x3 + bx2 + cx + d .

1. f ’ (x) = 3x2 + 2 b x + c

2. a) f ’ (x) = 3x2 + 2x + 1

b) f ’ (x) = 6x2 + 8x + 3

41

a) f ’ (x) = 9 x2 – 2x – 2

b) f ’ (x) = – 3 x2 + 3x – 10

42

a) f ’ (x) = 1,5 x2 – 2x + 2

b) f ’ (x) = –15 x2 c) f ’ (x) = 12 x2 – 10

43

a) f ’ (x) = 0,03x2 – 0,5x + 0,4

b) f ’ (x) = 2x2 – x

c) f ’ (x) = –3 x2 + 30x – 70

44

a) C ’ (q) = 3q2 – 12q + 13

b) B ’ (q) = q2 – 22 q

c) f ’ (t) = 3t2 – 12t + 30

45

f (x) = x3 – 12x2 + 45x + 20 sur [ 0 ; 10 ] .

a) f ’ (x) = 3x2 – 24x + 45 .

b) Signe de la dérivée et tableau des variations de la fonction

x 0 3f ’(x) + + – f(x)

20740

5

70

010

270

46

On a f (x) = 0,2 x3 – 2,1x2 + 3,6x + 6 sur [ 0 ; 30 ] .

a) f ’ (x) = 0,6x2 – 4,2x + 3,6

b) c) Signe de la dérivée et variation :

x 0 1f ’(x) + + – f(x)

67,70

6

–4,8

030

3624

La fonction est décroissante sur [ 1 ; 6 ] .

47

a) Tableau de signe de la dérivée :

x –2–4f ’(x) + + –

01 4 0

b) On obtient :

f ’ (x) 0 pour x ∈ [ – 4 ; – 2 ] ∪ [ 1 ; 4 ] .

49 f (x) = x3 – 3x2 + 1 sur R

1. Fonction croissante sur ] – ∞  ; 0 ] et sur [2 ; + ∞ [ , et décroissante sur [ 0 ; 2 ] .

2. a) Tableau de signe du produit :

x 03x (x – 2) + + –

02 0

b) f ’ (x) = 3x2 – 6x = 3x ( x – 2 )

c) Signe de la dérivée et variation :

x 0f ’(x) + + – f(x) 1

02

–3

0

Page 58: Math 1ere STMG

58 Chapitre 4 étude de fonCtions

50

f (x) = 13

x3 – 3x2 + 5x + 8 sur [ –2 ; 7 ] .

1. a) En A3 = $A2 + 1

b) Les deux erreurs : parenthèses à supprimer dans le premier terme et carré à ajouter dans le deuxième terme : en B2 , la formule à saisir est donc : = 1/3*A2^3+(-3)*A2^2+5*A2+8

c) La formule proposée est correcte pour obte-nir la fonction dérivée.

2. Signe de la dérivée et variation :

x –2 1f ’(x) + + – f(x)

–16,710,30

5

–0,33

07

10,3

L’équation f (x) = 0 semble avoir trois solutions : la première entre – 2 et 1 , la deuxième entre 1 et 5 et la troisième entre 5 et 7 .

51

1. a) Conjecture du sens de variation de la fonction f

x –7 –3

f(x) –147

613

–47

8 303

b) f (x) = x3 – 27x + 7 sur [ –7 ; 8 ]

2. a) f ’ (x) = 3x2 – 27 = 3 ( x2 – 9 )

= 3 ( x + 3 ) (x – 3)

b) Signe de la dérivée et variation :

x –7 –3f ’(x) + + – f(x)

–147610

3

–47

08

303

53

On a f (x) = x3 – 24x2 + 180x + 60 sur [ 0 ; 15 ] .

a) f ’ (x) = 3x2 – 48x + 180

b) Δ = 144 ; deux solutions : 6 et 10 .

c) d) Signe de la dérivée et variation :

x 0 6f ’(x) + + – f(x)

604920

10

460

015

735

55

f (x) = 2x3 – 18x2 + 57x sur [ 0 ; 8 ]

a) f ’ (x) = 6x2 – 36x + 57

b) Δ = – 72 : pas de solution ; la dérivée ne s’an-nule pas et reste positive, car a = 6 .

c) d) Signe de la dérivée et variation :

x 0f ’(x) + f(x)

0

8

328

56

f (x) = 0,1x3 – 10,5x2 + 300x + 150 sur [ 0 ; 60 ]

a) f ’ (x) = 0,3x2 – 21x + 300

Δ = 81 : les solutions sont 20 et 50 .

Signe de la dérivée et variation :

x 0 20f ’(x) + + – f(x)

1502750

050

1400

060

1950

57

1. a) Le coefficient directeur de la tangente en un point de la courbe d’abscisse k est le nombre dérivé de la fonction en k noté f ’ (k) .

b) Aux points A et B, les tangentes sont hori-zontales ; on en déduit que :

f ’ (–1) = 0 et f ’ (1) = 0 .

2. Signe de la dérivée et variation :

x –1f ’(x) + – – f (x)

1

01

50

3. a) f (0) = 3 et f ’ (0) = 3 .

Équation de la tangente à la courbe au point D :

y = 3x + 3 .

b) De l’équation réduite de la tangente au point C , soit y = – 9x + 19 , on déduit :

f(2) = 1 et f ’ (2) = – 9 .

Page 59: Math 1ere STMG

Chapitre 4 étude de fonCtions 59

58

D’après l’exercice 57, on connaît le signe de la dérivée f ’ (x) , on en déduit que la première courbe ne convient pas.

On sait que f ’ (0) = 3 , donc la troisième courbe ne convient pas.

Donc la fonction dérivée f ’ est représentée par la courbe B2 .

59

f (x) = – x3 + 12x + 2 sur [ – 6 ; 7 ] .

a) En A3 = A2 + 1

On arrête la recopie en A15.

b) En B2 =-A2^3+12*A2+2

c) Tableau de valeurs  : il semble que la fonction change de sens de variation en – 2 et en 2 . La fonction semble décrois-sante sur [ – 6 ; – 2 ] et sur [ 2  ; 7 ] , et c r o i s s a n t e s u r [ –2 ; 2 ].

2. a) b) f ’ (x) = –3 x2 + 12 = 3 ( 4 – x2 )

= 3 ( 2 + x ) ( 2 – x ) .

c) Signe de la dérivée et variation :

x –6 –2f ’(x) – – + f(x) 146

–14

02

180

7

–257

60

On a f (x) = x3 – 15x2 + 448 sur [ 0 ; 8 ] .

1. a) Par lecture graphique, la fonction f est décroissante sur [ 0 ; 8 ] .

b) f ’ (x) = 3x2 – 30x = 3x ( x – 10 )

x 03x (x – 10) + + –

010 0

D’où f ’ (x) 0 sur [ 0 ; 8 ] (dérivée négative ou nulle).

c) On en déduit que la fonction f est décrois-sante sur [ 0 ; 8 ] .

2. a) f (0) = 448 : on en déduit que la hauteur maximale du panneau est de 448 cm = 4,48 m.

b) f (4) = 272 . Donc, à 4 m du bord, la hauteur du panneau est 272 cm = 2,72 m.

61

On a M (t) = 45t2 – t3 pour t ∈ [ 0 ; 25 ] .

1. a) M ’ (t) = –3t2 + 90t et M ’ (5) = 375

La vitesse de propagation le cinquième jour est de 375 nouvelles personnes par jour atteintes de cette maladie.

b) La vitesse de propagation de la maladie est une fonction polynôme du second degré, avec un coefficient a = – 3, négatif : la para-bole qui la représente est tournée vers le bas et le maximum de M ’ (t) est atteint pour t = 15 .

La vitesse de propagation est maximale le15e jour et vaut M ’ (15) = 675 .

c) M ’ (t) > 600 ⇔ 90t – 3t2 > 600

⇔ –3t2 + 90t – 600 > 0 .

Δ = 900 : deux solutions : 20 et 10 .

20 2510 +

––0

On en déduit le signe sur [ 0 ; 25 ] de A(t) = –3t2 + 90t – 600 .

t 100A (t ) – – +

020 25 0

Du 10e au 20e jour, la vitesse de propagation est supérieure à 600 personnes par jour.

2.a) M ’ (t) = –3t2 + 90t = 3t ( – t + 30) .

3025

+––

0

On en déduit que la fonction dérivée M ’ est positive sur [ 0 ; 25 ] , par conséquent la fonc-tion M est croissante sur [ 0 ; 25 ] .

b) Courbe et tangentes en A , B et C.

Page 60: Math 1ere STMG

60 Chapitre 4 étude de fonCtions

4 6 8 10 12 14 16 18 20 22 24-2

2000

3000

4000

5000

6000

7000

8000

9000

1E4

1,1E4

0 2

1000

x

y

A

B

C

Sur [ 10 ; 15 ] , le coefficient directeur de la tangente augmente, puis il diminue sur [ 15 ; 20 ] .

Pour approfondir

62

1. a) La fonction de coût est croissante sur [ 0 ; 100 ] .

b) Les coûts fixes sont 30 € .

2. a) f (q) 60 à partir de 60 objets fabriqués.

b) f (q) 90 pour une quantité fabriquée infé-rieure à 90 objets.

3. a) Coefficient directeur de  : a = 1

b) g (70) = 70 . Pour 70 objets vendus, le chiffre d’affaires est 70 € .

4. a) Le bénéfice est positif pour la fabrication et la vente d’un nombre d’objets compris entre 55 et 90 .

b) B (10) = – 30 ; B (30) = – 20 ; B (55) = 0

B (70) = 10  et B (90) = 0 .

5. Au prix de 0,80 € l’objet :

a) 100 objets sont vendus 80 € .

b) Le chiffre d’affaires est :

h (q) = 0,8q pour q ∈ [ 0 ; 100 ] .

c) La représentation graphique du chiffre d’af-faires est la droite d passant par l’origine et le point de coordonnées ( 100 ; 80 ) .

Dans ce cas, la courbe du coût est au-dessus de la droite d. On en déduit que l’entreprise ne peut pas réaliser des bénéfices avec ce prix unitaire puisque les coûts seront toujours supérieurs au chiffre d’affaires.

63

On a f (x) = 0,05x2 + x + 35 sur R .

1. a) f ’ (x) = 0,1x + 1 .

Signe de la dérivée et variation :

x

30

+–10

f ’(x)

f (x)

0–

b) f (0) = 35 et f (50) = 210

c) f (x) = 275 ⇔ 0,05x2 + x + 35 = 275 ⇔ 0,05x2 + x – 240 = 0

Δ = 49 : deux solutions : – 80 et 60 .

f (x) > 275 ⇔ f (x) – 275 > 0 ⇔ 0,05x2 + x – 240 > 0

On trace l’allure de la parabole.

x –80f (x) – 275 + + –

060 0

D’où S = ] – ∞ ; – 80 [ ∪ ] 60 ; + ∞ [ .

2. C (x) = 0,05x2 + x + 35 sur [ 0 ; 80 ] .

Coût de production, en millier d’euros, de x millier de flacons de parfum.

a) La fonction C est confondue avec la fonction f sur [0 ; 80 ] , d’où les résultats suivants.

a) La fonction C est strictement croissante sur [ 0 ; 80 ] .

b) Coûts fixes : C (0) = 35 000 euros.

c) C (50) = 210 000 euros.

D’où le coût moyen de fabrication d’un flacon si l’entreprise en fabrique 50 000 dans le mois,

soit 21050

= 4,2 € par flacon.

d) La stratégie de diminution des coûts sera mise en place dès que la production mensuelle dépassera 60 000 flacons.

64

1. a) En A3 =A2-1

Page 61: Math 1ere STMG

Chapitre 4 étude de fonCtions 61

b) En B3 =B2+50

c) En C2 = A2*B2

2. La recette est maximale si la place est à 10 € .

3. a) Pour une réduction de x euros, à partir d’un billet à 12 €, le nombre de places vendues est 400 + 50x.

b) On en déduit la recette, sur [ 0 ; 12 ] :

R (x) = ( 12 – x ) ( 400 + 50x ) .

4. a) R (x) = – 50x2 + 200x + 4800 .

b) c) R ’ (x) = – 100 x + 200 .

On résout : – 100 x + 200 = 0 ⇔ x = 2 .

Signe de la dérivée et variation :

x

5000

+2 12 0

04800

R’(x)

R (x)

0 –

5. a) À 10 € le billet, la recette est maximale.

b) À ce prix, le directeur attend 500 spectateurs et une recette de 5 000 € .

65

1. On a f (t) = – 0,02 t2 + 0,16 t + 82,18 sur [ 0 ; 13 ] .

a) f (9) = 82 et f (10) = 81,78 .

Δ f = 81,78 – 82 = – 0,22 .

De 2009 à 2010, l’Allemagne a perdu 220 000 habitants.

b) f (13) = 80,88

80 880 000 habitants prévus en 2013.

2. f’(t) = – 0,04t + 0,16 . On résout :

– 0,04t + 0,16 = 0 ⇔ t = 4 .

Signe de la dérivée et variation :

x

82,5

+4 13 0

80,0882,18

f ’ (t )

f (t)

0 –

c) f ’ (9) = – 0,2 . Ce qui signifie qu’il y a une dimi-nution de 200 000 habitants par an en 2009.

On trouve une valeur proche de la variation absolue obtenue en 1. a) .

d) À très long terme, ce modèle ne convient pas car il conduit à des populations négatives.

66

On a f (x) = ax2 + bx + c .

1. a) En début d’algorithme, on entre les coef-ficients a et b.

Inutile d’entrer c qui n’intervient pas dans le calcul de la dérivée.

b) –B/(2*A) permet de calculer la valeur qui annule f ’ (x) .

c) Affichage de la dérivée : 2AX+B

2. a) Sur TI : Prompt A,B

Disp '' DERIVE 2AX+B '' , 2*A , '' X+ '' , BSur Casio : ''A'':?→A:''B''?→B

b) Programme TI

Programme Casio

d) Pour f (x) = – 0,01x2 + 0,3x – 4 , on obtient : f ’ (x) = – 0,02x + 3 .

67

On a f (x) = – 0,15x2 + 1,8x + 3 .

1. a) La droite n’est pas tangente à la courbe Bf .

b) Équations des droites

d 1

y = –1,2 (x – 10 ) + 6 y = 0,3x+0,3 y = 8,4

2. a) f (x) – ( 0,3x + 3 )

= – 0,15x2 + 1,8x + 3 – 0,3x – 3

= – 0,15x2 + 1,8x = 0,15x ( – x + 10 )

Page 62: Math 1ere STMG

62 Chapitre 4 étude de fonCtions

b) f (x) = 0,3x + 3

⇔ f (x) – ( 0,3x + 3 ) = 0

⇔ 0,15x ( – x + 10 ) = 0

⇔ 0,15x = 0 ou – x + 10 = 0

D’où S = { 0 ; 10 } .

La droite et la courbe Bf se coupent aux points d’abscisses 0 et 10 .

68

1. a) Cm(x) = 1,4 + 0,02x pour x quantité en tonne.

Cm(x) est le coût marginal en euro d’un kg sup-plémentaire quand on a produit x tonnes de chocolat. On peut faire travailler l’Atelier 2 p. 82 à ce sujet.

Comme x > 0, on a Cm(x) > 0 .

b) On calcule Cm(4) = 1,48 .

Si l’usine produit 4 tonnes, le kg supplémen-taire revient à 1,48 € .

2. C(x) = 25 + 1,4x + 0,01x2

Le coût total est en millier d’euros.

a) C(0) = 25 . On retrouve ainsi les coûts fixes de 25 000 €.

b) C ’ (x) = 1,4 + 0,02 x .

c) En C2 =1,4+0,02*A2

C(4,001) – C(4) = 0,00148001

Cela représente le coût supplémentaire en mil-lier d’euros d’un kilo de chocolat quand on en a produit 4 tonnes, soit 1,480 01 € par kg.

Or Cm(4) = 1,48 €. la différence est très faible, de 0,000 01 €.

d) C(4,1) – C(4) = 30,908 1 – 30,76

= 0,148 1 millier d’euros.

148,1 € représentent le coût supplémentaire de 100 kg de chocolat quand on en a produit 4 tonnes.

Un de ces 100 kg supplémentaire revient à 1,481 €.

La différence avec Cm(4) est alors de 0,001 €, ce qui reste encore très faible.

69

1. a) Il y a saturation pour une durée de 4 h de travail quotidien.

b) Il y a envie sur l’intervalle [ 0 ; 4 ] .

c) Il y a rejet sur l’intervalle [ 4 ; 8 ] .

d) v (4) = 0

2. La fonction d’envie, dérivée de la fonction de satisfaction f , est une fonction affine v de la forme :

v (x) = ax + b , définie sur [ 0 ; 8 ] .

D’après le graphique :

v (0) = 50 ⇔ b = 50 .

De plus, v (4) = 0 ⇔ a × 4 + 50 = 0

⇔ a = 504

− = –12,5 .

D’où v (x) = –12,5 x + 50 .

3. f (x) = – 6,25 x2 + 50 x sur [ 0 ; 8 ] .

a) f ’ (x) = – 12,5x + 50

b) f ’ (x) = 0 ⇔ x = 4

Signe de la dérivée et variation :

x

100

+4 8 0

0 0

f ’(x)

f (x)

0 –

On retrouve la durée de saturation égale à 4 h pour laquelle la fonction de satisfaction prend la valeur 100.

Vers le Bac

71

f (x) = –3x2 + 7x + 6 sur R .

1. b) f(–1) = – 4

Page 63: Math 1ere STMG

Chapitre 4 étude de fonCtions 63

2. b) f (x) = ( 3 – x ) ( 3x + 2 )

3. a) f ’ (x) = – 6x + 7

4. a) y = 13x + 9

5. b) [ 0 ; 1 ]

72

Une seule réponse exacte.

1. b) Faire faire le tableau des variations de g avec le signe de la dérivée g ’ (x) pour conclure.

2. d) 3 solutions

3. c) y = – 2

4. c) g (x) = 0,125 x3 – 0,75x2 + 2

5. b) y = – 1,5 x + 3

73

Partie A

f (x) = x2 + x + 15 pour x ∈ [ 0 ; 18 ] .

x est le nombre de CD produits et f (x) le coût journalier de production, en euro.

1. f (0) = 15 ; coût fixe journalier : 15 € .

f (10) = 125 ; coût journalier pour 10 CD pro-duits : 125 € .

2. f ’ (x) = 2x + 1 ; f ’ (x) > 0 sur [ 0 ; 18 ] .

3. Signe de la dérivée et variation :

x 0f ’ (x) + f (x)

15

18

357

4. Tableau de valeurs

x 0 2 4 6 8f (x) 15 21 35 57 87x 10 12 14 16 18f (x) 125 171 225 287 357

5. Voir la courbe ci-après.

Partie B

1.a) R (x) = 17x

b) Voir ci-après.

c) Le bénéfice est positif pour une production allant de 1 à 15 CD.

d) On cherche l’abscisse pour laquelle l’écart des ordonnées entre la recette et le coût est maximal.

Par lecture graphique, on estime que la pro-duction journalière de 8 CD engendre un bénéfice maximal.

4 6 8 10 12 14 16 18

80

120

160

200

240

280

320

0 2

40

x

y

CRCf

2. a) Bénéfice sur [ 0 ; 18 ]

B(x) = 17x – x2 – x – 15 = – x2 + 16x – 15

b) B ’ (x) = –2x + 16

c) Signe de la dérivée et variation :

x

49

+8 18 0

–51 –15

B’(x)

B (x)

0 –

d) Le résultat de la question 1. d) est confirmé.

Le bénéfice maximal est atteint pour une pro-duction journalière de 8 CD.

e) Bénéfice journalier maximal : 49 € .

74

1. a) f(x) = 52x sur [ 0 ; 16 ]

b) g (x) = x3 – 18x2 + 108x sur [ 0 ; 16 ]

c) L’équation f (x) = g (x) a trois solutions : 0, 4 et 14 en lignes 2, 6 et 16.

2. a) g (x) – f(x) = x3 – 18x2 + 56x

= x ( x2 –18x + 56 )

b) On résout x2 –18x + 56 = 0 .

Δ = 100 ; deux solutions : 4 et 14 .

Page 64: Math 1ere STMG

64 Chapitre 4 étude de fonCtions

c) D’où l’ensemble des solutions de l’équation :

g (x) = f (x) ⇔ g (x) – f (x) = 0

⇔ x ( x2 –18x + 56 ) = 0

⇔ x = 0 ou x2 –18x + 56 = 0

On retrouve l’équation précédente.

D’où S = { 0 ; 4 ; 14 } .

d) On a justifié ainsi les résultats obtenus dans la question 1.

75

Partie A

C

CR

20 30 40 50 60 70

200

300

400

500

600

0 10

100

q

y

1. a) C (60) ≈ 460 k€ (coût)

b) R(60) ≈ 490 k€ (recette)

c) La production mensuelle de 60 milliers de réveils est rentable, puisque R (60) > C(60) .

2. Pour une production mensuelle de 4 à 63 milliers de réveils, le bénéfice est positif.

Partie B

On a C (q) le coût total mensuel de production en millier d’euros et P (q) le prix de vente uni-taire exprimé en millier d’euros par millier de réveils.

C (q) = 0,1q2 + q + 40et P (q) = 11,2 – 0,05q ,

pour q millier de réveils, avec q ∈ [ 0 ; 72] .

1. a) R(10) = P(10) × 10

= ( 11,2 – 0,05 × 10) × 10 = 107 k€

b) Chiffre d’affaires mensuel :

R(q) = « prix » × « quantité » = P (q) × q

= ( 11,2 – 0,05q) × q = – 0,05q2 + 11,2q .

2. a) B (q) = R(q) – C(q)

– 0,05q2 + 11,2q – (0,1q2 + q + 40 )

= – 0,15q2 + 10,2q – 40 .

D’où B’(q) = – 0,3q + 10,2 .

b) B’(q) = 0 ⇔ q = 34 .

Signe de la dérivée et variation :

q

133,4

+34 72 0

–83,2 –40

B’(q)

B (q)

0 –

c) Avec 34  000 réveils fabriqués et vendus chaque mois, le bénéfice est maximal et vaut 133 400 €.

76

On a C (x) = 2 x3 – 30 x2 + 200 x , coût de pro-duction journalier, en euro, hors coûts fixes, de x litres de solvant, avec x ∈ [ 0 ; 15 ] .

C

4 6 8 10 12

1000

1500

2000

2500

3000

0 2

500

quantité en L

Coût en €

1. La fonction C semble croissante sur l’inter-valle [ 0 ; 15 ] .

2. Résolution à l’aide de TI-Nspire

a) Le coût de production de 10 litres est de 1000 €.

b) La troisième ligne du calcul formel donne C ’ (x) .

Page 65: Math 1ere STMG

Chapitre 4 étude de fonCtions 65

c) C ’ (x) = 0 n’a pas de solution.

3. a) Signe de 6x2 – 60x + 200 :a = 6 , positif6x2 – 60x + 200  est stricte-ment positif sur R .

b) Signe de la dérivée et variation :

x 0C’ (x) + C(x)

0

15

3000

Le sens de variation de la fonction C sur [ 0 ;15 ] est justifié.

77 On a P(t) = 100 ( – t3 + 6t2) .

t : temps de travail en heure, t ∈ [ 0 ; 5 ] ;

P(t) : production exprimée en objet.

1. a) P (1) = 500

b) P (2) = 1600

En 2 heures de travail, 1 600 objets sont pro-duits.

c) La quantité d’objets n’est pas proportion-nelle au temps de travail.

De 1 h à 2 h de travail, le temps est multiplié par 2 et la production par 3,2 .

2. a) P (t) = – 100t3 + 600t2

P ’ (t) = – 300t2 + 1 200t = 300t ( – t + 4 )

P ’ (t) : rythme de production par heure.

b) P ’ (2) = 1 200

Soit 1  200 objets produits à l’heure pour un temps de travail de 2 heures.

c) P ’ (t) = 0 ⇔ 300t ( – t + 4 ) = 0 ⇔ t = 0 ou t = 4

d) Signe de la dérivée et variation

t

3 200

+ 4 50

2 5000

P’(t )

P (t)

0 –

e) Un temps de travail de 4 heures engendre une production maximale de 3 200 objets.

78

Fonction f en Y1 et dérivée f ’ en Y2

1. f (x) = x3 – 7,5x2 + 12x

f (0) = 0 et f (1) = 1 – 7,5 + 12 = 5,5 .

2. a) La dérivée d’une fonction polynôme de degré 3 est une fonction polynôme du second degré qui s’annule deux fois au maximum.

b) c) Par lecture des 0 de la colonne Y2, on obtient f ’ (x) = 0 pour x = 1 ou x = 4 .

x 0 1 4 6f ’ (x) + 0 – 0 +

Signe de la dérivée et variation d’après le tableau de valeurs :

x 0 1f ’(x) + + – f(x)

05,50

4

– 8

06

18

3. a) f ’ (x) = 3x2 – 15x + 12

b) Δ = 81 ; deux solutions : 1 et 4 .a = 3, positif

x 0 1 4 6f’(x) + 0 – 0 +

c) Le signe de f ’ (x) donné en b) est ainsi justifié.

79

1. En B2 =A2-1950

2. a) En 1995 : 21,7 % des conseillers munici-paux sont des femmes .

En 2001  : 33% . Donc augmentation de 11,3 points de pourcentage.

b) CM = 33

21, 7 ≈ 1,5207 , soit une augmenta-

tion de 52,1 % à 0,1 % près.

Formule pour obtenir ce résultat en cellule E11 =C11/C10*100-100

Page 66: Math 1ere STMG

66 Chapitre 4 étude de fonCtions

Si la cellule E11 est au format pourcentage, la formule devient = C11/C10 -1

3. Modélisation de la part des femmes maires par la fonction f suivante :

f(x) = 0,005x2 – 0,064x + 0,83

où x est le rang de l’année 1950 + x et x ∈ [ 0 ; 80 ] ; et f(x) est la part en pourcentage des femmes parmi les maires.

a) En 2014, on obtient f (64) ≈ 17,2 .

Ainsi, en 2014, on peut prévoir que, parmi les maires, 17,2 % seront des femmes.

b) f ’ (x) = 0,01x – 0,064

On résout f ’ (x) = 0 ⇔ x = 6,4 .

Signe de la dérivée et variation :

x

0,6252

+6,4 80

27,710,83

0 f ’ (x)

f (x)

0–

Ce modèle ne convient pas pour les années de 1950 à 1960, c’est-à-dire pour x ∈ [ 0 ; 6 ] ; en effet, le pourcentage des femmes aurait été plus important au début des années  50 qu’à la fin des années 50, cela ne correspond pas à la réalité.

c) Résolution de f (x) > 20

On établit une table de valeurs de la fonction f à la calculatrice.

f (68) ≈ 19,6 et f(69) ≈ 20,2 .

69 est le rang de l’année 2019.

Selon ce modèle, en 2019, la part des femmes dans l’ensemble des maires dépassera 20 %.

80

On a f (x) = x3 – 30x2 + 300x , où x est la quan-tité de GPS produits, variant de 0 à 20, en mil-lier, et f (x) est le coût de production variable, en millier d’euros.

1. f ’ (x) = 3x2 – 60x + 300

Δ = 0 ; une solution : x = 10 .

b) a = 3, positif.

La dérivée est toujours positive ou nulle :

f ’ (x) 0 sur [ 0 ; 20 ] .

c) Signe de la dérivée et variation

x 0f ’ (x) + f (x)

0

20

2000

Cf

D

4 6 8 10 12 14 16 18 20

400

600

800

1000

1200

1400

1600

1800

2000

0 2

200

x

y

2. Chaque GPS est vendu 84 €. Un millier de GPS est vendu 84 milliers d’euros. D’où la recette : g (x) = 84x .

b) Bénéfice nul pour la vente de 0, 12 ou 18 mil-liers de GPS.

3. f (x) = g (x)

⇔ x3 – 30x2 + 300x – 84x = 0

⇔ x ( x2 – 30x +216) = 0

x = 0 ou x2 – 30x +216 = 0

Δ = 36 ; deux solutions : 12 et 18.

S = { 0 ; 12 ; 18 } .

81

Fonction d’offre :f(x) = 40x + 55 .

Fonction de demande :g (x) = x3 – 12x2 – 60x + 1000 ,

avec x le prix au kilo variant de 2 à 10 € et f (x) et g (x) , les quantités en tonne.

Page 67: Math 1ere STMG

Chapitre 4 étude de fonCtions 67

1. a) En B2 =40*A2+55

b) Fonction d’offre : fonction affine croissante sur [ 0 ; 10 ] .

c) Fonction de demande décroissante sur [ 0 ; 10 ] .

2. a) À 5 € le kilo, 255 tonnes sont offertes et 525 demandées.

La demande n’est pas satisfaite.

b) À 8 € le kilo, l’offre est de 375 tonnes et la demande de 264 tonnes.

Chiffre d’affaires espéré :

8 × 264 000 = 2 112 000 €.

c) Prix d’équilibre : 7 € le kilo.

Quantité d’équilibre : 335 tonnes.

Chiffre d’affaires à l’équilibre : 2 345 000 €.

3. a) g ’ (x) = 3x2 – 24x – 60

b) On résout 3x2 – 24x – 60 = 0 .

Δ = 1296 ; deux solutions : – 2 et 10 .

Signe de 3x2 – 24x – 60 sur R :

x –2 103x2 – 24x – 60 + 0 – 0 +

c) Signe de la dérivée et sens de variation :

x 2g’ (x) – g (x) 840

10

200

d) Cette étude confirme la réponse à la ques-tion 1. c).

Page 68: Math 1ere STMG
Page 69: Math 1ere STMG

Chapitre 5 probabilités 69

Chapitre 5 Probabilités

Intentions des auteuresLes élèves ont abordé les probabilités depuis la Seconde. Bien que cette partie ne soit pas explicitement indiquée dans le programme, nous avons voulu faire un lien entre les proba-bilités vues en Seconde et le chapitre 1 sur les proportions. Dans le cadre d’un enseignement en spirale, la partie 1 permet de réactualiser les connaissances des chapitres 1 et 3. En Terminale, cela permettra de mieux comprendre les proba-bilités conditionnelles. Cela permet de reprendre la notion d’événement contraire. Il n’est pas utile de s’attarder sur le premier paragraphe du cours.Nous avons choisi de couper les probabilités en deux parties, en gardant pour la seconde partie (dans le chapitre 7) la loi binomiale (calcul à la calculatrice) et la prise de décision dans l’échantillonnage. Dans ce chapitre, la lecture de l’arbre du schéma de Bernoulli est privilégiée. Nous avons introduit la notion de « principe multi-plicatif » que l’on pourra réutiliser en Terminale sur les arbres pondérés.Bien que le programme indique P( X < k ), nous avons plutôt mis en valeur P( X k ) qui est la probabilité donnée par les calculatrices. Nous avons choisi de noter P la loi de probabi-lité (P majuscule droit) pour ne pas confondre avec le nombre p , probabilité de succès dans la loi binomiale . Les QCM Avant de commencer permettent de revenir sur les connaissances de Seconde sur les probabilités, en utilisant, aux QCM 2, 3 et 4 ce qui a été vu aux chapitres 1 et 3. Dans le QCM 5, on aborde le schéma de Bernoulli.

Avant de commencer

Réponses fausses aux QCM1. b) 2. c) 3. b) 4. c) 5. c)

Cours

Probabilité d'événementCette partie est un rappel de Seconde. On peut aussi revoir la page 200.La situation proposée a été choisie pour faire travailler «  au moins  », «  moins de  » … et le vocabulaire qui sera utile sur les variables aléa-toires, sans le dire !

Réponses de l’Étude de situation p. 1041. a) On obtient le nombre total de clients visités suivant le nombre de clients visités par jour. b) En E3, on obtient 78 , soit 78 clients vus durant les 26 jours où Alexandre a vu 3 clients par jour.c) Formule =somme(B3:G3)/H22. a) P(B) = 5/80 = 0,0625P(C) = 1 – P(B) = 1 – 0,0625 = 0,9375b) C est l’événement contraire de B.3. a) « Moins de 4 » signifie « 0 1 2 ou 3 ».b) Pour P(D) , insister sur la somme des pro-babilités des événements élémentaires qui composent D :

P(D) = P(0) + P(1) +P(2) + P(3) = 0,725 .c) Le contraire de D est « au moins 4 clients » ou « plus de 3 clients ».4. a) S  :  »nombre de clients supérieur à la moyenne de 2,7 » ; soit « 3 4 ou 5 clients » ou encore « au moins 3 clients ».Donc D et S peuvent se réaliser en même temps : D ∩ S = « exactement 3 clients » = A .

P( D ∩ S ) = P(A) = 0,325b) D ∪ S = « moins de 4 clients ou au moins 3 clients  » donne toutes les possibilités. Donc P(D) = 1 .Mise en pratique p .105Cela permet de reprendre des connaissances des chapitres 3 (diagramme en boîte) et 1 (proportions sur les sous-populations).

1

Page 70: Math 1ere STMG

70 Chapitre 5 probabilités

Corrigés des Exercices d’application p.105 1

a) Événement A U B  : «  la facture est celle d’une TPE ou son montant est entre 100 et 300 € ». Et P(A U B) = 0,625 (dans l’exo résolu).b) Événement D ∩ A  : «  la facture est celle d’une TPE et son montant est entre 500 et 1 000 € ». Voir cellule D2.

P(D ∩ A) = 750

4000 = 0,1875

2

a) P(B ∩ A ) = 1004000 = 0,025 (utiliser la cel-

lule B3)

P(D U A) = + −2000 2400 750

4000 = 0,9125

b) D ∩ A  : « la facture est celle d’une PME et son montant est entre 500 et 1 000 € ». Voir en cellule D3.

P(D ∩ A ) = 12504000

= 0,3125

P(B U A ) = + −800 1600 100

4000 = 0,575

Schéma de BernoulliLa situation est simple, nous avons présenté les tirages successifs comme étant indépen-dants (vu le programme). La pratique de cette notion en classe de 1re ES nous conduit à insis-ter sur l’aspect « expériences toutes identiques, de même probabilité de succès et d’échecs » plutôt que la notion d’indépendance que les élèves confondent avec l’incompatibilité.

Nous avons préféré ne pas utiliser le tirage de «  boules  » dans les urnes, trop théorique et parfois sujet à rires dans la classe.

Pour parler d’une liste de résultats, nous avons choisi la notation des chemins employée dans la théorie des graphes (voir spécialité de Term ES), un arbre de probabilité étant un graphe particulier. Cela allège les notations.

Nous avons parfois omis le mot «  exacte-ment »  : il est sous-entendu dans tous les cas où on ne précise pas « au moins », « au plus », « moins de », « plus de »…

Réponses de l’Étude de situation p. 1061. a) C’est le principe multiplicatif. On peut revenir sur la page 14 du chapitre 1, et le pro-duit des proportions. Parmi tous les tirages de 3 factures, 40 pour 100 conduisent à une première facture où il faut envoyer une invi-tation, puis parmi ces tirages, 60 pour 100 conduisent au tirage d’une seconde facture sans envoi, puis parmi ces tirages 40 pour 100 conduisent à une 3e facture avec invita-tion à envoyer.

Au total : 0,4 × 0,6 × 0,4 = 0,096 .

Au vu de nos observations en ES , pour cer-tains élèves de cette série, cela restera un « truc ».

b) Faire comprendre que l’on obtient la même probabilité, comme dans le cas des proportions.La liste S-S-E a la même probabilité.

2. a) E-E-E : les trois factures ont un montant inférieur ou égal à 625 €.

P( E-E-E ) = 0,6 × 0,6 × 0,6 = 0,216

b) Le contraire de E-E-E  : « Kevin envoie au moins une invitation »; de probabilité :

1 – 0,216 = 0,784

Mise en pratique p. 107Utilisation d’un arbre à 4 niveaux, correspon-dant à chacune des 4 hôtesses. La résolution présente une rédaction pos-sible pour justifier que la situation est un schéma de Bernoulli.

Corrigé de l’Exercice d’application p. 107

3 a) Les listes permettant de gagner un seul bon de 625 € sont : E-E-E-S , E-E-S-E , E-S-E-E et S-E-E-E .

b) Probabilité de l’événement  : «  le client obtient exactement un bon sur les 4 enve-loppes tirées » :

4 × 0,02 × 0,983 ≈ 0,0753 .

c) P( E-E-E-E ) = 0,984 ≈ 0,9223 et P( S-S-S-S ) = 0,024 ≈ 1,6 ×10–7 .Ces deux événements ne sont pas contraires. On peut vérifier que la somme n’est pas égale à 1 . C’est une erreur classique des élèves.

2

Page 71: Math 1ere STMG

Chapitre 5 probabilités 71

Variable aléatoire X En STMG, c’est la seule variable qui sera étu-diée. Nous avons construit ce chapitre pour que ce paragraphe puisse être étudié en même temps que la page 154 du chapitre 7 si on le désire. On introduit les notations { X = k } et { X k } pour les événements et P( X = k ) et P( X k ) pour les probabilités de ces événements. On peut introduire { X < k } , événement « obte-nir moins de k succès », mais pour obtenir la probabilité sur les calculatrices, il faut calculer P ( X k – 1 } .

Réponses de l’Étude de situation p. 1081. a) X prend les valeurs 0 , 1 , 2 ou 3 .b) Il faut faire la somme des probabilités des 3 listes. Donc 3 × 0,144 = 0,432 .On introduit la notation P ( X = 1 ) .2. a) X 1, pour les listes : E-E-E , E-E-S , E-S-E et S-E-E .b) On effectue la somme de P( X = 0 ) et P (X = 1 ) . Or P( X = 0 ) = P ( E-E-E ) = 0,63 = 0,216 .On introduit la notation P ( X 1 ) .P( X 1 ) = P ( X = 0 ) + P ( X = 1 ) = 0,216 + 0,432 = 0,648 .3. a) E-S-S donne X = 2 , car il y a deux succès dans cette liste.S-E-S et S-S-E donnent aussi X = 2 .b) X = 0 correspond à E-E-E (déjà vu). { X > 0 } correspond à { X 1 } c’est-à-dire « au moins 1 succès » , contraire de « aucun succès ».Mise en pratique p. 109L’exercice présente les questions «  clas-siques » sur ce type de situation. On en pro-fite pour revoir la lecture sur calculatrice des nombres décimaux p. 190.Corrigés des Exercices d’application p. 109

4 P( X = 1 ) = 4 × 0,02 × 0,983 ≈ 0,0753 qui représente la probabilité de l’événe-ment : « le client obtient exactement un bon sur les 4 enveloppes tirées ».

5 {X 1} = {X = 0} U {X = 1}= { e-e-e-e ; e-e-e-s ; e-e-s-e ; e-s-e-e ; s-e-e-e }

D’où P (X 1 ) = 0,984 + 4 × 0,02 × 0,983 ≈ 0,9977 .Ainsi P(X > 1) = 1 – P (X 1 ) ≈ 0,0023 .

Atelier TICE

Nombre probable d'interventions

L’objectif est d’interpréter un diagramme en bâtons et d’utiliser les notations des probabi-lités P ( X = k ) et P( X k ) . C’est aussi une reprise du chapitre 3 pour la moyenne et du chapitre 1 pour les sous-popu-lations.1. a) Durant 31 jours, il y a eu exactement 3 interventions. b) Au total, 100 jours étudiés.2. a) En I2 =somme(B2:H2) b) La formule permet de calculer le nombre d’interventions total.En J3 =I3/I2 . La moyenne est m = 2,1 inter-ventions par jour.

3. a) P ( X = 2 ) = 31/100 = 0,31b) En B5 =B2/$I2 on doit « bloquer » la colonne I. Comme la recopie est sur une ligne, il est inu-tile de bloquer la ligne 2. Mais on peut le faire et entrer =B2/$I$2 .c) On cherche P( X < 2,1 ) , donc on calcule P( X = k ) pour k = 0 , 1 ou 2 .(15 + 18 + 21) /100 = 0,64On écrit en B6 =B5 , puis en C6 =B6+C5 que l’on recopie vers la droite jusqu’en H6 .d) Comme la probabilité d’avoir au plus 4 inter-ventions par jour est de 0,95 , on peut conseil-ler d’avoir 4 techniciens par jour pouvant intervenir : ils couvriront 95 % des cas.

Tirage aléatoire de probabilité donnée

L’objectif est de simuler N épreuves de Ber-noulli de probabilité p donnée et d’étudier la variabilité.

3

1

2

Page 72: Math 1ere STMG

72 Chapitre 5 probabilités

L’étude sur tableur est assez simple à réaliser. Il faut bien insister sur l’interprétation de la for-mule Ent( ALEA() + 0,8 ) ou plus généralement Ent (ALEA() + p ) .Edition 01  : lire «  20  % des valeurs sont dans [ 0,8 ; 1 [ » au lieu de « [ 0 ; 0,8 [ ».Ainsi, le résultat 1 correspond à un administré satisfait et 0 à un administré non satisfait. On calcule alors la fréquence de satisfaits dans les 200 administrés testés, en effectuant la somme des résultats (donc la somme des 1) divisée par le nombre de simulations.

A Sur tableura) C3 =somme(B3;B203)/200b) Dans cette simulation, 74,5 % des adminis-trés testés sont satisfaits.c) Il est possible d’obtenir 0,9 , soit 90  % de satisfaits.Sans critère, difficile de conforter l’affirmation du maire. C’est le but du chapitre 7.

B Sur calculatriceL’algorithme doit être expliqué aux élèves. On peut écrire le programme sur une calcula-trice et le transférer aux autres (voir le chapitre 2 p. 45).Sur TI on trouve :

Prompt Disp dans

EffListe dans

For et End dans

moyenne dans

Suite des écrans que l’on doit avoir pour écrire

ent(NbrAléat+P) :

Sur Casio :

? et dans

ClrList dans catalog

For Next dans Sélectionner COM par F1

puis F6

Meandanscatalog

Int Ran puis F6

Sélectionner PROB

Sélectionner RAND

Page 73: Math 1ere STMG

Chapitre 5 probabilités 73

Les avis de 4 habitantsOn simule 4 épreuves de Bernoulli, la valeur de X est la somme des résultats 0 ou 1 de cha-cune des 4 épreuves de Bernoulli simulées par Ent( ALEA() + p ) .Le programme avec calculatrice est plus déli-cat à réaliser avec les élèves. L’intérêt est l’utili-sation de la fonction NB.SI du tableur.

1. a) Ent(ALEA() + $B$1) a pour résultat 1 avec une probabilité égale au contenu de la cellule B1 (probabilité qu’un habitant soit satisfait), ou pour résultat 0 avec une probabilité de 1 – B1 .Comme on ajoute 4 fois cette fonction, on obtient 0 , 1 , 2 , 3 ou 4 , nombre d’habitants satisfaits parmi les 4 habitants testés.

b) =NB.SI($B3:$B102;D1) donne le nombre de fois où on rencontre le résultat D1 (c’est-à-dire 0) dans les cellules de B2 à B102 . On obtient ainsi le nombre de simulations parmi les 100 simulations faites, donnant X = 0 comme résultat, c’est-à-dire aucun habitant satisfait parmi les quatre. On obtient la fré-quence observée dans la simulation pour { X = 4 } en calculant en H3 =H2/100 .

2. À l’aide d’un arbre, on trouve :

P( X = 4 ) = 0,8 4 ≈ 0,4096 .C’est à comparer à la fréquence observée !

Corrigés des exercices

à l’oral

6

P(A) = 0,17 et P(B) = 0,38Faire travailler sur « au moins », « moins de », « plus de », « au plus ».

7 P(C) = 0,97 et P(D) = 0,83

9 1. Deux billets gagnants : probabilité de 0,25 .2. Un billet gagnant : probabilité de 0,5 .3. Aucun billet gagnant : probabilité de 0,25 .

10

1. Lancer un dé et nommer succès l’évènement S : « obtenir 1 » est une épreuve de Bernoulli.

Lancer trois dés et s’intéresser au nombre de succès S est la répétition de trois épreuves de Bernoulli identiques et indépendantes  : c’est

un schéma de Bernoulli et P(S) = 16

.

2. a) p = 16

  et q = 56

.

b) S-S-S

c) E-S-E : cette liste décrit l’évènement : « une face différente de 1 sur le premier dé, égale à 1 sur le deuxième dé et différente de 1 sur le troisième ».

P(E-S-E) = ⎛⎝⎜⎜⎞⎠⎟⎟⎟ ×

56

16

2

≈ 0,1157 .

d) Trois listes avec deux succès :

S-S-E ; S-E-S et E-S-S .

11

1. Prospection auprès d’un client :

Succès S : « le client est intéressé » ;

Échec E : « le client n’est pas intéressé » :

p = P(S) = 0,2 et q = P(E) = 0,8

Ceci décrit une épreuve de Bernoulli.

Cette épreuve de Bernoulli est répétée trois fois.

a) P(S-S-E ) = 0,22 × 0,8 = 0,032

b) P(E-S-E) = 0,2 × 0,82 = 0,128

c) P(S-S-S) = 0,23 = 0,008

12

a) X ∈ { 0 ; 1 ; 2 ; 3 } .

b) Trois listes contiennent deux succès.

c) {X = 1} = { S-E-E ; E-S-E ; E-E-S }

d) {X = 3} est l’évènement réalisé par la liste S-S-S .P(X = 3) = 0,23 = 0,008

e) Le contraire de l’événement {X = 0} est l’évé-nement {X 1} qui signifie : « obtenir au moins un succès ».

2

Page 74: Math 1ere STMG

74 Chapitre 5 probabilités

Pour s’entraîner

Probabilité d’événement14

1. P(F) = 0,4 ; P(C) = 0,48 et P( )F = 0,6 .

2. C ∩ F : « la personne est chef d’entreprise et est une femme »

P(C ∩ F ) = 0,12

C ∪ F : « la personne est chef d’entreprise ou est une femme »

P(C ∪ F) = 0,48 + 0,4 – 0,12 = 0,7615

1. Gains possibles en euro – 3 ; 2 ; 7 ; 97 .

2. a) Loi de probabilité

gain en € – 3 2 7 97

probabilité 0,69 0,2 0,1 0,01

b) La probabilité de ne pas perdre en jouant une seule fois est 0,31 .

c) La probabilité de gagner au moins 5 € est 0,11 .

16

a) Deux défauts : probabilité 0,16 .

b) Aucun défaut : probabilité 0,44 .

c) Au moins un défaut :

probabilité 1 – 0,44 = 0,56 .17

a) P(A) = 0,129 , P(B) = 0,18 et P(C) = 0,106

b) P(V) = P( A ∩ B ) = 0,02

P(D) = P( B ∪ C ) = 0,18 + 0,106 – 0,025

= 0,261

c) P( A ∩ B ∩ C ) = 0,005

et P( A ∪ B ∪ C ) = 0,361 .19

a) Nombre de codes possibles : 94 = 6 561 .

b) Probabilité de taper le bon code : 16 561

.20

Tirage sans remise parmi les cœurs.

Probabilité d’obtenir la dame : 18 = 0,125 .

Puis probabilité d’obtenir le roi : 17 .

Tirage avec remise parmi les cœurs.

La probabilité d’obtenir la dame, puis la proba-

bilité d’obtenir le roi sont chacune égales à :

18

= 0,125 .21

1. a) En C4 =E4-(B4+D4)

En C2 =C4-C3

b) En E2 =B2+C2+D2

2. N : « oranges navels »

et M : « oranges venant du Maroc ».

P(N U M) = 1 3502 000 = 0,675

23

2 4 6 8 10 12

1 3 5 7 9 11 13

1 3 5 7 9 11 13

1 3 5 7 9 11 13

3 5 7 9 11 13 15

3 5 7 9 11 13 15

5 7 9 11 13 15 17

1. P(A) = 1

36  ; P(B) = 1

12  ; P(C) = 49 .

2. a) P(E) = 0 ; P(F) = 1 .

b) E est un évènement impossible et F est un événement certain.

24

1. P(A) = 0,5 ; P(B) = 0,86 ; P(C) = 0,5

et P(D) = 0,27 .

2. a) B ∪ C  : «  le CA est inférieur à 600 k€ ou supérieur ou égal à 200 k€ »

D ∩ C = D« Le CA est d’au moins 400 k€. »

b) P(B U C) = 1 et P(D ∩ C) = 0,27

1

Page 75: Math 1ere STMG

Chapitre 5 probabilités 75

Schéma de Bernoulli25

1. Probabilité d’obtenir un autre numéro que 1 :

1 – 0,25 = 0,75 .

2. a) Épreuve de Bernoulli : lancer du dé tétraé-drique, l’évènement succès S « obtenir 1 » tel que : p = P(S) = 0,25 .

On obtient un schéma de Bernoulli en répé-tant deux fois cette épreuve de Bernoulli.

b) P(S-S) = 0,252 = 0,0625

c) La probabilité d’obtenir au moins une fois le n°1 est : 1 – 0,752 = 0,4375 .26

1. a) P(S) = p = 0,1

b) E : « Marvin a obtenu un entier autre que 5 »

P(E) = q = 0,9

c) Loi de probabilité

succès S échec E

probabilité 0,1 0,9

2. a) Arbre pondéré

E

E

S E S

0,9 0,1

0,10,90,10,9

S

b) Les listes obtenues : E-E  ; E-S ; S-E ; S-S .

3. a) P(A) = 0,12 = 0,01

b) B est réalisé par les listes : S-E et E-S .

P(B) = 2 × 0,1 × 0,9 = 0,1827

a) Choisir une carte dans un jeu de 32 cartes et nommer succès l’événement S : « obtenir un As » est une épreuve de Bernoulli de paramètre

p = 4

32 =

18 .

Comme on remet la carte dans le jeu, on répète deux fois la même expérience. Donc on a répé-tition de 2 expériences identiques et indépen-dantes.

b) Arbre pondéré

E

E

S E S

18

S18

18

78

78

78

A : « obtenir au moins un succès »

P(A) = 1 – ⎛⎝⎜⎜⎜⎞⎠⎟⎟⎟⎟

78

2

≈ 0,234

29 a) Si on ne remet pas le jeton dans la boîte, et que l’on en retire un deuxième, ce n’est pas un schéma de Bernoulli car on répète deux expériences qui ne sont pas identiques.

b) P(S) = 3

20 = 0,15 et P(E) = 0,85 .

E

E

S E S

0,85 0,15

0,150,850,150,85

S

A : « obtenir au moins un succès »P(A) = 1 – 0,852 = 0,277530 1. L’appel d’un client conduit à deux

issues :• le succès S : « la demande est traitée en moins de deux minutes », de probabilité p = 0,3 • l’échec E, l’événement contraire de S.Cette situation est une épreuve de Bernoulli .

succès S échec E

probabilité 0,3 0,7

2. Arbre pondéré

3. P(A) = 0,33 = 0,02731

a) L’événement B est réalisé par les listes  : E-E-S ; E-S-E ; S-E-E .

2

Page 76: Math 1ere STMG

76 Chapitre 5 probabilités

b) P(B) = 3 × 0,72 × 0,3 = 0,44133

1. a) Arbre pondéré

b) Listes de résultats possibles :

E-E-E ; E-E-N ; E-N-E ; E-N-N ; N-E-E ;

N-E-N ; N-N-E ; N-N-N .

2. a) P(A) = 3 × 0,64 × 0,362 ≈ 0,249 .

p(B) = 3 × 0,642 × 0,36 ≈ 0,442 .

b) P(F) = 0,363 ≈ 0,047 .

c) P(G) = 1 – P(F) = 1 – 0,363 ≈ 0,953 .

34

1. P(F) = 0,4 et P(D) = 0,6 .

2. Arbre pondéré

3. P(A) = 0,6

P(B) = 3 × 0,6 × 0,42 = 0,288

P(C) = 1 – 0,43 = 0,936

35

Choisir une carte dans un jeu de 32 cartes et nommer succès l’événement S  : « obtenir un As » est une épreuve de Bernoulli de paramètre

p = 4

32 =

18

.

Comme on remet à chaque fois la carte tirée dans le jeu, on répète trois fois la même

épreuve de Bernoulli. On reconnaît alors un schéma de Bernoulli de paramètres n = 3

et p = 18 .

2. a) P(S-S-S) = ⎛⎝⎜⎜⎜⎞⎠⎟⎟⎟⎟

18

3

≈ 0,002

b) Listes contenant deux succès :

E-S-S ; S-E-S ; S-S-E .

Probabilité d’obtenir deux succès :

3 × ⎛⎝⎜⎜⎜⎞⎠⎟⎟⎟⎟ ×

18

78

2

≈ 0,041 .

3. Si on ne remet pas la carte tirée dans le jeu, il ne s’agit plus d’un schéma de Bernoulli, car les expériences répétées ne sont plus identiques.

Variable aléatoire X

36

a) P( X = 1) = 0,25

b) { X 2 } est l’évènement : « le robot vérifié a au moins deux défauts ».

P( X 2 ) = 0,31

37

1. P(S) = 0,16 ; P(E) = 0,84

2. P(A) = 0,844 ≈ 0,4979

P(B) = 0,164 ≈ 0,0007

b) Six listes réalisent { X = 2 } .

P( X = 2 ) = 6 × 0,162 × 0,842 ≈ 0,1084

c) P(C) = P( X = 2 ) ≈ 0,1084

3. P( X = 1 ) = 4 × 0,16 × 0,843 ≈ 0,3793

Ce résultat est la probabilité qu’une seule balle sur les quatre lancées atteigne la zone rouge.

38

1. Prendre au hasard un pyjama dans l’échan-tillon, et observer s’il a un défaut, correspond à une épreuve de Bernoulli dont le succès est l’événement S  : «  le pyjama a un défaut » de probabilité P(S) = p = 0,1 .

Répéter 4 fois cette épreuve de Bernoulli se modélise par le schéma de Bernoulli de para-mètres n = 4 et p = 0,1.

Voir la représentation par un arbre identique à celui de l’exercice 37.

3

Page 77: Math 1ere STMG

Chapitre 5 probabilités 77

2. a) P( X = 0 ) = 0,94 = 0,6561

b) P( X = 1 ) = 4 × 0,1 × 0,93 = 0,2916

P( X 1 ) = 0,6561 + 0,2916 = 0,9477

c) P( X 1 ) = 1 – P( X = 0 ) = 0,3439

La probabilité pour qu’au moins un des pyja-mas pris au hasard parmi les quatre ait un défaut est égale à 0,3439 .

39

1. a) { X = 5 } : « Fanny réussit les 5 parties de la série »

{ X = 0 } : « Fanny ne réussit aucune partie »

{ X < 3 } : « Fanny réussit moins de 3 parties »

b) P( X = 5 ) = 0,65 = 0,07776

P( X = 0 ) = 0,45 = 0,01024

2. P( X < 3 ) = P( X 2 ) = 0,31744

P( X < 4 ) = P( X 3 ) = 0,66304

40

À partir de l’arbre analogue à celui de l’exercice 37, on lit les 16 listes de résultats possibles :

F-F-F-F ; V-F-F-F ; F-V-F-F ; F-F-V-F ; F-F-F-V ; V-V-F-F ; V-F-V-F ; V-F-F-V ; F-F-V-V ; F-V-F-V ; F-V-V-F ; V-V-V-F ; V-V-F-V ; V-F-V-V ; F-V-V-V ; V-V-V-V .

2. a) Chaque liste a la même probabilité de se réaliser : 0,54 = 0,0625 .

Quatre listes réalisent l’événement : « une seule des 4 réponses est correcte », d’où la probabi-lité de cet évènement : 4 × 0,0625 = 0,25 .

b) P( V-V-V-V ) = 0,0625

3. a) X prend les valeurs 0 ; 1 ; 2 ; 3 et 4 .

b) Loi de probabilité

k 0 1 2 3 4

P( X = k ) 0,0625 0,25 0,375 0,25 0,0625

c) P( X < 3 ) = 0,6875 .

La probabilité qu’un candidat ait moins de trois réponses correctes sur les quatre est 0,6875 .

4. Probabilité pour qu’un candidat soit apte :

P( X 3 ) = 0,25 + 0,0625 = 0,3125 .

Pour approfondir

41 1. Les colonnes des catégories d’âge donnent la répartition en pourcentage suivant le statut.b) Parmi les 15-24 ans qui ont un emploi, 98,1 % sont salariés.

2. a) En D2 =D4-D3 b) Tableau de répartition

c) p = 0,839 .

3. a) en C7 =C2*C$9/100 b) Tableau des effectifs

4. A : « la personne a entre 15 et 24 ans »B : « la personne est salariée »A : « la personne a plus de 24 ans »

P( A ) = 1 – P(A) = 1 – 2 07025 691

≈ 0,9194

b) A ∩ B : « la personne a entre 15 et 24 ans et est salariée »A ∪ B : « la personne a entre 15 et 24 ans ou est salariée »

P( A ∩ B ) = 2 031

25 691 ≈ 0,0791 

et P( A ∪ B ) = 22 91525 691 ≈ 0,8919

43

1.Tableau des effectifs

T non T total

E 38 82 120

non E 22 258 280

total 60 340 400

Page 78: Math 1ere STMG

78 Chapitre 5 probabilités

2. P(A) = P( E ∩ T ) = 38

400 = 0,095

P(B) = P( E ∪ T ) = 142400 = 0,355

La probabilité de l’évènement  : «  le salarié choisi a un enfant de moins de 3 ans et projette d’en avoir un dans les 5 ans » est 0,095 .

La probabilité de l’évènement  : «  le salarié choisi a un enfant de moins de 3 ans ou pro-jette d’en avoir un dans les 5 ans » est 0,355 .

3. On représente la situation à l’aide d’un schéma de Bernoulli.

Événement succès, S  : «  le salarié interrogé a répondu oui à la question sur le projet d’avoir un enfant dans les 5 ans » avec P(S) = 0,3 .

a) On représente la situation par un arbre pon-déré à 4 niveaux avec :

p = 0,3 et q = 0,7 .

Arbre pondéré

b) { X = 2 } est l’évènement :  « parmi les quatre salariés interrogés, deux ont répondu oui » .

P( X = 2 ) = 6 ×0,32 × 0,72 = 0,2646

4. a) { X < 2 }  : « moins de deux salariés ont répondu oui »

P( X < 2 ) = P( X = 0 ) + P( X = 1 ) = 0,6517

b) Comme P( X < 2 ) > 0,5 , le projet ne sera pas poursuivi.

44

Partie A

1. En B6 =SOMME(B2 : B4)

En B7 =B6/$E$6

Partie B

a) P(S) =0,36

b) Soit X le nombre de succès S lors du tirage au hasard de 5 bulletins de l’urne.

P( X 1 ) = 1 – P( X = 0 ) = 1 – 0,64 5 ≈ 0,8926

45

1. Probabilité de gagner exactement 2 € avec un seul ticket : p = 0,1 .

2. a) Acheter un ticket et nommer succès l’évé-nement S : «  gagner exactement 2€ » est une épreuve de Bernoulli.

Acheter trois tickets est la répétition de ces trois épreuves, cela correspond à un schéma de Bernoulli, où la probabilité du succès est P(S) = p = 0,1 .

b) Liste S-S-S avec P(S-S-S) = 0,001 .

3. Théo achète 10 tickets. Soit X le nombre de billets rapportant exactement 2 €.

P(A) = P( X 1 ) = 1 – P( X = 0 )

= 1 – 0,910 ≈ 0,6513

P(B) = P( X = 10 ) = 0,110 = 10–10

Vers le Bac

47 1. D’après l’énoncé, la catégorie B représente 56 % de l’ensemble des motards et 35 % des motards possédant une moto de la catégorie B possèdent une moto routière.

D’où P( B ∩ R ) = 0,35 × 0,56 = 0,196 .

On peut établir le tableau suivant :

A B total

R 17 % 19,6 % 36,6 %

S 27 % 36,4 % 63,4 %

total 44 % 56 % 100 %

2. P( S ∩ A ) = 0,27

3. La situation correspond à un schéma de Ber-noulli de paramètres n = 3 et p = P(B) = 0,56 .

Probabilité que l’un au moins d’entre eux soit de la catégorie B : 1 – 0,443 ≈ 0,915 .

Page 79: Math 1ere STMG

Chapitre 5 probabilités 79

48 Partie A

D’après la symétrie de l’histogramme du Lycée Pasteur, on peut associer :

Lycée Pasteur et boîte 1 Lycée Carnot et boîte 2 .Partie B

Lycée Carnot :

P(A) = 120320 = 0,375 et P(B) =

120320

= 0,375 .

Lycée Pasteur :

P(A) = 200280

≈ 0,714 et P(B) = 20

280 ≈ 0,071 .

Partie C

a) Effectif total N = 30 .

Me = +55 652 = 60 et Q1 = 20 ; Q3 = 105 .

Q1 Q320 30 40 50 60 70 80 90 100 110 1200 10

b) P(S) = 6

30 = 0,2

Partie D

a) La situation correspond à un schéma de Ber-noulli de paramètres n = 3 et p = 0,2 .

c) E  : «  au moins deux élèves parmi les trois interrogés ont un temps de trajet inférieur à 20 minutes »

Calculer P(E) revient à calculer la probabilité d’obtenir au moins deux succès.

D’où P(E) = P( X = 2 ) + P( X = 3 )

= 3 × 0,22 × 0,8 + 0,23 = 0,104 .

d) P( X = 0 ) = 0,83 = 0,512

P( X 1 ) = 1 – P( X = 0 ) = 0,488

L’événement  : «  au moins un des élèves interrogé a un temps de trajet inférieur à 20 minutes » est l’événement contraire de l’évé-nement  : «  aucun des élèves interrogés a un temps de trajet inférieur à 20 minutes ».

49 1. a) En B4 =B2+B3

b) En B5 =B2/B4

En F5, on obtient la proportion de femmes dans le club sportif.

2. a) P(A) = 0,4 et P(B) = 150500

= 0,3 .

b) P( A ∩ B ) = 60

500 = 0,12

P( A ∪ B ) = 0,4 + 0,3 – 0,12 = 0,58

3. Arbre pondéré

H

H

F H F

0,6 0,4

0,40,60,6 0,4

F

On calcule la probabilité d’obtenir les listes H-F et F-H.

On obtient : 2 ×0,4 × 0,6 = 0,48 .

50 1. a) Arbre pondéré

Page 80: Math 1ere STMG

80 Chapitre 5 probabilités

b) Toutes les branches sont pondérées par la même probabilité égale à 0,5 .

D’où la probabilité de chacune des huit listes :

0,53 = 0,125 .2. a) G = { – 6 ; 0 ; 6 ; 12 } .

b) Loi de probabilité

gain – 6 0 6 12

probabilité 0,125 0,375 0,375 0,125

c) Probabilité de perdre :

0,125 + 0,375 = 0,5 .

52 1. a) Le pourcentage de cette catégorie passe de 25,9  % à 31,3  % , soit une hausse de 5,4 points de pourcentage.

b) CM = 31, 325, 9 ≈ 1,2084  : augmentation de

20,8 % .

c) Si le taux se maintient, taux prévisible en 2015 : 31,3 × 1,208 ≈ 37,8 % .2. a) Le taux de chômage a diminué de 2005 à 2010 pour les diplômés de l’enseignement supérieur sortis depuis au moins 5 ans de for-mation initiale.

Catégorie B  : de 6 % à 5 %  ; une baisse d’un point de pourcentage ou de 16,7 % .

Catégorie C : de 4,4 % à 4,1 % ; une baisse de 0,3 point de pourcentage ou de 6,8 % .3. On considère l’ensemble des diplômés de l’enseignement supérieur de la catégorie B. On interroge au hasard une personne de cet ensemble.On note S  : «  le diplômé est au chômage  » et E : « le diplômé n’est pas au chômage » .On a P(S) = 0,05 , d’où P(E) = 0,95 .

4. a) La situation correspond à un schéma de Bernoulli représenté par l’arbre ci-dessous avec p = 0,05 et q = 0,95 .

Arbre pondéré

b) Probabilité qu’au moins un des diplômés soit au chômage : 1 – 0,954 ≈ 0,185 .

53 1.a) De 2000 à 2005 : CM =

8, 96, 5

≈ 1,369 , soit une hausse de 37 % .

De 2005 à 2010 : CM = 10, 558, 9

≈ 1,1854 , soit une hausse de 18,5 % .

b) Affirmation fausse  : les mises ont moins augmenté sur la période 2005-2010 que sur la période 2000-2005.

c) Chiffre d’affaires prévisible en 2015 :

10,55 × 1,185 ≈ 12,5 milliards d’euros.2. a) Chiffre d’affaires des jeux de grattage en

2009  : 4, 2901,106

≈ 3,87884 , soit 3 879 millions

d’euros à 1 million près.

b) 1,538 × 0,375 = 0,57675

En 2010, un montant de 576,75 millions est revenu à l’État sur les mises fait au Loto.

3. La situation correspond à un schéma de Ber-noulli de paramètres n = 5 et p = 0,0052 .

Probabilité de gagner moins de 10 € : 0,9948 .

Probabilité que Magali gagne au moins une fois 10 € en jouant 5 semaines de suite :

1 – 0,99485 ≈ 0,02573 .

54 1. a) En B3 =$C$1/4

En F3 =SOMME(C3:E3)

b) En E4 =$B3/4

En E10 =$B9/4

2. À la fin du jeu, s’ils jouent toujours de façon aléatoire, il reste 3,81 € à Philippe et Régis.

55 a) P( X = 0 ) =

44380 ≈ 0,116

Page 81: Math 1ere STMG

Chapitre 5 probabilités 81

P( X = 1 ) =

60380 ≈ 0,158

P( X = 2 ) = 118380 ≈ 0,311

b) P(X 2) = P( X = 0 ) + P( X = 1 )

+ P( X = 2 ) = 222380

≈ 0,584

c) P( X > 3 ) = 1 – P( X 3 )

= 1 – 299380 ≈ 0,213

56 1. Magasin A : x = 5 et σ ≈ 3,21 .

Magasin B : x = 5 et σ ≈ 2,26 .

Même nombre moyen de jours d’attente dans les deux magasins, mais dispersion du nombre de jours d’attente plus importante dans le magasin A.

2. a) Tableau de valeurs

b) P(E) = 0,44 et P(F) = 1 – 0,63 = 0,37 .

3. a) Schéma de Bernoulli représenté par un arbre à 3 niveaux.

Événement succès S  : «  le client du SAV du magasin A attend moins d’une semaine » donc on a P(S) = 0,63 .

Probabilité de l’événement échec : 0,37 .

b) La situation décrite pour Omar, Fred et Yann revient à chercher la probabilité de la liste :

échec-succès-échec.P(échec-succès-échec) = 0,372 ×0,63 ≈ 0,086

c) Probabilité que l’un au moins des trois clients attende une semaine et plus :

1 – 0,633 ≈ 0,75 .

Page 82: Math 1ere STMG
Page 83: Math 1ere STMG

Chapitre 6 suites numériques 83

Chapitre 6 Suites numériques

Intentions des auteuresBien que cette notion soit indiquée en premier sur le programme, nous avons préféré ne trai-ter cette notion qu’en fin d’année, notre pra-tique en classe nous ayant montré la difficulté de cette notion.Par rapport au programme précédent, il y a un grand changement : les suites ne sont vues que pour leur mode de génération, en particulier la formule de récurrence, avec l’aide des TICE.Nous avons donc utilisé au maximum le tableur, très facile à comprendre. Cela devrait aider les élèves de cette série.L’algorithmique prend sa place dans ce cha-pitre dans les ateliers TICE en particulier, et de nombreux exercices. Mais rien n’empêche de commencer ce chapitre bien avant (parties 1 et 2 ) dans un enseignement en spirale.Les QCM Avant de commencer permettent d’introduire la notation indicielle en revenant sur le rang (comme dans le chapitre 3) dans les QCM 1 à 3.Le QCM 4 revient sur le chapitre 1 et les taux d’évolution. Et le QCM 5 permet d’aborder un algorithme simple utilisant la calculatrice.

Avant de commencer

Réponses fausses aux QCM1. c) 2. b) 3. c) 4. b) 5. b)

Cours1 Modes de génération d'une suite

La difficulté pour les élèves est de faire la diffé-rence entre le rang et le terme.Nous avons choisi «  terme initial  » au lieu de « premier terme », car difficile pour des élèves d’avoir un premier terme qui n’est pas de rang n = 1 . Nous avons réintroduit « premier terme »

dans les exercices d’approfondissement, lorsque la notion a déjà bien été travaillée.Nous espérons que, ayant rencontré au cha-pitre 3 des données rangées, cela aidera les élèves. Nous avons donc parlé pour une suite de « liste de nombres », plus abordable que la notion de fonction.La graduation de l’axe donnée dans la défini-tion est volontairement non régulière ; il serait bien de présenter aussi des suites non mono-tones, ce que l’on a dans la situation proposée.La situation peut paraître « compliquée », mais nous espérons qu’elle parle aux élèves et qu’ils voudront faire les calculs (possibles à la main).

Réponses de l’Étude de situation p. 1281. a) b) En D3 =D2+C3-B3 et c)

2. a) En D9 = D8 – 40 + 5b) En août de l’année suivante, au rang 20, il ne restera que 10 €, donc pas de quoi payer le téléphone.On peut proposer d’autres évolutions, en actualisant les valeurs des frais de téléphone en particulier.Mise en pratique p. 129Le calcul de termes d’une suite sur tableur a été vu en page 128.On se propose d’utiliser la calculatrice pour deux suites, l’une définie comme suite asso-ciée à une fonction (restriction d’une fonc-tion f pour les valeurs entières de la variable) et l’autre définie par un procédé algorith-mique utilisant la touche Ans ou Rep de la calculatrice.

Page 84: Math 1ere STMG

84 Chapitre 6 suites numériques

Le second jeu est repris dans l’Atelier TICE 2, réalisé à l’aide d’un programme sur calcula-trice.Corrigés des Exercices d’application p. 129

1 a) b) d) Récurrence.c) Explicite.

2

a) b) c) d)

u1 21 147 3,5 10

u2 23 217.5 3 10

3 a) 10 ; 15 ; 25 ; 45b) 5 ; 5 ; 5 ; 5 : suite constantec) 2 ; – 1 ; – 7 ; – 19 ; – 43

2 Représentation et sens de variationLa représentation des suites à l’aide d’un tableur est vue en Atelier 1 p. 136. De nom-breuses représentations ont été vues dans les chapitres 2, 3 et 4.Notre pratique en classe nous a montré que la représentation d’une suite définie par récur-rence sur calculatrice demande trop d’inves-tissement en temps pour un piètre résultat auprès des élèves.C’est pourquoi nous avons privilégié la repré-sentation sur tableur :• suites définies par une formule explicite dans le cours pour faire plus facilement le lien avec les fonctions ;• suites par formule de récurrence dans l’exer-cice résolu.

Réponses de l’Étude de situation p. 1301. a) b0 = 250 , soit 250 € le jour de l’ouver-ture et 500 € le 5e jour.b) a10 = 840 et b10 = 1250 ; 840 < 1250 .2. b) En B2 =20*A2^2-240*A2+12403. a) La recette a augmentée dans le maga-sin Best.b) À partir du 6e jour, la recette du magasin Best dépasse celle du magasin Aless.Mise en pratique p. 131Comparaison de trois suites de même relation de récurrence, mais pas de même terme initial.

Corrigés des Exercices d’application p. 1314

a) Si Ulysse a 40 € au départ, ses économies augmentent :u0 = 40 ; u1 = 42 ; u2 = 43,6 ; u3 = 44,8 .b) Si Victor a 60 € au départ, ses économies diminuent :u0 = 60 ; u1 = 58 ; u2 = 56,4 ; u3 = 55,12 .

5 À long terme, on peut penser que, pour cha-cun d’eux, les économies se stabilisent à 50 € .

3 Suites arithmétiquesNous avons insisté sur des éléments mnémo-techniques pour ce type de suites, c’est pour-quoi nous avons préféré la lettre a à la lettre r pour la raison : addition, graduation régulière, points alignés, lien avec la fonction affine.Il est important de faire le lien entre arithmé-tique et variation absolue constante.

Réponses de l’Étude de situation p. 1321. a) Table des valeurs

On peut placer la raison dans une cellule pour permettre de changer.b) un+1 = un + 3000c) En B3 =B2+3000d) 10 dépôts sont nécessaires.2. On peut entrer une nouvelle suite comme indiqué ci-dessus.En C3 =C2-4Il faut 7 mois pour former tous les manuten-tionnaires.Mise en pratique p. 133On trouve dans les premiers exercices d’en-traînement des situations de base.

Page 85: Math 1ere STMG

Chapitre 6 suites numériques 85

L’étude d’un placement à intérêts simples est une bonne application des suites arithmé-tiques ; il faut cependant bien décomposer la modélisation, car il faut faire comprendre que les intérêts sont calculés sur le capital de départ.La recherche de seuil est vue ici par lecture simple du tableau de valeurs.Un programme peut-être réalisé sur la calcu-latrice, comme à l’exercice 64 p. 148.

Corrigé de l’Exercice d’application p. 1336

a) C0 = 4 000 € . Intérêts annuels : 200 € .b) Capital acquis au bout de 3 ans :

C3 = 4 600 € ,soit 4 000 +200 + 200 + 200 .

4 Suites géométriquesNous avons pris appui sur la notion de coeffi-cient multiplicateur pour introduire les suites géométriques et ainsi reprendre le chapitre 1, et insister sur le lien entre variation relative constante et suite géométrique. La modélisa-tion se fait simplement ; la raison est :

q = CM = +1100

t .

Réponses de l’Étude de situation p. 1341. a) Faux. Erreur classique.b) Par une phrase : diminuer de 20 %, c’est-à-dire multiplier par 0,8 .c) un+1 = un – 20/100 × un = 0,8 × un .d) En B3 =B2 – 20/100*B2ou = B2*0,8

e) Tableau des résultats

Pour la rédaction, demander toujours de donner les deux valeurs permettant de conclure (voir l’exercice résolu p. 135).Pour plus de rigueur, il faut préciser le sens de variation de la suite.En n = 8, u8 = 8 388,6 < 10 000 ,alors que u7 = 10 486 > 10 000.Donc en 2018.2. La production est augmentée de 50  % donc :

CM = 1 + 0,5 = 1,5 .vn+1 = vn + 0,5 vn = 1,5 vn .En n = 8 , v8 = 5 126 > 5 000alors que v7 = 3 417 < 5 000 .Mise en pratique p. 135Les premiers exercices d’entraînement en p. 145 sont à traiter après le cours : de l’exer-cice 47 à 54.La mise en pratique sur le placement à inté-rêts composés est plus délicat.La recherche d’un seuil n’est vu qu’en exer-cice 60, quand les calculs de termes d’une suite géométrique ont été travaillés.Corrigés des Exercices d’application p. 135

8 Au départ, K0 = 5 000 .a) Intérêts acquis la 1re année : 250 € .b) K1 = 5 250  et K2 = 5 512,5 .Relation K2 = K1 + 0,05 K1 = 1,05 × K1 .c) Kn + 1 = Kn + 0,05 Kn = 1,05 × Kn .

Page 86: Math 1ere STMG

86 Chapitre 6 suites numériques

d) À l’aide du tableur actualisé :

K18 ≈ 12  033. Ainsi, à 18 ans, Pauline peut acheter la voiture à 9 000 € .10

a) CM = 119500100000

= 1,195 = q

La variation relative est de 19,5 % .Donc Pn + 1 = 1,195 × Pn .b) À l’aide du tableur (ou en programmant la calculatrice), on obtient :

P14 ≈ 203 926 .203 926 habitants au 1er juillet 2014.

Atelier TICE

1 Coût limiteL’objectif est d’utiliser une formule de récur-rence donnée, établir la feuille de calcul, repré-senter la suite et déterminer un seuil.Cet atelier est complet. Il peut paraître difficile à certains élèves, bien que tout soit donné. Nous conseillons de le faire réaliser par binôme en salle informatique.Pour les meilleurs binômes, on peut terminer en demandant de calculer la somme des coûts (suite donnant le coût total en fonction de la quantité) et de représenter ce coût total fonc-tion de la quantité produite.1. En B3 =0,85*B2+60 2. a) Représentationb) À partir de 24 tonnes, la dernière tonne pro-duite coûte 409,5 €, donc moins de 410 € .

3. L’instruction SI est importante et très facile à comprendre par les élèves, plus facile que la programmation d’une calculatrice.a) En C20, la réponse est « 19 » et en C24, la réponse est « 23 ».Le seuil de stabilité est 24 tonnes.b) Nouvelle formule en C2 :

=SI(B2<430;»seuil»;A2)Le nouveau seuil est de 17 ans.

2 Gain à la n-ième partieLa situation est celle de la page 129.Nous avons choisi de donner cet algorithme pour le calcul d’un terme d’une suite quel-conque. Pour une autre suite, il suffit de chan-ger la formule de récurrence.Pour changer dans un programme  : il faut faire attention, car on ne peut pas revenir en arrière. Souvent les programmes des élèves ne tournent pas parce qu’ils ont effacé une ins-truction.Sur TI : entrer dans le programme :

EDIT

Se placer sur l’instruction à changer :• ce que l’on tape remplace ce qui est écrit où se situe le curseur• supprime le caractère (ou l’instruction) où se situe le curseur ;• supprime la ligne complète ;

• permet d’insérer ce que l’on

tape au clavier (le curseur n’est plus qu’un trait horizontal en bas) .Si on se trompe d’instruction, mieux vaut vali-der dans un premier temps, puis supprimer ensuite.Pour quitter le programme, et ainsi l’enregis-trer : .

Sur Casio :

Se placer sur le nom du programme à changer et EDIT par F2 .

Page 87: Math 1ere STMG

Chapitre 6 suites numériques 87

Se placer sur l’instruction à changer :• ce que l’on tape s’insère à la position du curseur.• efface le caractère AVANT le curseur.

• permet d’insérer les caractères que l’on tape au niveau du curseur.Pour quitter le programme et ainsi l’enregis-trer : .

a) Le cache remplace V * 2 – 21 .b) On utilise le programme en entrant 20 pour V0 = 20, et N = 2 : donc V2 = 17 et V6 = – 43 .Pour exécuter un programme :

sur Casio : (voir ci-dessus).Sélectionner le programme par les flèches, puis EXE par F1 .

Entrer les valeurs 20 , puis 2 par

Cliquer deux fois sur pour sélectionner de nouveau le programme et le relancer.Sur TI :

Sélectionner et lancer le programme en tapant le numéro du programme (ici ).

Taper 20 puis 6 et valider par .

Relancer le programme par pour une autre valeur de N.

3 Suite et seuilUne application importante des suites est la recherche du rang d’un terme pour atteindre, dépasser, ou devenir inférieur à une valeur donnée. Ce rang est appelé « seuil ».1. a) La suite est arithmétique de raison – 203,5 négative donc ; la suite est donc décroissante.b) Sous le cache : U - 203.5 → ULa valeur test est A = 0 .

Le seuil est N = 7 , c’est-à-dire u7 négatif.2. a) La suite est géométrique ; comme la rai-son est 0,9, cette suite est décroissante.b) Nouveau programme

Pour accéder aux relations de test : sur TI :

sur Casio :

F6F3

Page 88: Math 1ere STMG

88 Chapitre 6 suites numériques

Et sélectionner la relation par la touche F

4 Achat à créditIl est souhaitable que les élèves fassent eux-mêmes cet atelier devant l’ordinateur.

On pourra leur demander de rendre le fichier tableur contenant les trois propositions sur 3 feuilles de calcul différentes.

1. Les intérêts le premier mois représentent 1,5 % de 2 000 €, soit 30 € .

Le capital en fin de mois est augmenté de 1,5 % , donc il est multiplié par :

CM = 1 +1,5/100 = 1,015 .

Et le capital restant dû après le premier verse-ment de 100 € est :

C3 = 2000*1,015 – 100 = 1 930 .

2. À la fin du 13e mois, il reste 1  003,42 € . Somme des versements : 2 203,42 € .

3. À la fin du 23e mois, le capital restant dû devient inférieur à 100 € .

Donc il y a 23 versements de 100 € et un verse-ment final de 94,24 € .

Somme des versements : 2 394,24 € .

4. Il y a 28 versements de 100 € et un dernier de 28,14 € . Somme : 2 528,14 € .

On peut demander de calculer le pourcentage que représente le coût du crédit. On peut aussi, sur tableur, demander le taux annuel pour un taux mensuel de 1,5  % et rechercher le taux d’usure officiel à la date de l’ouvrage.

Corrigés des exercices

à l’oral

11

1. 5 ; 16 ; 38 ; 82 .

2. 100 ; 90 ; 81 ; 72,9 .

3. 200 ; 200 ; 200 ; 200 .12

On a : u(n) = – n2 + 8n + 10 .

1. u(0) = 10 .

2. u(1) = 17 et u(2) = 22 .

3. u(6) = 22 et u(10) = – 10 .13

a) À partir de u0 = 1 , on passe d’un terme au suivant en multipliant par 2 et en ajoutant 1 .

b) u3 = 15 .14

La figure 2 montrant une courbe ne représente pas une suite. En effet, dans un repère, la repré-sentation d’une suite est donnée par une suc-cession de points dont les abscisses sont des entiers naturels.

Fig 1 : représentation d’une suite croissante.

Fig 3 : représentation d’une suite décroissante.

Page 89: Math 1ere STMG

Chapitre 6 suites numériques 89

Fig 4 : représentation d’une suite ni croissante, ni décroissante.

On peut revenir sur cet exercice pour deman-der la nature des suites, et pourquoi pas, don-ner une idée de la raison.15

1. On reconnaît la suite arithmétique de pre-mier terme u0 = 125 et de raison 12.

2. u1 = 137 et u2 = 149 .

3. La suite ( un ) est croissante.16

a) Salaire en mars 2012 : 1 375 € .

b) On modélise par la suite ( un ) définie par u0 = 1 300 et un + 1 = un + 25 .

un représente le salaire n mois après décembre 2011.

c) La suite des salaires est arithmétique, de pre-mier terme u0 = 1 300 et de raison a = 25 .17

a) En C1 - 1200 en C3 8000

b) En B4 =B3 + 1

c) En C4 =C3+$C$1

18

On a u0 = 8000 et un + 1 = 0,75 × un .

a) On passe d’un terme au suivant en multi-pliant toujours par le même nombre 0,75.

b) u1 = 6 000 ; u2 = 4500 ; u4 = 2 531,25 .

c) La suite ( un ) semble décroissante.

19

On a w0 = 15 et wn + 1 = 1,4 × wn .

a) On passe d’un terme au suivant en multi-pliant toujours par le même nombre 1,4 .

b) w1 = 21 ; w2 = 29,4 ; w4 = 57,624 .

c) La suite semble croissante.20

On a toujours u0 = 100 .a) Suite arithmétique de raison – 3b) On a un + 1 = 1,3 × un , suite géométrique de raison 1,3 .c) Ni l’un ni l’autre.

d) On peut écrire un + 1 = 0,7 × un , suite géo-métrique de raison 0,7 .

Pour s’entraîner

1 Modes de génération d’une suite21

a) u8 = 5 ; u25 = 0 ; u38= 0 .

b) Rang du premier chiffre 1 : n = 19 .Rang du premier chiffre 5 : n = 5 .Rang du premier chiffre 0 : n = 2 .c) On a un = 9 pour n = 14 , 26 et 34 .

d) u37 = 3 et u38 = 0 .22

1. a) u1 = 35 ; u2 = 45 et u10 =125 .b) un = 10n + 25 .2. 10n + 25 = 205 ⇔ n = 18 .

À 18 ans, Teddy recevra 205 € .

23

1. Résultats

entrée 0 1 2résultat 2,5 7 11,5

Page 90: Math 1ere STMG

90 Chapitre 6 suites numériques

2. a) u0 = 2,5 ; u1 = 7  et u2 = 11,5 .

b) u3 = 16 et u4 = 20,5 .

c) un = × +9 52n . Cette formule permet de

vérifier les résultats précédents.

25

a) un = – 3n + 12 . En B4, on a u2 = 6 .

b) u8 = – 12 et u10 = – 18 .

26

a) vn + 1 = 2 × vn – 50 .

b) v4 = 2 × 130 – 50 = 210 et v5 = 370 .

27

1. a) u3 = 52 481 ; u10 = 43 783

En mars 2011, 52 481 entreprises ont été créées et 43 783 en octobre 2011.b) Le nombre de créations d’entreprises en mai 2012 est représenté par le terme u17 .

2. a) En colonne D, terme initial de rang 1  : v1 = – 5 264 .

b) En D3 =C3-C2

c) La valeur manquante en D8 : v6 = – 3 171 .

La variation absolue a été la plus forte entre août et septembre 2011, avec 10 395 créations d’entreprises.

2 Représentation et sens de variation29

On peut supposer que la suite ( un ) est crois-sante.

30

suite représentation sens de variation

( un ) figure 2 décroissante( vn ) figure 1 croissante( wn ) figure 3 décroissante

31

Pour la suite ( un ), on passe d’un terme au sui-vant en retranchant 10, avec u0 = 100 .

D’où un + 1 = un – 10 .

32

Pour la suite ( vn ), on passe d’un terme au sui-vant en multipliant par 1,5 et en ajoutant 1, en commençant par v0 = 2 .

vn + 1 = 1,5 × vn + 1 .

3 Suites arithmétiques34

1. En A3 =A2+1

Les entiers naturels forment la suite arithmétique de terme initial 0 et de raison 1 .

2. a) En B4, valeur 13 .

b) On passe d’un terme au suivant en ajoutant le même nombre.

La suite ( un ) est arithmétique, commençant en u0 = 10 et de raison 1,5 .

c) un + 1 = un + 1,5 .

u1 = 11,5 ; u2 = 13 et u3 = 14,5 .

35

a) u1 = 52 ; u2 = 56  et u3 = 60 .

b) un + 1 = un + 4 .

c) La suite ( un ) est croissante.

Page 91: Math 1ere STMG

Chapitre 6 suites numériques 91

37

La situation est modélisée par la suite arithmé-tique ( un ), définie pour tout entier naturel non nul par u1 = 5 et un + 1 = un + 3 .

un représente le nombre de poésies que Dunhkann connaît la n-ième semaine de l’année.

u1 = 5 ; u2 = 8 ; u3 = 11 ; u4 = 14 ; u5 = 17 .38

On a u1 = 220 et un + 1 = un – 5 .

a) u2 = 220 – 5 = 215 ; u3 = 210 .

b) « Pour tout entier naturel non nul » signifie que la suite ne commence qu’en n = 1 et non 0 . La formule n’est valable que pour n 1. Le terme initial de la suite est le terme u1 . Ainsi, 220 est le terme de rang 1.

c) u5 = 200 .39

On a S0 = 4 000

1. a) S1 = 4000 – 60 = 3 940 et S2 = 3 880

b) Sn + 1 = Sn – 60 .

La suite ( Sn ) est arithmétique .

2. Au bout d’un an, Morgane dispose de la somme S12 = 3280 € .40

a) Raison de la suite arithmé-tique ( un ) : a = 2

b) En B4 =B3+2

c) En B4 =B3 + (B3-B2)

Formule valable pour toute suite arithmétique de terme initial u0 = 100 et u1 égal au contenu de la cellule B3, quel que soit le nombre mis en B3 .41

1. Algorithme

Entrée : U, A, N (variables numériques)

Pour K variant de 1 à N

faire le calcul U + A

et stocker dans U

Fin de la boucle « Pour »

Sortie : afficher U

2. Pour U = 100, A = 4 et N = 2, la boucle « POUR » tourne deux fois.

b) 100 est le terme de rang N = 0 .

3. Utilisation sur TI Pour N = 2 : 108 .

Et pour N = 6 : 124 .

43

a) Premiers termes représentés

n 1 2 3 4un 4 6 8 10

b) Ces points représentent la suite arithmé-tique de premier terme u1 = 1 et de raison a = 2 .

c) La raison a est le coefficient directeur de la droite qui passe par les points alignés repré-sentant la suite ( un ) .

d) Le terme u8 est l’ordonnée du point de la droite d’abscisse 8 .

u8 = 18 .

2 3 4 5 6 7 8

4

6

8

10

12

14

16

18

20

0 1

2

x

y

44

1. Intérêts annuels : 105 € .

2. a) On a C0 = 3 000  ; C1 = 3 105 ; C2 = 3 210 et C3 = 3 315 .

b) On passe d’un terme au suivant en ajoutant le même nombre 105. On en déduit que la

Page 92: Math 1ere STMG

92 Chapitre 6 suites numériques

suite ( Cn ) est arithmétique avec C0 = 3 000  et la raison a = 105 .

3. L’année 2013 est l’année de rang 5, car  : 2013 = 2008 + 5 et C5 = 3 525 . Celia dispose de 3 525 € le 2 janvier 2013.

46

On note u1 = 2300 le nombre de paniers fabri-qués en janvierun est le n-ième mois de l’année commencée en janvier et le mois de janvier suivant est noté u13 .La suite ( un ) est arithmé-tique de premier terme u1 et de raison a = – 200.On utilise une feuille de calcul pour déterminer le dernier mois où la produc-tion est positive.

En B3, on entre la formule =B2-200 et on recopie vers le bas.La production s’arrêtera en décembre de la même année avec 100 paniers fabriqués.

4 Suites géométriques47

On a u0 = 10 ; q = 10 .1. u1 = 30 ; u2 = 90 ; u3 = 270 .2. un + 1 = 3 × un .3. Suite croissante, car q > 1 .

48 On a u0 = 700 ; q = 0,81. u1 = 560 ; u2 = 448 ; u3 = 358,4 .2. un + 1 = 0,8 × un .2. Suite décroissante, car q ] 0 ; 1 [ .

49 u1 = 7 ; u2 = 14 et u3 = 28 .On en déduit la raison : q = 2 .u4 = 56 ; u5 = 112 .un + 1 = 2 × un .

50 u1 = 4,5 ; u2 = 1,5 et u3 = 0,5 .

2

1

uu =

13

 ; 3

2

uu =

13

.

La suite ( un ) peut être une suite géométrique

de raison q = 13

.

51

1. La suite représentée n’est pas arithmétique car la différence entre deux termes consécutifs n’est pas constante.

2. La représentation donnée est celle des cinq premiers termes de la suite géométrique de terme initial u0 = 16 et de raison q = 1,5 .

52

1. a) CA l’année suivante : 33 000 €

b) On passe du CA d’une année au CA de l’an-née suivante en multipliant par 1,1 .

La suite des CA est géométrique de terme ini-tial u0 = 30 000 et de raison q = 1,1.

2. En B3 =B2*1,1

3. Après 4 ans d’augmentation :

CA = 43 923 € .

53

u0 = 100 , baisse de l’indice de 8 % par an.

a) u1 = 92 ; u2 ≈ 84,6 ; u3 ≈ 77,9 .

b) un + 1 = 0,92 × un

c) La suite ( un ) est géométrique, de premier terme u0 = 100 et de raison q = 0,92 .

54

Page 93: Math 1ere STMG

Chapitre 6 suites numériques 93

Valeur de revente de la voiture 4 ans après l’achat : 3 006 € à 1 € près.55

1. a) b) Algorithme modifié :

2.a) En 2012 : p0 = 43 .

Si l’augmentation de 2,5 % par an se maintient, la population du Kenya en 2014 sera :

p2 = 43× 1,0252 ≈ 45,2 millions .

b) Utilisation du programme :

On retrouve le résultat de la question 2. a) .

c) Utilisation du programme

Population estimée au bout de 10 ans : 55 millions d’habitants.

56

1. a) u1 = 95 et u2 = 90,25 .

b) un + 1 = 0,95 × un

2. Formule que l’on peut saisir en B5 : b) c) ou d) .

3. Résultats

a) u5 ≈ 77,4 .

b) 77,4 – 100 = – 22,6

L’indice a baissé de 22,6 % en 5 mois.

57

1. Salaire mensuel en 2013 : 1 872 € .2. a) u0 = 1 800 ; u2 ≈ 1 947 et u3 ≈ 2 025, à 1 € près.b) un + 1 = 1,04 × un .c) La suite ( un ) est géométrique, de premier terme u0 = 1 800 et de raison q = 1,04 .3. Sur cinq années consécutives, le salaire de Karine est multiplié par :

1,045 ≈ 1,217 ,soit une hausse de 21,7 % .

59 B0 = 30 000 ; C0 = 125 000 .1. a) Hausse de 5 % du chiffre d’affaires.C1 = 131 250 et C2 = 137 812,5 .b) Cn + 1 = 1,05 × Cn .c) La suite ( Cn ) est géométrique, de premier terme C0 = 125 000 et de raison q = 1,05 .2. a) Hausse de 6,5 % du bénéfice.B1 = 31 950 et B2 = 34 026,75 .b) Bn + 1 = 1,065 × Bn .c) La suite ( Bn ) est géométrique, de premier terme B0 = 30 000 et de raison q = 1,065.3. Tableau des résultats

Au bout de 5 ans, le bénéfice représente 26 % du chiffre d’affaires et 27 % au bout de 8 ans (à 1 % près).

60

1. C1 = 1Si le conducteur n’a pas d’accident : C2 = 0,95 .2. a) Cn + 1 = 0,95 × Cn .

Page 94: Math 1ere STMG

94 Chapitre 6 suites numériques

b) La suite ( Cn ) est géométrique, de premier terme C0 = 1 et de raison q = 0,95 .

3. On applique le programme BONUS :

Le conducteur, s’il n’a pas d’accident, diminue de moitié la prime d’assurance au bout de 15 ans.

Pour approfondir

61

1. Les points du graphique sont presque alignés, donc l’évolution de la population mondiale peut être modélisée par une suite arithmétique.

2. De 1960 à 2010, on compte 5 décennies.

( 6 841 – 3 027 ) / 5 ≈ 763 .

La population mondiale a augmenté, à 1 million près, de 763 millions d’habitants par décennie.

3. 6 841 + 763 = 7 604

En 2020, on peut prévoir 7 milliards 604 mil-lions de personnes.62

1. Les points du graphique ne sont pas alignés, on ne peut pas modéliser la population afri-caine par une suite arithmétique.

2. a) En C3 =B3/B2

b) De 1950 à 2000, la population a été multi-pliée par 3,6 (à 0,1 près), Soit une hausse de 260 % en 50 ans.

3. De 1990 à 2020, on modélise par une suite géométrique de raison q = 1,025.a) u1 ≈ 654,7  ; u2 ≈ 671  ; u3 ≈ 687,8 .b) u10 ≈ 817,6 .c) Avec ce modèle, la population de l’Afrique est estimée en 2000 à 817,6 millions d’habi-tants.d) 819,5 – 817,6 = 1,9Le modèle donne un écart de 1,9 millions avec la population réelle connue en 2000.63

1. a) B1 = 1 900 + 400 = 2 300C1 = 1900 × 1,15 = 2185b) Bn + 1 = Bn + 400La suite ( Bn ) est arithmétique, de premier terme B0 = 1 900 et de raison a = 400 .c) Cn + 1 = 1,15 × Cn .La suite ( Cn ) est géométrique, de premier terme C0 = 1 900 et de raison q = 1,15 .2. Tableau de valeurs

a) En B4 =B3+400

b) En C4 =C3*1,153. a) En C9 : 3822 × 1,15 ≈ 4395 .En B10 : 4300 + 400 = 4700 .b) Jusqu’à 5 ventes par mois, la proposition B est la plus intéressante.De 6 à 9 ventes, la proposition C est plus inté-ressante.64

1. a) La suite ( Bn ) est géométrique, de premier terme B0 = 2 000 et de raison q = 1,12 .b) On peut faire fonctionner l’algorithme à la main ou à la calculatrice :

Page 95: Math 1ere STMG

Chapitre 6 suites numériques 95

Utilisation du programme

Cet algorithme permet de déterminer la plus petite valeur de n telle que :

Bn 4 000 .

2. D’après la question 1, le chef d’entreprise verra son bénéfice doubler en 7 ans d’activité.

Pour supprimer un programme :

sur TI : entrer dans la gestion des mémoires

sélectionner le programme

et supprimer le programme

65

1. a) La suite ( un ) est géométrique, de premier terme u0 = 1 700 et de raison q = 1,02 .

b) un + 1 = 1,02 × un

c) u3 ≈ 1804

En 2013, on peut prévoir 1 804 clients.

2. Tableau des résultats

a) En C3 =C2*(1+D$2) ou =C2*1,02 .

b) En 2019, le nombre de clients de l’agence sera supérieur à 2 000.66

1. CM = 488690 ≈ 0,7072

CM×100 – 100 = 29,28Baisse de 29,3 % de la taille du fichier après une compression de niveau 1.

2. Tableau des coefficients multiplicateurs d’un niveau sur l’autre.

a) La suite ( Tn ) est géométrique, de premier terme T0 = 690 et de raison q = 0,707 .b) À l’aide de la formule, on retrouve les valeurs du tableau à une unité près :

T1 = 487,83 ≈ 488et T2 = 344,89581 ≈ 345 .

3. a) En C3 =C2*0,707

Page 96: Math 1ere STMG

96 Chapitre 6 suites numériques

b) Avec une compression au niveau 9, la taille du fichier sera inférieure à 40 ko.

Vers le Bac

68

On revient sur le chapitre 5 de statistiques et le chapitre 1 sur les proportions et l’évolution.

Partie 1

1. Tableau des effectifs

âge 19 22 27 33 41 51 66

eff. 190 300 360 450 400 200 200

Résultats à l’aide de 1-Var stats L1 , L 2

Âge moyen : m ≈ 35,5 ans .

Écart type : α ≈ 13,5 .

2. 8502100

≈ 0,405 ; les 18-30 ans représentent

40,5 % de l’échantillon.

Partie 2

1. a) CM = 2 8792 678

≈ 1,075

De 2008 à 2009, fréquentation du site en hausse de 7,5 % .

b) Sur les années suivantes, la hausse est encore de 7,5 % chaque année.

2. a) Le choix de q = 1,075 est justifié en tant que coefficient multiplicateur de chaque évo-lution annuelle observée précédemment.

u1 = u0 × 1,075 ≈ 3327 × 1,075 ≈ 3577

b) un + 1 = 1,075 × un

c) u4 ≈ 4443

En 2015, prévision de 4  443 connexions par jour en moyenne.

d) u8 ≈ 5933 et u9 ≈ 6379 .

Selon ce modèle, les 6 000 connexions journa-lières seront dépassées en 2020.69

1. a) 1

0

uu

= 320300 ≈ 1,067 .

Et uu

2

1 =

340320

= 1,0625 .

Comme les résultats sont différents, on en déduit que la suite ( un ) n’est pas géométrique.

b) En admettant que la suite ( un ) soit arithmé-tique, la raison est a = 20 .

Relation de récurrence : un + 1 = un + 20 .

c) u6 = 420 . En 2006, la ville X a récupéré 420 tonnes de verre.

2. a) v0 = 250 ; v1 = 275  ; v2 = 302,5 .

b) La suite ( vn ) est géométrique de raison 1,1 . D’où vn + 1 = 1,1 × vn .

c) En D3 = 1,1 × D2

3. À partir de 2005, la collecte dans la ville Y a dépassé la collecte dans la ville X.

4. a) La collecte de la ville X dépasse les 700 tonnes à partir de 2021 avec 720 tonnes.

b) La collecte de la ville Y dépasse les 700 tonnes à partir de 2011 avec 713,3 tonnes.

5. a) Faux : v0 = 250  et v2 = 302,5 , donc ;

CM = 302, 5

250 = 1,21 .

De 2000 à 2002, la quantité de verre récupérée par la ville Y a augmenté de 21 % .

b) Vrai : u0 = 300 et u9 = 480 .

CM = 480300 = 1,6 soit une hausse de 60 % .

Page 97: Math 1ere STMG

Chapitre 6 suites numériques 97

70

Partie1

1. CM = 52 50050 000

= 1,05 .

Soit une augmentation de 5 % de la popula-tion entre 2010 et 2011.2. P2 = 52 500 × 1,05 = 55 125 .3. a) La suite ( Pn ) est géométrique, de premier terme P0 = 50 000 et de raison 1,05 .b) Pn + 1 = 1,05 × Pn

c) P5 ≈ 63  800  ; c’est-à-dire qu’en 2015, on estime que la population de l’agglomération urbaine sera de 63 800 habitants (à 100 près).d) Algorithme :

Début : N et P variables numériques 0 → N 50 000 → PTant que P < 100 000 Faire N + 1 → N 1,05 × P → P

Fin Tant QueSortie : Afficher N

À l’aide de la calculatrice, on utilise cet algo-rithme et on constate que selon ce modèle, la population va doubler en 15 ans.

Partie 2 :1. D’une année à l’autre, la consommation moyenne d’eau par habitant a augmenté de 5 litres par jour.Pour la consommation d’eau, on peut modé-liser par une suite arithmétique de premier terme 295 et de raison 5 .

2. a) En C3 = 1,05 × C2

b) En D3 = D2 + 53. Quantité d’eau nécessaire par jour en 2011 pour la population totale de l’agglomération :

52 500 × 305 = 16 012 500 litres .

4. Les besoins en eau auront doublé en 2021 d’après le graphique ou le tableau de valeurs.

Page 98: Math 1ere STMG
Page 99: Math 1ere STMG

Chapitre 7 loi binomiale – éChantillonnage 99

Chapitre 7 Loi binomiale – Échantillonnage

Intentions des auteuresCe chapitre termine la partie statistiques et probabilités. Cependant, nous conseillons de garder une partie du chapitre 4 sur les fonc-tions, ou des exercices très concrets sur les suites pour terminer l’année scolaire (voir les propositions d’enseignement en spirale en début d’ouvrage).Nous avons souhaité couper en deux parties la loi binomiale :• au chapitre 5, le schéma de Bernoulli et la lecture sur un arbre des probabilités suivant la valeur de la variable aléatoire donnant le nombre de succès ;• au chapitre 7, la loi binomiale calculée à l’aide de la calculatrice et son application à l’échan-tillonnage.La calculatrice est obligatoire pour ce chapitre pour effectuer les calculs de P( X k ) en liste 2 pour toute valeur de la probabilité de succès et la taille n de l’échantillon . Mais nous avons insisté sur le tableur, pour conserver le côté visuel de la recherche de l’intervalle de fluctua-tion, sans trop utiliser l’algorithmique, lourd pour ces élèves, et qui n’est pas une capacité attendue du programme pour ce chapitre.Les instructions Somme (List20,025) et Somme (List2<0,975) donnent les bornes de l’intervalle [ a ; b ] de fluctuation en gardant l’idée de la définition.Pour la prise de décision, il faut penser que la notion d’hypothèse vue ici est bien celle des sciences (physique ou SVT) : en Première, on connaît la proportion p du caractère étu-dié dans la population, et on regarde la fré-quence observée dans l’échantillon pour rejeter ou non l’hypothèse faite sur la popu-lation. En  Terminale, on étudie le contraire, on va rechercher dans quel intervalle on peut «  raisonnablement  » situer la proportion à partir d’une fréquence observée (sondage).Les QCM Avant de commencer permettent de revenir sur le schéma de Bernoulli du chapitre 5,

la notation P(X < k ) et la variable aléatoire X associée au nombre de succès dans une liste.

Avant de commencer

Réponses fausses aux QCM1. c) 2. b) 3. b) 4. c) 5. a)

Cours

Loi binomialeDans le chapitre 5, nous avons utilisé des arbres de niveau 2 puis 3 en exercices pour habituer les élèves et quelques uns de niveau 4. La situation proposée ici s’appuie sur un arbre de niveau 4.La représentation en diagramme en bâtons est vue dans les exercices 26 et 27.

Le professeur peut le faire à partir d’un fichier tableur pour cette situation. Il sera utilisé dans la partie 2.

Réponses de l’Étude de situation p. 1541. a) « Aucun client satisfait » correspond à la liste E-E-E-E de probabilité :

P( X = 0 ) = 0,2 4 = 0,0016 .b) D’après l’arbre, il y a 4 chemins qui conduisent à X = 1 .P( X = 1 ) = 4 × P( S-E-E-E)= 4 × 0,8 × 0,23 = 0,02562. a) Tableau des résultats (loi de probabilité) :

k 0 1 2 3 4P( X =

k )0,0016 0,0256 0,1536 0,4096 0,4096

Faire vérifier à la calculatrice que la somme des probabilités est bien 1.b) P ( X 2 ) = 0,1808= P( X = 0 ) + P( X = 1) + P( X = 2 ) .Ou bien 1 – (P( X = 3 ) + P( X = 4 )) .

1

Page 100: Math 1ere STMG

100 Chapitre 7 loi binomiale – éChantillonnage

3. Tableaux de valeurs à la calculatrice

La moyenne est 3,2 = 4 × 0,8 .Ne pas hésiter à prendre d’autres valeurs de p (y compris des fractions) et même d’autres valeurs de n (3 ou 5 pour que l’on puisse encore faire l’arbre et compter les chemins) pour vérifier que la moyenne est toujours le produit n × p , et ainsi mettre en place l’espé-rance de la loi binomiale.Mise en pratique p. 155On passe au cas où la taille de l’échantillon n est plus grande. Pour reconnaître une situa-tion de loi binomiale, insister sur le fait que l’on répète toujours la même expérience, donc des expériences répétées identiques (ne pas trop insister sur l’indépendance, qui ne sera pas revue en Terminale, et que les élèves confondent trop souvent avec l’in-compatibilité).Sans calculatrice, on ne peut faire des exer-cices qu’avec le tableur ou en consultant des tables de valeurs p. 175.Résultats sur tableur :

Diagramme en bâtons de la loi binomiale B ( 20 ; 0,8 ) .

Corrigés des Exercices d’application p. 1551

Utilisation de la calculatrice pour d’autres valeurs de k .

a) P(X = 16) ≈ 0,218 2b) P(X = 20) ≈ 0,011 5

2 Diagramme en bâtons dans le cas où 95 % des clients sont satisfaits :

Page 101: Math 1ere STMG

Chapitre 7 loi binomiale – éChantillonnage 101

a) P(X = 10) ≈ 10 – 8 ≈ 0b) P(X = 15) ≈ 0,002 2

2 Échantillonnage et intervalle de fluctuation à 95 %

C’est la partie la plus délicate du chapitre. Bien faire comprendre que l’on connaît la probabi-lité p de succès pour la totalité de la popula-tion, et que l’on regarde la fréquence de succès sur un échantillon de taille n , ici n = 100 .La figure proposée ci-dessous peut faire com-prendre que, pour tout échantillon de taille 100, on n’aura pas forcément la même fré-quence observée f , et que l’on a une « fluctua-tion » du résultat autour de la valeur p .

Le but est de mesurer cette fluctuation en éta-blissant un intervalle contenant 95 % des fré-quences que l’on peut observer si on répète un grand nombre de fois l’expérience (tirage de 100 individus dans la population).Si nécessaire, on peut aussi donner comme exemple un grand sac contenant un très grand nombre de billes (des milliers) de deux couleurs (rouge et bleue) avec 80  % billes rouges et 20 % de bleues, et on tire au hasard 100 billes. On compte combien il y a de billes rouges (nombre de succès) dans un échantillon tiré. On obtient alors une fréquence observée. Mais si on recommence un très grand nombre de fois ce tirage de 100 billes, on peut obtenir de 0 à 100 billes rouges ! Cependant certaines valeurs de X sont plus probables que d’autres.On se pose alors la question : dans quel inter-valle le nombre de billes rouges dans l’échan-tillon tiré va-t-il se situer, dans 95 % des très nombreux tirages que l’on peut effectuer ?

On n’exclut pas 5 % des valeurs de k, mais 5 % des valeurs de la variable aléatoire X, pondé-rées par leurs probabilités.

Réponses de l’Étude de situation p. 1561. Les probabilités sont très faibles (proches de 0). Faire calculer à la calculatrice quelques valeurs.2. Expliquer « sous certaines conditions ».a) Faire lire la table et faire le lien avec le graphique de la partie À retenir sur le dia-gramme en boîte sur tableur.

P( X 72 ) ≈ 0,034 2 d’après le tableau de valeurs, donc bien supérieur à 0,025 alors que P( X 71 ) ≈ 0,020 0 < 0,025 .b) Par lecture de la table :

P( X 87 ) ≈ 0,974 7 < 0,975et P(X 88) ≈ 0,987 4 0,975.

Donc le plus petit entier est b = 88 tel que :P( X b ) 0,975 .

On peut se contenter de > 0,975, cela ne changera pas l’intervalle [ a ; b ] .L’idée est que l’intervalle fermé contient bien 95 % des valeurs de X pondérées par leurs probabilités.D’où [ a ; b ] = [ 72 ; 88 ] .Ainsi, dans 95  % des tirages que l’on peut faire, le nombre de clients satisfaits, observé dans cet échantillon de 100 clients testés, sera entre 72 et 88 clients.c) L’intervalle de fluctuation à environ 95 % de la fréquence est [ 0,72 ; 0,88 ] pour une probabilité p = 0,8 .On peut expliquer aux élèves que si l’on veut un intervalle de fluctuation à 99 % , il faut laisser 0,5 % avant l’intervalle et 0,5 % après, donc trouver le plus petit entier a tel que :

P( X a ) > 0,005 ,et le plus petit entier b tel que :

P( X b ) 0,995 .

Page 102: Math 1ere STMG

102 Chapitre 7 loi binomiale – éChantillonnage

Mise en pratique p. 157On propose un échantillon de taille 25. La taille est petite pour montrer aux élèves que faire une étude sur 25 personnes ne signifie pas grand-chose vu l’étendue de l’intervalle de fluctuation que l’on obtient.C’est aussi ce qui se passe pour la réussite au Bac où on attend 80 % de réussite dans une classe de 25 élèves… Il n’est pas « anormal » de trouver un taux de réussite de 65 % seu-lement !L’exercice 3 permet de pratiquer la calcula-trice.

Corrigés de l’Exercice d’application p. 157

3 a) Dans les instructions de la calculatrice, on remplace 25 par 100 . On doit retrouver ce qui a été fait sur tableur.

D’où a = 72  et b = 88 .b) On en déduit l’intervalle de fluctuation à environ 95 % : [ 0,72 ; 0,88 ] .c) Instructions sur calculatrice :

D’où a = 110 et b = 129 .Intervalle de fluctuation à environ 95 % :

⎣⎢⎢110150

 ; ⎤

⎦⎥⎥

129150

≈ [ 0,733 ; 0,86 ] .

Prendre des bornes [ c ; d ] qui englobent l’intervalle : c a / n et d b / n .

3 Prise de décisionLa situation propose une autre façon de lire l’intervalle [ a ; b ] , à partir de la représentation graphique. On peut aussi visualiser les points sur tableur ou sur le logiciel TI-Nspir à l’aide du Player.

Réaliser le diagramme pour P( X k )

Page 103: Math 1ere STMG

Chapitre 7 loi binomiale – éChantillonnage 103

On peut faire une mise en forme de l’axe des ordonnées, en graduation secondaire de 0,025 en 0,025 pour visualiser les deux droites hori-zontales d’équations y = 0,025 et y = 0,975 .

Réponses de l’Étude de situation p. 158

a) La fréquence observée est 1220

= 0,6 .

b) On lit le plus petit entier a tel que :P( X a ) > 0,025 : donc on regarde le pre-mier point situé au-dessus de la droite verte d’équation y = 0,025 : a = 12 .On lit le plus petit entier b tel que :

P( X b ) 0,975 ,donc on regarde le premier point situé au dessus de la droite violette d’équation y = 0,975 : b = 19 .Donc [ a ; b ] = [ 12 ; 19 ] et l’intervalle de fluctuation à environ 95 % de la fréquence de cette loi binomiale est :

[ 0,6 ; 0,95 ] .c) La fréquence observée est dans cet inter-valle, donc il est tout à fait probable d’avoir une fréquence de 0,6 alors que la proportion réelle est de 0,8 .Il faut faire comprendre aux élèves que l’on ne peut pas conclure par « on accepte l’hypothèse », mais par « on ne rejette pas l’hypothèse ».En revanche, si la fréquence observée n’est pas dans l’intervalle, « on rejette l’hypothèse, mais avec un risque d’erreur de 5 % ».

Mise en pratique p. 159Nous avons proposé une situation où la fréquence observée est la même, mais la prise de décision change suivant la taille de l’échantillon.On peut vérifier les intervalles proposés (en particulier si on a un programme sur la cal-culatrice).

Corrigés des Exercices d’application p. 159

4

a) On a 0,476 = 500a

et 0,564 = 500

b .

D’où a = 238 et b = 282 .b) Selon cet institut, avec 275 voix sur 500, le candidat X obtiendrait 55  % des voix,

ce qui reste dans la fourchette précédente, car selon l’hypothèse initiale, sur un très grand nombre d’échantillons de 500 votants, dans 95  % des cas, le candidat X obtient entre 47,6 % et 56,4 % des voix. Donc le jour-naliste ne peut pas dire que le candidat a pris 3 points de plus.

DocumentDerniers sondages le 19 avril 2012, avant les élections du 22 avril 2012.

1er TOUR• 28  % pour François Hollande selon la moyenne des dernières études des huit instituts de sondages  : 27  % (TNS-SOFRES et LH2), 27,5  % (Harris Interactive et Opi-nionWay), 28 % (Ifop et CSA), 29 % (Ipsos) et 30 % (BVA).• 26,4 % pour Nicolas Sarkozy : 25 % (CSA), 25,5  % (Ipsos), 26,5  % (LH2, BVA et Harris Interactive), 27 % (Ifop et TNS-Sofres), 27,5 % (OpinionWay).• 15,75 % pour Marine Le Pen : 14 % (BVA), 15,5 % (Ifop et LH2), 16 % (CSA, Ipsos, Opi-nionWay et Harris Interactive) et 17 % (TNS-Sofres).• 13,75 % pour Jean-Luc Mélenchon : 12 % (Harris Interactive), 13 % (TNS-Sofres et Opi-nionWay), 14 % (BVA et Ipsos), 14,5 % (Ifop et CSA) et 15 % (LH2).• 10,1 % pour François Bayrou : 9,5 % (Ifop), 10 % (Ipsos, TNS-Sofres, BVA OpinionWay et LH2), 10,5  % (CSA) et 11  % (Harris Interac-tive).• De 2 à 3 % pour Eva Joly dans l’ensemble des études.• De 1 à 2 % pour Nicolas Dupont-Aignan.• De 1 à 2 % pour Philippe Poutou.• Au plus 1 % pour Nathalie Arthaud.• Au plus 0,5 % pour Jacques Cheminade.

2e TOUR• 55,7 % pour François Hollande : 54 % (Harris Interactive), 55 % (TNS-Sofres, OpinionWay), 55,5  % (Ifop), 56  % (Ipsos et LH2) et 57  % (BVA et CSA).• 44,3 % pour Nicolas Sarkozy : 43 % (BVA et CSA), 44 % (Ipsos et LH2), 44,5% (Ifop), 45 % (OpinionWay, TNS-Sofres) et 46 % (Harris).

Page 104: Math 1ere STMG

104 Chapitre 7 loi binomiale – éChantillonnage

Atelier TICE

1 Forfait téléphoneIl est souhaitable que ce TD se fasse à deux élèves par poste, en salle informatique. Mais on peut faire faire ce TD individuellement en devoir à la maison, avec rendu par ENT.On peut aussi faire la partie A en classe entière avec vidéo-projecteur, le professeur projetant la correction faite ou la réalisant devant la classe, les parties B et C étant à faire à la mai-son. On trouvera ci-joint la copie des tables de valeurs que l’on peut donner aux élèves.On pourra ainsi voir que, plus la taille de l’échantillon augmente, plus l’étendue de l’in-tervalle diminue.

A. Cas n = 101. a) Formule b) Table de valeursc) Diagramme en bâton d) Formule2. Interprétation  : peut se faire sans tableur, juste sur calculatrice.a) La probabilité que Léna, après 10 appels, n’ait aucun abonné acceptant le nouveau for-fait est 0,278 50 (voir écran p. 160). En B14, on trouve :

0,1210 ≈ 0,619×10–10 ≈ 0 .

b) En C5, on va trouver la probabilité que, parmi les 10 appels de Léna, au plus un abonné accepte le nouveau forfait.On peut faire le calcul par la somme des deux cellules B4+B5 .

B. Cas n = 50 .a) En cellule A2 , on saisit n = 50 .b) La plus grande probabilité est pour X = 6, avec P( X = 6 ) ≈ 0,1712 .

c) On trouve a = 2 et b = 11 .

C. Cas n = 100a) Nuage de points d’abscisse k et d’ordonnées la probabilité P(X k ) :On sélectionne la plage A4:A30 pour les abs-cisses et la plage C4:C30 pour les ordonnées.

Page 105: Math 1ere STMG

Chapitre 7 loi binomiale – éChantillonnage 105

On lit a = 6 et b = 19 .

Entre 6 et 19 abonnés sur 100 acceptent le changement de forfait, dans environ 95 % des échantillons de taille 100.

2 Nombre de forfaits espéré

On cherche, en moyenne, combien d’abon-nés vont changer de forfait sur 10 abonnés contactés.

L’atelier est à faire faire aux élèves en utilisant le fichier prêt.

En feuille 1  : la simulation des 25 lots de 10  abonnés et le calcul de la moyenne  : le nombre d’abonnés changeant de forfait parmi un lot de 10 est en ligne 2 (en jaune). Les 25 lots sont mis en colonne.

En feuille 2  : la simulation de 100 lots de 10  abonnés (en colonne C) et le calcul de la moyenne de ces 100 lots en cellule E3.

On demande aux élèves d’effectuer un grand nombre de simulations. On doit trouver des nombres qui se situent dans un intervalle cen-tré en 1,2 , mais qui montrent la variabilité (sur 10 abonnés, elle est très grande).

On peut faire faire la moyenne des moyennes obtenues, on trouvera un nombre proche de 1,2 .

3 Testé "scientifiquement"

On trouve souvent des publicités indiquant qu’un produit a été testé… sur 25 personnes ! Le but de cet atelier est de montrer qu’obtenir 60  % de résultats «  positifs  » ne signifie pas grand-chose sur la valeur du produit. On peut aller plus loin en faisant l’hypothèse d’une proportion p = 0,45 et voir qu’obtenir 15 per-sonnes satisfaites sur 25 ne conduit pas à reje-ter cette hypothèse.

Pour la partie A, le professeur peut utiliser directement le fichier fourni sur le CD (égale-ment en téléchargement sur www.hachette-livre.fr).

Pour la partie B , à la calculatrice, utiliser le script du livre numérique ou les instructions indiquées dans le manuel élève.

A. Lecture sur tableur

1. Si n = 100 , on lit [ a ; b ] = [ 50 ; 69 ] .

L’hypothèse p = 0,6 n’est pas rejetée.

Visualisation du nuage de points en feuille1 :

2. Si n = 50 , on lit [ a ; b ] = [ 23 ; 37 ] .

Page 106: Math 1ere STMG

106 Chapitre 7 loi binomiale – éChantillonnage

D’où l’intervalle de fluctuation à environ 95 % de la fréquence : [ 0,46 ; 0,74 ] .

Avec une fréquence observée de 4050

= 0,8 ,

l’hypothèse p = 0,8 est à rejeter, avec un risque d’erreur de 5 % .

3. Si n = 25 , on lit [ a ; b ] = [ 10 ; 20 ] .

D’où l’intervalle de fluctuation à environ 95 % de la fréquence : [ 0,4 ; 0,8 ] .

On aurait pu trouver de 10 à 20 personnes satisfaites du produit, sans rejeter l’hypothèse de 60 % de satisfaits.

La fourchette est trop étendue pour que l’on puisse généraliser d’après cet échantillon de 25 personnes.

B. à la calculatriceToutes les réponses sont données pour le cas n = 100 sur les copies d’écran du manuel élève.Il est bon d’habituer les élèves à manipuler leur calculatrice pour obtenir la liste 2 des probabi-lités cumulées, puis lire cette liste ou trouver la somme comme proposée (plus facile).

Corrigés des exercices

à l’oral

5

a) X ∈ { 0 ; 1 ; 2 ; … ; 6 }

b) B ( 6 ; 0,5 )

6

a) X ∈ { 0 ; 1 ; 2 ; 3 ; 4 }

b) B ( 4 ; 16 )

8

1. a) X ∈ { 0 ; 1 ; 2 ; 3 }

b) 3 issues à exactement 1 succès et 3 issues à exactement 2 succès.

2. a) P (S-E-S ) = 0,42 × 0,6 = 0,096

b) P( X = 2 ) = 3 × 0,096 = 0,288

9

a) B ( 20 ; 0,6 )

b) E(X) = 20 × 0,6 = 12

Sur un grand nombre d’échantillons de 20 salariés de cette entreprise, on comptera en moyenne 12 employés par échantillon.

10

Page 107: Math 1ere STMG

Chapitre 7 loi binomiale – éChantillonnage 107

a) P( X = 10 ) ≈ 0,1171

b) P( X 10 ) ≈ 0,2447

On choisit au hasard 20 salariés, la probabilité que moins de 11 salariés soient employés est 0,2447 .

11

a) Vrai : on peut avoir 10 jetons rouges sur 20 .

b) Vrai : P(S) = 250

1000 = 0,25 .

c) Faux : B ( 20 ; 0,25 ) .

12

a) Vrai : l’échantillon est de taille 40 et la proba-bilité du succès « obtenir Pile » est égale à 0,5 .

b) Faux : a = 14 et b = 26 .

D’où l’intervalle de fluctuation à environ 95 % de la fréquence :

[ c ; d ] = ⎡

⎣⎢⎢1440

 ; ⎤

⎦⎥⎥

2640

= [0,35 ; 0,65 ] .

13

Loi binomiale B ( 50 ; 0,2 )

a = 5 et b = 16 d’où l’intervalle de fluctuation à environ 95 % de la fréquence :

[ c ; d ] = ⎡

⎣⎢⎢550

 ; ⎤

⎦⎥⎥

1650

= [ 0,1 ; 0,32 ] .

Fréquence observée : f = 0,31 .

Comme 0,31 ∈ [ 0,1 ; 0,32 ] , l’hypothèse p = 0,2 faite sur la proportion p n’est pas à rejeter.

14

a) 31,4 % des élèves de première dans l’acadé-mie étudient l’espagnol.

b) Loi binomiale B ( 50 ; 0,314 )

c) a = 9 et b = 22 : l’intervalle de fluctuation à environ 95 % de la fréquence :

[ c ; d ] = [ 0,18 ; 0,44 ] .

d) Fréquence observée dans ce lycée :

f = 2450

= 0,48 et f ∉ [ 0,18 ; 0,44 ] .

L’hypothèse sur la proportion p est rejetée et le proviseur devra changer l’organisation de l’enseignement de l’espagnol.

Pour s’entraîner

1 Loi binomiale

15

1. a) q = 1 – p = 0,7

b) Schéma de Bernoulli

E

E

E

S E S0 1 1 2 1 2 2 3

0,7

0,7

0,7

0,3

0,3 0,7 0,3

0,3

0,7 0,3 0,7 0,3 0,7 0,3

S

E

E

S

S E S

S

2. X suit la Loi binomiale B ( 3 ; 0,3 ) .

16

a) P( X = 3 ) = 0,027

P( X = 2 ) = 3 × 0,3² × 0,7 = 0,189

b) P( X 2 ) = 0,973

La probabilité d’obtenir un nombre de succès inférieur ou égal à 2 est 0,973 .

18

a) Loi binomiale B ( 3 ; 0,2 )

b) Loi de probabilité

k 0 1 2 3

P( X = k ) 0,512 0,384 0,096 0,008

c) P( X 1) = 1 – P( X = 0 ) = 1 – 0,512

= 0,488

19

1. Situation qui ne correspond pas à un schéma de Bernoulli, car on ne peut pas remettre la première part mangée.

Les trois épreuves répétées ne sont pas iden-tiques.

2. Loi binomiale B ( 6 ; 0,5 )

Page 108: Math 1ere STMG

108 Chapitre 7 loi binomiale – éChantillonnage

3. Loi binomiale B ( 10 ; 0,45 )

20

1. Loi binomiale B ( 4 ; 0,65 )

2. a) P(X = 3) ≈ 0,3845

La probabilité que trois personnes parmi les quatre interrogées soient satisfaites du dépan-nage est 0,3845 .

b) P( X 1 ) = 1 – P( X = 0 ) = 1 – 0,354 ≈ 0,9850

21

1. a) Cette commande donne à chaque fois un nombre entier égal à 1, 2, 3, ou 4 .

b) Probabilité d’obtenir 2 : p = 14

= 0,25 .

2. Loi simulée par cinq lancers du dé :

B ( 5 ; 0,25 ) .22

1. a) p = 13

b) Probabilité q d’obtenir les huit réponses exactes au QCM en répondant au hasard à chaque question :

q ≈ 1,5 × 10– 4 = 0,00015 .2. Répétition de huit questions avec pour chaque question la même probabilité de suc-

cès p = 13

.

D’où la loi binomiale suivie par X : B ( 8 ; 13

) .

3. a) { X = 4 } est l’événement : « Karim a bien répondu à la moitié des questions ».

Donc P( X = 4 ) = 0,170 .

Et P(X 4) = 0,275 (on effectue la somme des valeurs du tableau pour k = 4, 5, 6, 7 et 8 ).

23

1. P(S) = 36

= 0,5

Loi de X : B ( 8 ; 0,5 )

2. P(A) = P( X = 4 ) = 0,273

P(B) = P( X = 0 ) = 0,004

P(C) = P( X 5 ) = 0,8555

P(D) = P( X 4 ) = 1- P( X ≤ 3 )

= 1 – 0,3633 = 0,6367

24

P( X = 2 ) ≈ 0,234 et P ( X = 5 ) ≈ 0,094 .

P( X 3 ) ≈ 0,656

25

1. Loi binomiale B ( 30 ; 0,08 )

2. Événement { X = 5 } : « 5 bouteilles du lot de 30 ont une étiquette déchirée ».

P( X = 5 ) ≈ 0,0581

Événement { X = 8 } : « 8 bouteilles du lot de 30 ont une étiquette déchirée ».

P( X = 8 ) ≈ 0,00157

Événement { X 5 } : « dans un lot de 30 bou-teilles, au plus 5 bouteilles ont une étiquette déchirée ».

P(X 5) ≈ 0,971

3. a) E(X) = 30 × 0,08 = 2,4

Sur un très grand nombre de lots de 30 bou-teilles, le nombre moyen de bouteilles avec une étiquette déchirée est 2,4 .

26

P( X = 2 ) = 0,121 ; P( X = 4 ) = 0,251 ;

P( X = 8 ) = 0,016 .

Et E(X) = 4 .

27

1. Pour la variable aléatoire X la probabilité du succès est 0,8 .

Elle est supérieure à celle du succès de la variable aléatoire Y.

On aura, par exemple :

P( X = 15 ) > P( Y =15 ) .

Page 109: Math 1ere STMG

Chapitre 7 loi binomiale – éChantillonnage 109

On en déduit que la variable X est eprésentée par le diagramme 3 ; la variable Y est représen-tée par le diagramme 1.

2. On ne voit pas les 20 bâtons, car pour cer-taines valeurs de k, les probabilités P( X = k ) sont trop faibles.

3. Le diagramme 2 représente la loi binomiale B ( 20 ; 0,5 ) .

4. Valeur de la variable aléatoire ayant la plus grande probabilité :

diagramme 1 : Y = 4 ;

diagramme 2 : Z = 10 ;

diagramme 3 : X = 16 ;

5. E(X) = 16 ; E(Y) = 4 ; E(Z) = 10 .

Ici, l’espérance est égale à la valeur de la variable ayant la plus grande probabilité. Cependant, dans le cas général, l’espérance, qui est une moyenne, n’est pas une valeur de la variable aléatoire, qui est toujours un entier. On aura seulement une espérance proche de la valeur de la variable aléatoire ayant la plus grande probabilité.

28

1. Loi binomiale B ( 80 ; 0,75 )

P(A) = P ( X = 50 ) ≈ 0,0044

P(B) = P( X = 70 ) ≈ 0,0028

2. E(X) = 60

Sur un grand nombre d’échantillons de 80 personnes qui entrent dans le magasin, en moyenne 60 ont le bon publicitaire.

3. En passant à des échantillons de 100 per-sonnes, l’espérance devient 75 .

On retrouve le pourcentage de 75 % utilisé au départ.

29

1. X prend les valeurs entières de 0 à 8.

X suit la loi binomiale B ( 8 ; 0,1 ) .

2. a) P( X = 2 ) ≈ 0,1488

b) La probabilité que deux enfants sur les huit obtiennent un bonbon rouge est 0,1488 .

3. E(X) = 0,8 . Sur une longue durée, le nombre moyen de bonbons rouges obtenu chaque

jour est 0,8 (moins de 1 bonbon rouge par jour en moyenne).

2 échantillonnage et intervalle de fluctuation à 95 %

30

a) Tableau des résultats

1 2 3 4 5 61 2 3 4 5 6 72 3 4 5 6 7 83 4 5 6 7 8 94 5 6 7 8 9 105 6 7 8 9 10 116 7 8 9 10 11 12

A : « La somme est égale à 7 ».

B : « La somme est égale à 8 ».

C : « La somme est égale à 9 ».

P(A) = 6

36 ≈ 0,17  ; P(B) =

536

≈ 0,14 ;

P(C) = 4

36 ≈ 0,11 .

2. Au cours des deux séries de 100 lancers, les fréquences d’apparition de 7 , 8 ou 9 sont différentes d’une série à l’autre et différentes également des probabilités (fréquences théo-riques). Cela correspond à la notion de fluctua-tion d’échantillonnage.

31

1. a) a = 18 b) b = 30

c) L’intervalle [ a ; b ] = [18 ; 30 ] .

On dit que l’intervalle [ 18 ; 30 ] contient envi-ron 95 % des valeurs de X, pondérées par les probabilités.

2. Intervalle de fluctuation à environ 95 % de la fréquence correspondant à cette variable aléatoire X :

[ c ; d ] = [ 1840

 ; 3040

] = [ 0,45 ; 0,75 ]

33

Loi de X : loi binomiale B ( 40 ; 0,6 ) .

1. On utilise la table de valeurs donnée dans l’exercice 31 : J = [ 0,45 ; 0,75 ] .

Page 110: Math 1ere STMG

110 Chapitre 7 loi binomiale – éChantillonnage

2. On simule l’expérience de cette tombola.

Sur les 40 personnes interrogées, la proba-bilité que la fréquence observée du nombre de gagnants soit dans l’intervalle J est égale à 0,95 .

Autrement dit : sur 40 personnes ayant acheté un billet, la probabilité qu’il y en ait entre 18 et 30 à avoir un billet gagnant est d’environ 0,95.

34

1. a) Loi binomiale B ( 20 ; 0,4 )

[ a ; b ] = [ 4 ; 12 ]

Donc l’intervalle de fluctuation à environ 95 % de la fréquence est :

[ c ; d ] = [ 0,2 ; 0,6 ] .

b) Loi binomiale B ( 100 ; 0,4 )

[ a ; b ] = [ 31 ; 50 ]

[ c ; d ] = [ 0,31 ; 0,5 ]

c) Loi binomiale B ( 200 ; 0,4 )

[ a ; b ] = [67 ; 94 ]

[ c ; d ] = [ 0,335 ; 0,47 ]

d) Loi binomiale B ( 500 ; 0,4 )

[ a ; b ] = [179 ; 222 ]

[ c ; d ] = [ 0,358 ; 0,444 ]

2. Plus la taille de l’échantillon augmente, plus la longueur des intervalles [ c ; d ] diminue.

Les bornes se rapprochent de la fréquence théorique (probabilité de succès) égale à 0,4 .

35

1. a) Loi binomiale B ( 50 ; 0,5 )

[ a ; b ] = [ 18 ; 32 ]

[ c ; d ] = [ 0,36 ; 0,64 ]

b) Loi binomiale B ( 100 ; 0,5 )

[ a ; b ] = [ 40 ; 60 ]

[ c ; d ] = [ 0,4 ; 0,6 ]

c) Loi binomiale B ( 200 ; 0,5 ) .

[ a ; b ] = [86 ; 114 ] 

et [ c ; d ] = [ 0,43 ; 0,57 ] .

d) Loi binomiale B ( 400 ; 0,5 )

[ a ; b ] = [180 ; 220

[ c ; d ] = [ 0,45 ; 0,55 ]

2. a) m = 25 b) m = 50

c) m = 100 d) m = 200

36

1. X prend les valeurs entières de 0 à 15.

P(S) = 0,6

2. a) [ a ; b ] = [ 5 ; 13 ]

b) [ c ; d ] = ⎡

⎣⎢⎢515

 ; ⎤

⎦⎥⎥

1315

≈ [ 0,333 ; 0,867 ]

37

1. a) L’effectif de l’échantillon des personnes interrogées est n = 150 et P(S) = p = 0,4 .

D’où la loi binomiale B ( 150 ; 0,4 ) suivie par la variable aléatoire X.

b) [ a ; b ] = [ 48 ; 72 ]

[ c ; d ] = [ 0,32 ; 0,48 ]

2. Fréquence observée :

f = 47

150 ≈ 0,313 et f ∉ I .

3 Prise de décision

38

a) D’après l’exercice 36 : l’intervalle de fluctua-tion à environ 95 % de la fréquence est :

[ c ; d ] = ⎡

⎣⎢⎢515

 ; ⎤

⎦⎥⎥

1315

≈ [ 0,333 ; 0,867 ] .

Fréquence observée : f = 0,4 .

b) Comme f ∈ [ c ; d ] , l’hypothèse d’une pro-portion de 60 % de moins de 16 ans dans cette population de jeunes n’est pas à rejeter.

39

1. X prend les valeurs entières de 0 à 40.

P(S) = 0,7 .

2. a) [ a ; b ] = [ 22 ; 33 ]

b) [ c ; d ] = ⎡

⎣⎢⎢2240

 ; ⎤

⎦⎥⎥

3340

= [ 0,55 ; 0,825]

Page 111: Math 1ere STMG

Chapitre 7 loi binomiale – éChantillonnage 111

3. Fréquence observée : f = 0,82 .

Comme f ∈ [ c ; d ] , l’hypothèse p = 0,7 n’est pas à rejeter.

40

On utilise la loi binomiale B ( 200 ; 0,5 ), véri-fiée par la variable aléatoire X égale au nombre de voix du candidat Z pour cet échantillon de 200 bulletins.

L’intervalle de fluctuation à environ 95 % de la fréquence associée à cette variable aléatoire X est [ 0,43 ; 0,57 ] .

Fréquence observée : f = 0,45 .

Comme 0,45 ∈ [ 0,43  ; 0,57 ] , la prévision de l’institut n’est pas à rejeter.

4 1

Si X est le nombre de personnes qui achètent le nouveau produit dans cet échantillon, on suppose que X suit la loi binomiale B (120 ; 0,1) .

À partir de la table page 175, on obtient :

[ a ; b ] = [6 ; 19 ] ;

[ c ; d ] ≈ [ 0,05 ; 0,16 ] .

La fréquence observée est f = 7

120 ≈ 0,058 .

Comme f ∈ [ c ; d ] , on ne met pas en doute les résultats de l’étude de marché.

4 2

1. a) Loi binomiale B ( 50 ; 0,1 )

[ a ; b ] = [ 1 ; 9 ]

[ c ; d ] = [ 0,02 ; 0,18 ]

Fréquence des pièces défectueuses observée :

f = 0,2 et f ∉ [ 0,02 ; 0,18 ] .

On rejette l’hypothèse sur la proportion de pièces défectueuses, avec risque d’erreur de 5 % .

b) f = 6

50= 0,12 et f ∈ [ 0,02 ; 0,18 ] .

L’hypothèse sur la proportion de pièces défec-tueuses n’est pas à rejeter.

c) a) Loi binomiale B ( 120 ; 0,1 )

[ a ; b ] = [ 6 ; 19 ]

[ c ; d ] = [ 0,05 ; 0,16 ]

f = 21

120 = 0,175 et f ∉ [ 0,05 ; 0,16 ] .

On rejette l’hypothèse sur la proportion de pièces défectueuses, avec un risque d’erreur de 5 % .

2. Dans les cas a) et b) , la fréquence observée pour les pièces défectueuses est plus grande que celle attendue.

Le service qualité devra revoir la fabrication.

Pour approfondir

44

1. a) D’après l’énoncé, la variable aléatoire X suit la loi binomiale B ( 10 ; 0,8 ) .

b) P( X = 6 ) ≈ 0,0881 .

P( X 1 ) = 1 – P( X = 0 ) ≈ 0,9999 .

2. a) La variable aléatoire Y suit la loi binomiale B ( 120 ; 0,8 ) .

b) E(Y) = = 96

Si on considère un grand nombre d’échan-tillons de 120 salariés, le nombre de salariés satisfaits est en moyenne de 96.

c) P( Y = 96 ) ≈ 0,9071

3. a) Loi binomiale B ( 120 ; 0,8 )

[ a ; b ] = [ 87 ; 104 ]

[ c ; d ] = [ 0,725 ; 0,867 ]

b) Si 90 sur 120 sont réellement satisfaits :

f = 90

120 = 0,75 .

Comme f ∈ [ c ; d ] , on ne met pas en doute l’affirmation du directeur général.

46

1. a) En 2010,le taux de réussite au bac tech-nologique est de 81,4 % .

b) CM = 82,381, 4

≈ 1,0111

Soit une hausse de 1,1 % .

3. La taille de l’échantillon est n = 50 et la pro-babilité de succès est p = 0,823 .

Page 112: Math 1ere STMG

112 Chapitre 7 loi binomiale – éChantillonnage

D’où la loi binomiale B ( 50 ; 0,823 ) suivie par la variable aléatoire X égale au nombre de can-didats admis parmi les 50 interrogés.

4.a) P( X = 39 ) ≈ 0,1001 représente la proba-bilité que 39 candidats sur 50 soient admis au bac.

b) [ a ; b ] = [ 36 ; 46 ]

5. a) Dans ce groupe 76 % des candidats ont réussi.

b) Comme 38 ∈ [ 36 ; 46 ] , le résultat obtenu par ce groupe est conforme aux résultats nationaux.

Vers le Bac

4 9

Partie A1. Tableau des effectifs

tri sélectif

tri non sélectif total

consommateurs de produits bio. 84 9 93

de produits non bio. 126 81 207

total 210 90 300

2. a) B ∩ T : « le ménage consomme bio et pra-tique le tri sélectif ».

P( B ∩ T ) = 84300

= 0,28

B ∪ T : « le ménage consomme bio ou pratique le tri sélectif ».

P( B ∪ T ) = P(B) + P(T) – P( B ∩ T )

= +93

300210300

- 84

300 =

219300

= 0,73

B ∩ T :  « le ménage ne consomme pas de pro-duits bio et pratique le tri sélectif ».

P( B ∩ T ) = 126300

= 0,42

B ∩ T   : « le ménage consomme des produits bio et ne pratique pas le tri sélectif ».

P ( B ∩ T ) = 9

300 = 0,03

3. P(E) = 8493

≈ 0,903

Partie Ba) La situation correspond à un schéma de Ber-noulli de paramètres n = 3 et p = 0,28 , repré-senté par l’arbre pondéré ci-dessous :

E

E

E

S E S

0,72

0,72

0,72

0,28

0,28 0,72 0,28

0,28

0,72 0,28 0,72 0,28 0,72 0,28

S

E

E

S

S E S

S

b) P(A) = 0,28 3 ≈ 0,220

P(B) = 3 × 0,28 ² × 0,72 ≈ 0,169

P(C) = 1 – 0,72 3 ≈ 0,627

Partie C1. a) La variable aléatoire X suit la loi binomiale B ( 200 ; 0,28 ) .

b) { X = 50 }: « 50 ménages ont répondu oui à la question posée ».

P(X) = 50 ≈ 0,0412

{ X ≤ 60 } : « au plus 60 ménages ont répondu oui à la question posée ».

P( X 60 ) ≈ 0,7625

2. [ c ; d ] = [ 44

200 ;

69200

] = [ 0,22 ; 0,345 ]

3. f = 110200

= 0,55 . Comme f ∉ [ c ; d ] , on peut

mettre en doute l’hypothèse d’une proportion p = 0,28 .

50

Partie A1. Données en L1 et effec-tifs en L2

Page 113: Math 1ere STMG

Chapitre 7 loi binomiale – éChantillonnage 113

Moyenne : x ≈ 57,2 .

Écart type : σ ≈ 0,362 .

2. Parmi les balles testées, 940 sont homolo-guées, soit 94 % .

Partie B1. a) X suit la loi B ( 150 ; 0,94 ) .

b) On utilise la calculatrice :

P(X = 140) ≈ 0,1223

et P(X = 145) ≈ 0,0584 .

c) { X 140 } : « le nombre de balles homolo-guées dans ce lot de 150 balles est inférieur ou égal à 140 » (« au plus 140 »).

P( X 140 ) ≈ 0,413

2. E(X) = 141

Si on effectue un grand nombre d’achats de lots de 150 balles, le nombre de balles homolo-guées sera, en moyenne, de 141 balles par lot.

51

1.a) Arbre pondéré

E

E

0,3

0,3

0,7 0,3 0,7

0,7

S E

S

S

P(A) = 0,72 = 0,49

et P(B) = 1 – 0,3² = 0,91 .

2. X suit la loi binomiale B ( 130 ; 0,7 ) .

[ c ; d ] = [ 81

130 ;

101130

] ≈ [ 0,623 ; 0,777 ]

3. Le résultat de l’enquête donne une fré-

quence f = 92

130 ≈ 0,708 du nombre de per-

sonnes qui regrettent la boulangerie.

Comme f ∈ [ c ; d ] , on ne met pas en doute l’af-firmation de l’association de consommateurs.

52

1. a) Tableau des effectifs

B B total

C 6 5 11

C 9 80 89

total 15 85 100

b) P(D) = + +6 9 5100

= 0,2

c) D : « la cliente prend un chemisier ayant au moins un défaut ».

D   : «  la cliente prend un chemisier n’ayant aucun défaut ».

P( D ) = 1 – P(D) = 0,8 et P(E) = 0,14 .

2. a) La variable aléatoire X suit la loi binomiale B ( 3 ; 0,2 )

b) Loi de probabilité

k 0 1 2 3

P( X = k ) 0,512 0,384 0,096 0,008

3. a) Y suit la loi binomiale B ( 60 ; 0,2 ) .

b) E(Y) = 12

Sur un très grand nombre de lots de 60 chemi-siers, 12 chemisiers ont en moyenne au moins un défaut.

c) Douze chemisiers sont vendus à moitié prix.

12 × 10 + 48 × 20 = 1 080

Montant moyen de la facture : 1 080 € .

Page 114: Math 1ere STMG
Page 115: Math 1ere STMG
Page 116: Math 1ere STMG
Page 117: Math 1ere STMG
Page 118: Math 1ere STMG
Page 119: Math 1ere STMG
Page 120: Math 1ere STMG